You are on page 1of 99

Commissioner

Jose

T.

Almonte,

petitioner v

Honorable

Conrado

Vasquez,

respondent

Facts:
Ombudsman Vasquez required Rogado and Rivera of Economic Intelligence and Investigation Bureau
(EIIB) to produce all documents relating to Personal Service Funds yr. 1988 and all evidence for the whole
plantilla of EIIB for 1988. The subpoena duces tecum was issued in connection with the investigation of
funds representing savings from unfilled positions in the EIIB which were legally disbursed. Almonte and
Perez denied the anomalous activities that circulate around the EIIB office. They moved to quash the
subpoena duces tecum. They claim privilege of an agency of the Government.
Petitioner Jose T. Almonte was formerly Commissioner of the EIIB, while Villamor C. Perez is Chief of the
EIIB's Budget and Fiscal Management Division. The subpoena duces tecum was issued by the
Ombudsman in connection with his investigation of an anonymous letter alleging that funds representing
savings from unfilled positions in the EIIB had been illegally disbursed. The letter, purporting to have
been written by an employee of the EIIB and a concerned citizen, was addressed to the Secretary of
Finance, with copies furnished several government offices, including the Office of the Ombudsman.
Issue:
Whether or not an Ombudsman can oblige the petitioners by virtue of subpoena duces tecum to provide
documents relating tWhether or not an Ombudsman can oblige the petitioners by virtue of subpoena
duces tecum to provide documents relating to personal service and salary vouchers of EIIB employers.o
personal service and salary vouchers of EIIB employers.
Ruling:
In the case at bar, there is no claim that military or diplomatic secrets will be disclosed by the production
of records pertaining to the personnel of the EIIB. Indeed, EIIB's function is the gathering and evaluation
of intelligence reports and information. "illegal activities affecting the national economy, such as, but not
limited to, economic sabotage, smuggling, tax evasion, dollar salting." Consequently, while in cases which
involve state secrets it may be sufficient to determine from the circumstances of the case that there is
reasonable danger that compulsion of the evidence will expose military matters without compelling
production, no similar excuse can be made for a privilege resting on other considerations.
WHEREFORE, the petition is DISMISSED, but it is directed that the inspection of subpoenaed
documents be made personally in camera by the Ombudsman, and with all the safeguards outlined in this
decision.

ASIAN SURETY and INSURANCE COMPANY, INC., petitioner


v
HON. JOSE HERRERA, respondent

Facts:
Petition to quash and annul a search warrant issued by respondent Judge Jose Herrera of the City Court
of Manila, and to command respondents to return immediately the documents, papers, receipts and
records alleged to have been illegally seized thereunder by agents of the National Bureau of Investigation
(NBI) led by respondent Celso Zoleta, Jr.
On October 27, 1965, respondent Judge Herrera, upon the sworn application of NBI agent Celso Zoleta,
Jr. supported by the deposition of his witness, Manuel Cuaresma, issued a search warrant in connection
with an undocketed criminal case for estafa, falsification, insurance fraud, and tax evasion, against the
Asian Surety and Insurance Co., a corporation duly organized and existing under the laws of the
Philippines, with principal office at Room 200 Republic Supermarket Bldg., Rizal Avenue, Manila.
Armed with the search warrant Zoleta and other agents assigned to the Anti-graft Division of the NBI
entered the premises of the Republic Supermarket Building and served the search warrant upon Atty.
Alidio of the insurance company, in the presence of Mr. William Li Yao, president and chairman of the
board of directors of the insurance firm. After the search they seized and carried away two (2) carloads of
documents, papers and receipts.
Issue:
Whether or not the search warrant is void.
Ruling:
In the case at bar, the search warrant was issued for four separate and distinct offenses of : (1) estafa, (2)
falsification, (3) tax evasion and (4) insurance fraud, in contravention of the explicit command of Section
3, Rule 126, of the Rules providing that: "no search warrant shall issue for more than one specific offense."
PREMISES CONSIDERED, petition is hereby granted; the search warrant of October 27, 1965, is nullified
and set aside, and the respondents are hereby ordered to return immediately all documents, papers and
other objects seized or taken thereunder. Without costs.

Malacat v Court of Appeals

Facts:
On August 29, 1990 at about 6:30 in the evening, allegedly in response to bomb threats reported seven
days earlier, Rodolfo Yu of the Western Police District, Metropolitan Police Force of the Integrated
National Police, Police Station No. 3, Quiapo, Manila, was on foot patrol with three other police officers

(all of them in uniform) along Quezon Boulevard, Quiapo, Manila, near the Mercury Drug store at Plaza
Miranda. They chanced upon two groups of Muslim-looking men, with each group, comprised of three to
four men, posted at opposite sides of the corner of stop and frisk, where a warrant and seizure can be
effected without necessarily being preceded by an arrest and whose object is either to maintain the
status quo momentarily while the police officer seeks to obtain more information; and that the seizure of
the grenade from Malacat was incidental to a lawful arrest. The trial court thus found Malacat guilty of the
crime of illegal possession of explosives under Section 3 of PD 1866, and sentenced him to suffer the
penalty of not less than 17 years, 4 months and 1 day of Reclusion Temporal, as minimum, and not more
than 30 years of Reclusion Perpetua, as maximum. On 18 February 1994, Malacat filed a notice of appeal
indicating that he was appealing to the Supreme Court. However, the record of the case was forwarded to
the Court of Appeals (CA-GR CR 15988). In its decision of 24 January 1996, the Court of Appeals affirmed
the trial court. Manalili filed a petition for review with the Supreme Court.
Issue:
Whether the search made on Malacat is valid, pursuant to the exception of stop and frisk.
Ruling:
The trial court ruled that the warrantless search and seizure of petitioner was akin to a stop and frisk,
where a warrant and seizure can be effected without necessarily being preceded by an arrest and whose
object is either to maintain the status quo momentarily while the police officer seeks to obtain more
information. Probable cause was not required as it was not certain that a crime had been committed,
however, the situation called for an investigation, hence to require probable cause would have been
premature. The RTC emphasized that Yu and his companions were confronted with an emergency, in
which the delay necessary to obtain a warrant, threatens the destruction of evidence and the officers had
to act in haste, as petitioner and his companions were acting suspiciously, considering the time, place
and reported cases of bombing. Further, petitioners group suddenly ran away in different directions as
they saw the arresting officers approach, thus it is reasonable for an officer to conduct a limited search,
the purpose of which is not necessarily to discover evidence of a crime, but to allow the officer to pursue
his investigation without fear of violence. The trial court then ruled that the seizure of the grenade from
petitioner was incidental to a lawful arrest, and since petitioner later voluntarily admitted such fact to the
police investigator for the purpose of bombing the Mercury Drug Store, concluded that sufficient
evidence existed to establish petitioners guilt beyond reasonable doubt.
WHEREFORE, the challenged decision of the Seventeenth Division of the Court of Appeals in CA-G.R. CR
No. 15988 is SET ASIDE for lack of jurisdiction on the part of said Court and, on ground of reasonable
doubt, the decision of 10 February 1994 of Branch 5 of the Regional Trial Court of Manila is REVERSED
and petitioner SAMMY MALACAT y MANDAR is hereby ACQUITTED and ORDERED immediately
released from detention, unless his further detention is justified for any other lawful cause.
PEOPLE

V
RUBEN MONTILLA

Facts:
Ruben Montilla, was charged for violating Section 4, Article 2 of the Dangerous Drugs Act of 1972, R. A.
No. 6425, as amended by R. A. No. 7659 in an information which alleges that on or about June 20, 1994,
at Brgy. Salitran, Dasmarinas, Cavite, the above named accused, not being authorized by law, did then
and there willfully, unlawfully and feloniously, administer, transport and deliver 28 kilos of dried
marijuana leaves which are considered prohibited drugs.
Issue:
Whether the warrantless search conducted on appellant invalidates the evidence obtained from him?
Ruling:
A legitimate warrantless arrest necessarily cloaks the arresting officer with authority to validly search and
seize from the offender (1) dangerous weapons and (2) those that may be used as proof of the commission
of an offense. On the defense argument that the warrantless search conducted on appellant invalidates the
evidence obtained from him, still the search on his belongings and the consequent confiscation of the
illegal drugs as aresult thereof was justified as a search incidental to a lawful arrest under Section 5 (a)
Rule 113 of the Rules of Court.
THE PEOPLE OF THE PHILIPPINES, plaintiff-appellee,
vs.
SANTIAGO SY JUCO, defendant.

Facts:
The crime alleged is fraud of revenue against the Government. Pursuant to a search warrant issued, the
officers searched the building occupied by Santiago Sy Juco. In the process, the authorities seized, among
others, an art metal filing cabinet claimed by Atty. Remo to be his and contained some letters, documents
and papers belonging to his clients. Also, books belonging to Salakam Lumber Co., Inc., were seized.
Issue:
1. Is the search warrant in question valid or not, taking into consideration the provisions of the law and of
the Constitution relative thereto?
2. Does the art metal filing cabinet seized by the agents of the Bureau of Internal Revenue belong to
Santiago Sy Juco or to Teopisto B. Remo?
Ruling:
The search and seizure was not valid. It is not stated in the affidavit that the books, documents or records
referred to therein are being used or are intended to be used in the commission of fraud against the

Government and, notwithstanding the lack of such allegation; the warrant avers that they are actually
being used for such purpose.
Also, it assumes that the entire building is occupied by Santiago Sy Juco, when the only ground upon
which such assumption is based is the BIR agent's statement which is mere hearsay (coming from an
informant) and when in fact part thereof was occupied by Atty. Remo. It was not asked that the things
belonging to Atty. Remo and to others also be searched and seized.
For all the foregoing reasons, and finding that the errors assigned by the appellant are very well founded,
the appealed judgment is reversed, and it is ordered that the art metal filing cabinet, together with the key
thereof seized by the internal revenue agent by virtue of the judicial warrant in question, which is hereby
declared null and void, be immediately returned unopened to the appellant; and that a copy of this
decision be sent to the Solicitor-General for him to take action, if he deems it justified, upon careful
investigation of the facts, against the internal revenue agent or agents who obtained and executed the
warrant in question, in accordance with the provisions of article 129 of the Revised Penal Code, without
special pronouncement as to costs.
GOVERNMENT SERVICE INSURANCE SYSTEM, Cebu City Branch, petitioner,
vs.
MILAGROS O. MONTESCLAROS, respondent.

FACTS:
Sangguniang Bayan member Nicolas Montesclaros married Milagros Orbiso on 10 July
1983.] Nicolas was a 72- year old widower when he married Milagros who was then 43 years old. On 4
January 1985, Nicolas filed with the GSIS an application for retirement benefits effective 18 February
1985 under Presidential Decree No. 1146 or the Revised Government Service Insurance Act of 1977 (PD
1146). In his retirement application, Nicolas designated his wife Milagros as his sole beneficiary.[Milagros
filed with GSIS a claim for survivorship pension under PD 1146. On 8 June 1992, GSIS denied the claim
because under Section 18 of PD 1146, the surviving spouse has no right to survivorship pension if the
surviving spouse contracted the marriage with the pensioner within three years before the pensioner
qualified for the pension.] According to GSIS, Nicolas wed Milagros on 10 July 1983, less than one year
from his date of retirement on 17 February 1984.
On 2 October 1992, Milagros filed with the trial court a special civil action for declaratory relief
questioning the validity of Section 18 of PD 1146 disqualifying her from receiving survivorship pension
and was granted of the survivorship pension.
GSIS appealed to the COA, which affirmed the decision of the trial court.

In a letter dated 10 January 2003, Milagros informed the Court that she already accepted the
decision of the GSIS and is not interested anymore of the survivorship pension. GSIS wants the court to
decide on the case basing on the merit of the letter given by Milagros(respondent).
ISSUE:
1.

Whether or not the decision of Section 18 of PD 1146 is unconstitutional.


RULING:
SEC. 18. Death of a Pensioner. Upon the death of a pensioner, the primary beneficiaries shall
receive the applicable pension mentioned under paragraph (b) of section seventeen of this
Act: Provided, That, the dependent spouse shall not be entitled to said pension if his
marriage with the pensioner is contracted within three years before the pensioner
qualified for the pension. When the pensioner dies within the period covered by the lump sum, the
survivorship pension shall be paid only after the expiration of the said period. This shall also apply to the
pensioners living as of the effectivity of this Act, but the survivorship benefit shall be based on the
monthly pension being received at the time of death.
The main question for resolution is the validity of the proviso in Section 18 of PD 1146, which
proviso prohibits the dependent spouse from receiving survivorship pension if such dependent spouse
married the pensioner within three years before the pensioner qualified for the pension (the proviso).
We hold that the proviso, which was the sole basis for the rejection by GSIS of Milagros claim, is
unconstitutional because it violates the due process clause. The proviso is also discriminatory and denies
equal protection of the law.
WHEREFORE, the petition is DENIED for want of merit. We declare VOID for being violative of
the constitutional guarantees of due process and equal protection of the law the proviso in Section 18 of
Presidential Decree No. 1146, which proviso states that the dependent spouse shall not be entitled to said
pension if his marriage with the pensioner is contracted within three years before the pensioner qualified
for the pension. The Government Service Insurance System cannot deny the claim of Milagros O.
Montesclaros for survivorship benefits based on this invalid provison.
No pronouncement as to costs.

BENJAMIN V. KHO and ELIZABETH ALINDOGAN, petitioners,


v HON. ROBERTO L. MAKALINTAL and NATIONAL BUREAU OF INVESTIGATION, respondents.

FACTS:
On May 15, 1990, NBI Agent Max B. Salvador applied for the issuance of search warrants by the
respondent Judge against Banjamin V. Kho, now petitioner, in his residence at No. 45 Bb. Ramona
Tirona St., BF Homes, Phase I, Paranaque. On the same day, Eduardo T. Arugay, another NBI agent,
applied with the same court for the issuance of search warrants against the said petitioner in his house at
No. 326 McDivitt St., Bgy. Moonwalk, Paranaque. The search warrants were applied for after teams of
NBI agents had conducted a personal surveillance and investigation in the two houses referred to on the
basis of confidential information they received that the said places were being used as storage centers for
unlicensed firearms and chop-chop vehicles. On the same day, the respondent Judge conducted the
necessary examination of the applicants and their witnesses, after which he issued Search Warrant Nos.
90-11, 90-12, 90-13, 90-14, and 90-15.
On the following day, May 16, 1990, NBI conducted the simultaneous searches on the said
residences of the petitioner (Kho) and they were able to confiscate the above mention objects stated in the
warrant and the simultaneous searches also resulted in the confiscation of various radio and
telecommunication equipment. The confiscated items were verified in Camp Crame and were proven that
all of them are unlicensed.
Petitioner (Kho) question the validity of the warrant and filed a Motion to Quash the previous
decision.
ISSUES:
1.

Whether or not the issuance of the search warrant by the respondent Judge valid?

2.

Whether or not the Motion to Quash filed by the petitioner (Kho) alleging that there was an abuse
enforcement of the challenge search warrant valid?

3.

Whether or not the Petitioners sought to restrain the respondent National Bureau of Investigation (NBI)
from using the objects seized by virtue of such warrants in any case or cases filed or to be filed against
them and to return immediately the said items valid.
RULING:
The Court believes, and so holds, that the said warrants comply with Constitutional and statutory
requirements. The law does not require that the things to be seized must be described in precise and
minute detail as to leave no room for doubt on the part of the searching authorities. Otherwise, it would
be virtually impossible for the applicants to obtain a warrant as they would not know exactly what kind of
things they are looking for. Since the element of time is very crucial in criminal cases, the effort and time
spent in researching on the details to be embodied in the warrant would render the purpose of the search
nugatory.

The question of whether there was abuse in the enforcement of the challanged search warrants is
not within the scope of a Motion to Quash. In a Motion to Quash, what is assailed is the validity of the
issuance of the warrant. The manner of serving the warrant and of effecting the search are not an issue to
be resolved here. As aptly opined and ruled by the respondent Judge, petitioners have remedies under
pertinent penal, civil and administrative laws for their problem at hand, which cannot be solved by their
present motion to quash.
Considering that cases for Illegal Possession of Firearms and Explosives and Violation of Section 3 in
relation to Section 14 of Republic Act No. 6539, otherwise known as the Anti-Carnapping Act of 1972,
have been instituted against the petitioners, the petition for mandamus with preliminary and mandatory
injunction to return all objects seized and to restrain respondent NBI from using the said objects as
evidence, has become moot and academic.
WHEREFORE, for want of merit and on the ground that it has become moot and academic, the petition
at bar is hereby DISMISSED. No pronoucement as to costs.

G.R. No. 82585 November 14, 1988


MAXIMO V. SOLIVEN, ANTONIO V.ROCES, FREDERICK K. AGCAOLI, and
GODOFREDO L. MANZANAS, petitioners,
vs.
THE HON. RAMON P. MAKASIAR, Presiding Judge of the Regional Trial Court of
Manila, Branch 35, UNDERSECRETARY SILVESTRE BELLO III, of the Department
of Justice, LUIS C. VICTOR, THE CITY FISCAL OF MANILA and PRESIDENT
CORAZON C. AQUINO, respondents.
G.R. No. 82827 November 14, 1988
LUIS D. BELTRAN, petitioner,
vs.
THE HON. RAMON P. MAKASIAR, Presiding Judge of Branch 35 of the Regional
Trial Court, at Manila, THE HON. LUIS VICTOR, CITY FISCAL OF MANILA, PEOPLE
OF THE PHILIPPINES, SUPERINTENDENT OF THE WESTERN POLICE DISTRICT,
and THE MEMBERS OF THE PROCESS SERVING UNIT AT THE REGIONAL TRIAL
COURT OF MANILA, respondents.
G.R. No. 83979 November 14, 1988.
LUIS D. BELTRAN, petitioner,
vs.

EXECUTIVE SECRETARY CATALINO MACARAIG, SECRETARY OF JUSTICE


SEDFREY ORDOEZ, UNDERSECRETARY OF JUSTICE SILVESTRE BELLO III,
THE CITY FISCAL OF MANILA JESUS F. GUERRERO, and JUDGE RAMON P.
MAKASIAR, Presiding Judge of Branch 35 of the Regional Trial Court, at Manila,
respondents.
Angara, Abello, Concepcion, Regala and Cruz for petitioners in G.R. No. 82585.
Perfecto V. Fernandez, Jose P. Fernandez and Cristobal P. Fernandez for
petitioner in G.R. Nos. 82827 and 83979.

FACTS:
. On March 30, 1988, the Secretary of Justice denied petitioners' motion for reconsideration and
upheld the resolution of the Undersecretary of Justice sustaining the City Fiscal's finding of a prima facie
case against petitioners. A second motion for reconsideration filed by petitioner Beltran was denied by the
Secretary of Justice on April 7, 1988. On appeal, the President, through the Executive Secretary, affirmed
the resolution of the Secretary of Justice on May 2, 1988. The motion for reconsideration was denied by
the Executive Secretary on May 16, 1988. With these developments, petitioners' contention that they have
been denied the administrative remedies available under the law has lost factual support.
Petitioner Beltran also said that the issuance of the warrant of arrest is not valid because the
judge did not personally examined the complainant and witnesses.
.On the other hand, Petitioner Beltran argues that "the reasons which necessitate presidential
immunity from suit impose a correlative disability to file suit." He contends that if criminal proceedings
ensue by virtue of the President's filing of her complaint-affidavit, she may subsequently have to be a
witness for the prosecution, bringing her under the trial court's jurisdiction. This, continues Beltran,
would in an indirect way defeat her privilege of immunity from suit, as by testifying on the witness stand,
she would be exposing herself to possible contempt of court or perjury.
So, the petitioners raised 3 questions to contend if there is a grave abuse of discretion amounting to
lack or excess of jurisdiction on the part of the public respondents, respectively:
(1) whether or not petitioners were denied due process when informations for libel were filed against
them although the finding of the existence of a prima facie case was still under review by the Secretary of
Justice and, subsequently, by the President; (2) whether or not the constitutional rights of Beltran were
violated when respondent RTC judge issued a warrant for his arrest without personally examining the
complainant and the witnesses, if any, to determine probable cause; and (3) whether or not the President
of the Philippines, under the Constitution, may initiate criminal proceedings against the petitioners
through the filing of a complaint-affidavit.
ISSUE:

Whether or not there is a grave abuse of discretion amounting to lack or excess of jurisdiction on
the part of the public respondent.
RULING:
The first question was rendered moot and academic. The allegation of denial of due process of law
in the preliminary investigation is negated by the fact that instead of submitting his counter- affidavits, he
filed a "Motion to Declare Proceedings Closed," in effect waiving his right to refute the complaint by filing
counter-affidavits. Due process of law does not require that the respondent in a criminal case actually file
his counter-affidavits before the preliminary investigation is deemed completed. All that is required is
that the respondent be given the opportunity to submit counter-affidavits if he is so minded.
The second question, court interpreted the provision under Article III, sec 2, which states that, no
search warrant or warrant of arrest shall issue except upon probable cause to be determined personally by
the judge after examination under oath or affirmation of the complainant and the witnesses he may
produce, and particularly describing the place to be searched and the persons or things to be seized.
The court said that, what the Constitution underscores is the exclusive and personal responsibility of
the issuing judge to satisfy himself of the existence of probable cause. In satisfying himself of the existence
of probable cause for the issuance of a warrant of arrest, the judge is not required to personally examine
the complainant and his witnesses. Following established doctrine and procedure, he shall: (1) personally
evaluate the report and the supporting documents submitted by the fiscal regarding the existence of
probable cause and, on the basis thereof, issue a warrant of arrest; or (2) if on the basis thereof he finds no
probable cause, he may disregard the fiscal's report and require the submission of supporting affidavits of
witnesses to aid him in arriving at a conclusion as to the existence of probable cause.
The third question, court said that, the rationale for the grant to the President of the privilege of
immunity from suit is to assure the exercise of Presidential duties and functions free from any hindrance
or distraction, considering that being the Chief Executive of the Government is a job that, aside from
requiring all of the office holder's time, also demands undivided attention.
The petitions fail to establish that public respondents, through their separate acts, gravely abused
their discretion as to amount to lack of jurisdiction. Hence, the writs of certiorari and prohibition prayed
for cannot issue.
WHEREFORE, finding no grave abuse of discretion amounting to excess or lack of jurisdiction on
the part of the public respondents, the Court Resolved to DISMISS the petitions in G. R. Nos. 82585,
82827 and 83979. The Order to maintain the status quo contained in the Resolution of the Court en banc
dated April 7, 1988 and reiterated in the Resolution dated April 26, 1988 is LIFTED.

G.R.No. 74869

July 6, 1988

PEOPLE OF THE PHILIPPINES, plaintiff-appellee,

vs.
IDEL AMINNUDIN y AHNI, defendant-appellant.
The Solicitor General for plaintiff-appellee.
Herminio T. Llariza counsel de-officio for defendant-appellant.

CRUZ, J.:
FACTS:
Idel Aminnudin was arrested on June 25, 1984, shortly after disembarking from the M/V
Wilcon, in Iloilo City. The PC officers who were in fact waiting for him simply accosted him, inspected his
bag and finding what looked liked marijuana leaves took him to their headquarters for investigation. The
two bundles of suspect articles were confiscated from him and later taken to the NBI laboratory for
examination. When they were verified as marijuana leaves, an information for violation of the Dangerous
Drugs Act was filed against him.
In his defense, Aminnudin disclaimed the marijuana, averring that all he had in his bag was his
clothing consisting of a jacket, two shirts and two pairs of pants. His bag was confiscated without a search
warrant. At the PC headquarters, he was manhandled to force him to admit he was carrying the
marijuana, the investigator hitting him with a piece of wood in the chest and arms even as he parried the
blows while he was still handcuffed. The trial court was not convinced of the statement of the Aminnudin
(accused-appellant) because of lack of proof to show that he was really manhandled to force him to admit
the crime and trial court said that it was valid through Rule 113, sec 6(b) the Rules of Court on warrantless
arrests. This made the search also valid as incidental to a lawful arrest.
ISSUE:
Whether or not the application of Rule 113, sec 6(b) the Rules of Court on warrantless arrests is
valid.
Whether or not the decision of the Trial court in convicting the accused-appellant valid.
RULING:
In the case at bar, there was no warrant of arrest or search warrant issued by a judge after personal
determination by him of the existence of probable cause. Contrary to the averments of the government,
the accused-appellant was not caught in flagrante nor was a crime about to be committed or had just been
committed to justify the warrantless arrest allowed under Rule 113 of the Rules of Court. Even expediency
could not be invoked to dispense with the obtention of the warrant as in the case of Roldan v. Arca, 24 for
example. Here it was held that vessels and aircraft are subject to warrantless searches and seizures for
violation of the customs law because these vehicles may be quickly moved out of the locality or
jurisdiction before the warrant can be secured.
Without the evidence of the marijuana allegedly seized from Aminnudin, the case of the prosecution
must fall. That evidence cannot be admitted, and should never have been considered by the trial court for

the simple fact is that the marijuana was seized illegally. It is the fruit of the poisonous tree, to use Justice
Holmes' felicitous phrase. The search was not an incident of a lawful arrest because there was no warrant
of arrest and the warrantless arrest did not come under the exceptions allowed by the Rules of Court.
Hence, the warrantless search was also illegal and the evidence obtained thereby was inadmissible.
We find that with the exclusion of the illegally seized marijuana as evidence against the accusedappellant, his guilt has not been proved beyond reasonable doubt and he must therefore be discharged on
the presumption that he is innocent.
ACCORDINGLY, the decision of the trial court is REVERSED and the accused-appellant is
ACQUITTED. It is so ordered.

THE PEOPLE OF THE PHILIPPINES, plaintiff-appellee,


vs.
ROSA ARUTA y MENGUIN, accused-appellant.

ROMERO, J.:
FACTS:

On December 13, 1988, P/Lt. Abello was tipped off by his informant, known only as Benjie, that a
certain Aling Rosa would be arriving from Baguio City the following day, December 14, 1988, with a
large volume of marijuana. Acting on said tip, P/Lt. Abello assembled a team composed of P/Lt. Jose
Domingo, Sgt. Angel Sudiacal, Sgt. Oscar Imperial, Sgt. Danilo Santiago and Sgt. Efren Quirubin.
Having ascertained that accused-appellant was Aling Rosa, the team approached her and introduced
themselves as NARCOM agents. When P/Lt. Abello asked Aling Rosa about the contents of her bag, the
latter handed it to the former. Upon inspection, the bag was found to contain dried marijuana leaves
packed in a plastic bag marked Cash Katutak. Upon examination of the seized marijuana specimen at
the PC/INP Crime Laboratory, Camp Olivas, Pampanga, P/Maj. Marlene Salangad, a Forensic Chemist,
prepared a Technical Report stating that said specimen yielded positive results for marijuana, a
prohibited drug. She was charged with violating the Section 4, Article II of Republic Act No. 6425 or the
Dangerous Drugs Act and sentenced her to life imprisonment and to pay a fine of twenty thousand
(P20,000.00) pesos without subsidiary imprisonment in case of insolvency.
On her defence, accused-appellant reputed the above accusations against her and said that immediately
prior to her arrest, she had just come from Choice Theater where she watched the movie Balweg. While
about to cross the road, an old woman asked her help in carrying a shoulder bag. In the middle of the
road, Lt. Abello and Lt. Domingo arrested her and asked her to go with them to the NARCOM Office.
During investigation at said office, she disclaimed any knowledge as to the identity of the woman and
averred that the old woman was nowhere to be found after she was arrested. Moreover, she added that no
search warrant was shown to her by the arresting officers.
ISSUE:
1.

Whether or not the search and seizure conducted by P/Lt. Abello and his team valid.
RULING:
Accused-appellant Aruta cannot be said to be committing a crime. Neither was she about to commit
one nor had she just committed a crime. Accused-appellant was merely crossing the street and was not
acting in any manner that would engender a reasonable ground for the NARCOM agents to suspect and
conclude that she was committing a crime. It was only when the informant pointed to accused-appellant
and identified her to the agents as the carrier of the marijuana that she was singled out as the suspect.
The NARCOM agents would not have apprehended accused-appellant were it not for the furtive finger of
the informant because, as clearly illustrated by the evidence on record, there was no reason whatsoever
for them to suspect that accused-appellant was committing a crime, except for the pointing finger of the
informant. This the Court could neither sanction nor tolerate as it is a clear violation of the constitutional
guarantee against unreasonable search and seizure. Neither was there any semblance of any compliance
with the rigid requirements of probable cause and warrantless arrests.
Consequently, there was no legal basis for the NARCOM agents to effect a warrantless search of
accused-appellants bag, there being no probable cause and the accused-appellant not having been
lawfully arrested. Stated otherwise, the arrest being incipiently illegal, it logically follows that the

subsequent search was similarly illegal, it being not incidental to a lawful arrest. The constitutional
guarantee against unreasonable search and seizure must perforce operate in favor of accusedappellant. As such, the articles seized could not be used as evidence against accused-appellant for these
are fruits of a poisoned tree and, therefore, must be rejected, pursuant to Article III, Sec. 3(2) of the
Constitution.
Unreasonable searches and seizures are the menace against which the constitutional guarantees afford
full protection. While the power to search and seize may at times be necessary to the public welfare, still it
may be exercised and the law enforced without transgressing the constitutional rights of the citizens, for
the enforcement of no statute is of sufficient importance to justify indifference to the basic principles of
government.
WHEREFORE, in view of the foregoing, the decision of the Regional Trial Court, Branch 73,
Olongapo City, is hereby REVERSED and SET ASIDE. For lack of evidence to establish her guilt beyond
reasonable doubt, accused-appellant ROSA ARUTA Y MENGUIN is hereby ACQUITTED and ordered
RELEASED from confinement unless she is being held for some other legal grounds. No costs.
SO ORDERED.

ORMOC SUGAR COMPANY, INC.


VS.
THE TREASURER OF ORMOC CITY
G.R. No. L-23794
Facts:

The Municipal Board of Ormoc City passed Ordinance No. 4, Series of 1964 imposing any of all
productions of centrifugal sugar milled at the Ormoc Sugar Co. Inc., in Ormoc City a municipal tax
equivalent to 1% per export sale to the United States and other foreign countries.
Ormoc Sugar Company paid a total of P12, 087.50 under protest and subsequently filed a case before the
Court of 1st Instance of Leyte for being unconstitutional as it violates the equal protection clause (Sec. 1 [1],
Art. III) and the rule of uniformity of taxation (Sec. 22 [1], Art. VI).
On August 6, 1964, the Court rendered a decision that upheld the constitutionality of the ordinance and
declared the taxing power of defendant chartered city broadened by the Local Autonomy Act to include all
other forms of taxes, licenses or fees not excluded in its charter.
Issue:
Whether or not there has been a violation of equal protection.

Ruling:
Yes. The ordinance is discriminatory for its taxes only the Ormoc Sugar Company and none other. The
taxing ordinance should not be singular and exclusive as to exclude any subsequently sugar central from
the coverage of the tax. Even later when a similar company will be set up, it cannot be subject to tax
because the ordinance expressly points only the Ormoc Sugar Company.
WHEREFORE, the decision appealed from is hereby reversed, the challenged ordinance is declared
unconstitutional and the defendants-appellees are hereby ordered to refund the P12, 087.50 plaintiffappellant paid under protest. No costs. So ordered

YOUSEF AL-GHOUL, ET AL.


VS.
COURT OF APPEALS, ET AL.
G.R. No. 126859
Facts:

On March 31, 1995, Presiding Judge Mangay of the Regional Trial Court of Kalookan City issued search
warrants for the search and seizure of certain of certain items in Apartment No. 2 at 154 Obiniana
Compound, Deparo Road, Kalookan City and found the 2 M-16 rifles with 2 magazines and 20 live M16 ammunitions, among others. Likewise, the police searched also the Apartment No. 8, in the same
compound and found one (1) .45 caliber pistol. The firearms, ammunitions, explosives and other
incendiary devices seized at the apartments were acknowledged in the receipt signed by SPO2 Melanio de
la Cruz.
Petitioners were charged before the Regional Trial Court of Kalookan City, Branch 123, in informations
docketed as Criminal Cases Nos. C-48666-67, accusing them with illegal possession of firearms,
ammunitions and explosives, pursuant to Presidential Decree No. 1866. Thereafter, petitioners were
arrested and detained.
At the hearing for bail, the RTC denied petitioners' motion for bail earlier filed. Petitioners
contend that the search and seizure orders violated Sections 2 and 3 of the Bill of Rights as well as
Section 3 of Rule 126 of the Rules of Court on Criminal Procedure because the place searched
and articles seized were not described with particularity.

Issue:
Whether or not the respondent court erred and gravely abused its discretion when it ruled
that the search and seizure orders in question were valid and the objects seized admissiblein evidence.

Ruling:
WHEREFORE, the petition is PARTIALLY GRANTED. The search conducted at Apartment No. 8 is
hereby declared illegal and the item (.45 caliber pistol) seized therein is inadmissible in evidence.
However, the search at Apartment No. 2 pursuant to Search Warrant 55-95 is hereby declared valid and
legal, and the articles seized from Apartment No. 2 are found admissible in evidence. Let this can be
remanded to the Regional Trial Court of Kalookan City, Branch 123, for trial on the merits of Criminal
Cases Nos. C-48666-67 with dispatch.

PEOPLE OF THE PHILIPPINES


VS.

PRISCILLA DEL NORTE


G.R. No. 149462
Facts:
On August 1, 1997, a search warrant was served on Ising Gutierrez Diwa, residing at No.275 North
Service Road corner Cruzada St., Bagong Barrio, Caloocan City, by SPO1 Angel Lumabas and their group for alleged violation of
Republic Act No. 6425. They were ordered to forthwith seize and take possession of an undetermined quality of shabu and
marijuana leaves. They found a bundle of marijuana wrapped in manila paper under the bed and inside the room. Appellant was
brought to the police headquarters for further investigation.
Appellant assailed the validity of the search warrant against her. She asserted that she lived at 376 Dama de Noche, Brgy.
Balsa, Caloocan City and that she was just visiting a friend, Marlyn, who lived at 275 North Service Road corner Cruzada Street.,
Bagong Barrio, Caloocan City.
Issue:
Whether or not the accused is guilty beyond reasonable doubt.
Ruling:
No. The search warrant has irregularity because the authorities did not have personal knowledge of the
circumstances surrounding the search. They did not conduct surveillance before obtaining the warrant and their knowledge
was based on pure hearsay only. The prosecution witness failed to establish appellants ownership of the house where the
prohibited drugs were discovered.
IN VIEW WHEREOF, the decision of Branch 28 of the Regional Trial Court of Caloocan City is reversed. Appellant is
acquitted based on reasonable doubt.
SO ORDERED.

PEOPLE OF THE PHILIPPINES


VS.
ALVANO SAYCON Y BUQUIRAN
G.R. No. 110995
Facts:
The trial court rendered, on June 15, 1993, a judgment of conviction. The court found Alvano
Saycon guilty beyond reasonable doubt of having transported four (4) grams of Metamphetamine
hydrochloride (shabu) and sentenced him to life imprisonment and to pay a fine of P20,000.00

On July 8, 1992, 6:00 A.M. the Coastguard personnel received information from Narcotics
Command agent Ruben Laddaran that a suspected shabu courier by the name of Alvaro Saycon was on
board the MV Doa Virginia, which was arriving at that moment in Dumaguete City. Upon receipt of the
information, the Coastguard chief officer CPO Tolin, instructed them to intercept the suspect. NarCom
agents, Philippine Coastguard personnel and Senior Police Officers posted themselves at the gate of Pier 1.
The MV Doa Virginia docked at 6:00 a.m. that same morning at Pier 1 in Dumaguete City.
Alvaro Saycon alighted from the boat carrying a black bag and went through the checkpoint manned by
the Philippine Coastguard where he was identified by police officer. He was invited to the Coastguard
Headquarters at the Pier area and willingly went with them. The coastguard asked him to open his bag
and willingly obliged. In it were personal belongings and a maong wallet. Inside the maong wallet was a
Marlboro pack containing the suspected shabu. The police officer, Winifredo Noble asked
Saycon whether the Marlboro pack containing the suspected shabu was his, Saycon merely bowed his
head. Then Saycon, his bag and the suspected shabu were brought to the NARCOM office for booking.
When he was arrested, the NARCOM agents did not have a warrant of arrest.
Alvaro Saycon appeal before this court seeking reversal of the decision of the trial court and
contends that the search of his bag was illegal because it had been made without a search warrant and that
therefore, the shabu discovered during the illegal search was inadmissible in evidence against him.
Issue:
Is the warrantless search valid? Is the warrantless arrest valid?
Rulings:
Yes. Peace officers may lawfully conduct searches of moving vehicles- automobiles, trucks,
etc. without need of a warrant, it not being practicable to secure a judicial warrant before searching a
vehicle, since such vehicle can be quickly moved out of the locality or jurisdiction in which the warrant
may be sought. Also, a peace officer may without a warrant, arrest a person when he has probable cause to
believe based on personal knowledge of facts and circumstances.
WHEREFORE, the decision of the trial court in Criminal Case No. 10325, is hereby AFFIRMED,
with the MODIFICATIONS, however, appellant shall suffer imprisonment for an indeterminate period
ranging from six (6) months of arresto mayor as minimum to six (6) years of prision correctional as
maximum, and that the fine of P20,000.00 shall be DELETED. No pronouncement as to cost.

PHILIPPINE JUDGES ASSOCIATION ET AL


VS.
HON. PETE PRADO ET AL
G.R. No. 110995
Facts:
The petition assails the constitutionality of Sec. 35 of Republic Act No. 7354 as implemented by
the Philippine Postal Corporation through its Circular No. 92-98. These measures withdraw the franking
privilege from the Supreme Court, the Court of Appeals, the Regional Trial Courts, the Metropolitan Trial
Courts, the Municipal Trial Courts, and the Land Registration Commission and its Registers of Deeds,
along with certain other government offices.
The Philippine Judges Association averred that the law is discriminatory as it disallowed the
franking privilege of the Judiciary but has not disallowed the franking privilege of others such as the
President of the Philippines, the Vice President of the Philippines, Senators and Members of the House of
Representatives, the COMELEC and former Presidents of the Philippines among others.
Issue:
Whether or not there is a violation of equal protection before the law.
Ruling:
Yes. The equal protection clause does not require the universal application of the laws on all
persons or things without distinction. What the clause requires is equality among equals as determined
according to a valid classification. By classification is meant the grouping of persons or things similar to
each other in certain particulars and different from all others in these same particulars
ACCORDINGLY, the petition is partially GRANTED and Sections 35 of R.A. No. 7354 is
declared UNCONSTITUTIONAL. Circular No. 92-28 is SET ASIDE insofar as it withdraws the franking
privilege from the Supreme Court, the Court of Appeals, the Regional Trial Courts, the Metropolitan Trial
Courts, the Municipal Trial Courts, and the Land Registration Commission and its Registers of Deeds to
all of which offices the said privilege shall be RESTORED. The temporary restraining order dated June 2,
1992, is made permanent.

TELECOMMUNICATIONS AND BROADCAST ATTORNEYS OF THE PHILIPPINES, INC.


and GMA NETWORK, INC., petitioners,
vs.
THE COMMISSION ON ELECTIONS, respondent.
[G.R. No. 132922. April 21, 1998]
FACTS:
Petitioner Telecommunications and Broadcast Attorneys of the Philippines, Inc. (TELEBAP) is
an organization of lawyers of radio and television broadcasting companies. They are suing as citizens,
taxpayers and registered voters. It was declared to be without legal standing to sue in this case as, among
other reasons, it was not able to show that it was to suffer from actual or threatened injury as a result of
the subject law. Other petitioner, GMA Network, Inc., appears to have the requisite standing to bring this
constitutional challenge. Petitioner operates radio and television broadcast stations in the Philippines
affected by the enforcement of Sec. 92 of B.P Blg. 881 requiring radio and television broadcast companies
to provide free air time to the COMELEC for the use of candidates for campaign and other political
purposes. Petitioners challenge the validity of Sec. 92 on the ground (1) that it takes property without due
process of law and without just compensation; (2) that it denies radio and television broadcast companies
the equal protection of the laws; and (3) that it is in excess of the power given to the COMELEC to
supervise or regulate the operation of media of communication or information during the period of
election. Petitioner claims that it suffered losses running to several million pesos in providing COMELEC
Time in connection with the 1992 presidential election and 1995 senatorial election and that it stands to
suffer even more should it be required to do so again this year. Petitioners claim that the primary source
of revenue of the radio and television stations is the sale of air time to advertisers and to require these
stations to provide free air time is to authorize unjust taking of private property. According to petitioners,
in 1992 it lost P22,498,560.00 in providing free air time for one hour each day and, in this years
elections, it stands to lost P58,980,850.00 in view of COMELECs requirement that it provide at least 30
minutes of prime time daily for COMELEC Time.
ISSUES:
(1) Whether or not Section 92 of B.P. No. 881 denies radio and television broadcast companies the equal
protection of the laws.
(2) Whether or not Section 92 of B.P. No. 881 constitutes taking of property without due process of law
and without just compensation.

RULING:
Petitioners argument is without merit. All broadcasting, whether radio or by television stations,
is licensed by the government. Airwave frequencies have to be allocated as there are more individuals who
want to broadcast that there are frequencies to assign. Radio and television broadcasting companies,
which are given franchises, do not own the airwaves and frequencies through which they transmit
broadcast signals and images. They are merely given the temporary privilege to use them. Thus, such
exercise of the privilege may reasonably be burdened with the performance by the grantee of some form of
public service. In granting the privilege to operate broadcast stations and supervising radio and television
stations, the state spends considerable public funds in licensing and supervising them.
The argument that the subject law singles out radio and television stations to provide free air time
as against newspapers and magazines which require payment of just compensation for the print space
they may provide is likewise without merit. Regulation of the broadcast industry requires spending of
public funds which it does not do in the case of print media. To require the broadcast industry to provide
free air time for COMELEC is a fair exchange for what the industry gets.
As radio and television broadcast stations do not own the airwaves, no private property is taken
by the requirement that they provide air time to the COMELEC. The use of property bears a social
function and is subject to the states duty to intervene for the common good. Broadcast media can find
their just and highest reward in the fact that whatever altruistic service they may render in connection
with the holding of elections is for that common good.
For the foregoing reasons, the petition is dismissed.

MARIA CASTRO and CO LING petitioners,


vs.
HONORABLE JAVIER PABALAN, Judge of the Court of First Instance of La Union, and
SGT. ERNESTO LUMANG, respondents.
G.R. No. L-28642 April 30, 1976
FACTS:
Respondent Ernesto I. Lumang admitted that "he has been informed" and was of the belief that
Maria Castro and Co Ling, whose place of residence was not indicated, although subsequently mention
was made of their being at Barrio Padasil, Bangar, La Union, "have in possession narcotics and other
contraband." There is a claim that he had verified the report and that he had "reasons to believe that a
Search Warrant should be issued to enable the undersigned to take possession" of such narcotics and
other contraband. The application was accompanied by the joint affidavit of a Sergeant Francisco C.
Molina and a Corporal Lorenzo G. Apilado of the Philippine Constabulary. Again, mention was merely
made of their information about narcotics and other contraband being kept by Castro and Co Ling. They
allege that they conducted rigid surveillance, but all they could come out with is that petitioner Co Ling is

an overstaying alien for almost ten years conducting such traffic and that after verification, he was not
registered in the Immigration Office. Then, on the very same day, the search warrant was issued for illegal
traffic of narcotics and contraband. Again, there was reference to the possession by Castro and Co Ling of
such forbidden goods.
As to the complete and detailed description of the properties to be seized, the search warrant
merely mentioned illegal traffic of narcotics and contraband inside the warehouse and premises of
petitioners. In the resolution upholding the validity of the search warrant, respondent Judge did state the
following: "On July 10, 1967, Ernesto Lumang, Sgt. of the PC, with a long service behind, appeared in
chamber before the Presiding Judge of Branch I of this Court. With him were Sgt. Molina and Cpl. Apilado
both of the PC Command of La Union. The three submitted to the Presiding Judge in chamber an
application for search warrant which is Exhibit I in this case and a joint affidavit supporting the search
warrant asked. As Sgt. Lumang said, testifying regarding this incident, those appearing were asked,
although not in writing and not recorded, some questions by the Presiding Judge regarding their request
of the search warrant on the knowledge of Molina and Apilado on the facts stated on the application and
on the joint affidavit. The inquiry was brief. The barrio to be searched was handwritten in ink, Maria
Cristina cancelling the typewritten name Padasil. But this correction was not done in the duplicates.
Anyhow Padasil and Maria Cristina are adjoining barrios. After the routine taking of their oath and
examination questions and answers, the Presiding Judge of this Branch signed the application for search
warrant, the joint affidavits, and forthwith issued the search warrant.
ISSUE:
Whether or not a search warrant issued without complying with the requisites of the Constitution and the
Rules of Court should have been nullified
RULING:
Respondent Judge conducted the required "examination under oath" be justified merely because
respondent Lumang was "a Sergeant of the PC, with a long service behind him.
He did not even bother to take the depositions of the witnesses in writing, attaching them to the
record. There was thus a manifest and palpable violation of the constitutional standard as to the quantum
of proof to show the existence of probable cause. The Constitution requires, for the validity of a search
warrant, that there be a particular description of "the place to be searched and the persons or things to be
seized." As admitted by the judge in the challenged resolution, there was a mistake concerning the
residence of petitioners, which was set forth in the search warrant as being in Barrio Padasil when in fact
it is in Barrio Maria Cristina. Another infirmity was the failure to comply with the basic procedural
requisite that a search warrant "shall not issue but upon probable cause in connection with one specific
offense."
Reference was made to "an illegal traffic of narcotics and contraband." The latter is a generic term
covering all goods exported from or imported into the country contrary to applicable statutes. More than

one offense could arise from the activity designated as illegal traffic of narcotics and contraband. As a
matter of fact, in the challenged order, reference was made to at least three charges having been filed, the
violation of Section 203 of the Internal Revenue Code, its Section 1039 on tax evasion, as well as illegal
possession of opium. It is the established doctrine in this jurisdiction that the illegality of the search
warrant does not call for the return of the things seized, the possession of which is prohibited by law. The
issuance of the search warrant in question the judge did not comply with the requirements of section 98 of
General Orders No. 58, the petitioners are not entitled to the return of the opium and its paraphernalia
which were found and seized under said warrant, and much less are they entitled to be exonerated
because of such omission of the judge. The SC held that the search warrant in question is tainted by
illegality for being violative both of the Constitution and the Rules of Court. Likewise notwithstanding the
illegality of such search warrant, the challenged order of respondent Judge can be sustained only insofar
as it would limit the return of the articles seized to the liquor, the pack of playing cards, the bottle of
distilled water and five bottles of Streptomycin taken under such search warrant.

BERNARD R. NALA, petitioner,


vs.
JUDGE JESUS M. BARROSO, JR., Presiding Judge, Regional Trial Court, Branch 10, 10th
Judicial Region, Malaybalay City, respondent
G.R No. 153087, August 7, 2003
FACTS:
On June 25, 2001, PO3 Macrino L. Alcoser together with his witness Ruel Nalagon applied for
the issuance of a warrant to search the person and residence of petitioner Bernard R. Nala, who was
referred to in the application as Rumolo Nala alias Long of Purok 4, Poblacion, Kitaotao, Bukidnon.
The application was filed in connection with petitioners alleged illegal possession of one caliber .22
magnum and one 9 mm. pistol in violation of Illegal Possession of Firearms. On the same day,
respondent Presiding Judge of RTC of Malaybalay City, issued Search and Seizure Warrant .On July 4,
2001, at around 6:30 in the morning, Alcoser and other police officer search the petitioners house and
allegedly seized the following: (1) one piece caliber .38 revolver (snub-nose) with Serial Number 1125609;
(2) one pc. Fragmentation grenade (cacao type); (3) one pc. .22 long barrel; (4) 5- pcs live ammunition for
caliber .38 revolver; and (5) 4- four pcs. of disposable lighter and unestimated numbers of cellophane
used for packing of shabu. Petitioner questioned the validity of the search warrant and filed an Omnibus
Motion to Quash but was denied by the judge.
Lower court found that probable cause was duly established from the deposition and examination
of witness Ruel Nalagon and the testimony of PO3 Alcoser who personally conducted a surveillance to
confirm the information given by Nalagon. The fact that the items seized were not exactly the items listed
in the warrant does not invalidate the same because the items seized bear a direct relation to the crime of

illegal possession of firearms. Respondent judge also found that petitioner was sufficiently identified in
the warrant although his first name was erroneously stated therein as Romulo and not Bernard,
considering that the warrant was couched in terms that would make it enforceable against the person and
residence of petitioner and no other.
ISSUES:
(1) Was petitioner sufficiently described in the search and seizure warrant?
(2) Was there probable cause for the issuance of a search and seizure warrant against petitioner?
(3) Whether or not the firearms and explosive allegedly found in petitioners residence are admissible in
evidence against him even though said firearms were not listed in the search and seizure warrant.
Immaterial due to a void search warrant.
RULING:
(1) YES. the failure to correctly state in the search and seizure warrant the first name of petitioner, which is
Bernard and not Romulo or Rumolo, does not invalidate the warrant because the additional
description alias Lolong Nala who is said to be residing at Purok 4, Poblacion, Kitaotao, Bukidnon
sufficiently enabled the police officers to locate and identify the petitioner. . What is prohibited is a
warrant against an unnamed party, and not one which, as in the instant case, contains a descriptio
personae that will enable the officer to identify the accused without difficulty.
(2) NO. Nowhere in the affidavit and testimony of witness Ruel Nalagon nor in PO3 Macrino L. Alcosers
application for the issuance of a search warrant was it mentioned that petitioner had no license to possess
a firearm. PO3 Alcoser and his witness Ruel Nalagon did not have personal knowledge but only
personal belief of petitioners lack of license to possess firearms, ammunitions and explosives; and did
not adduce the evidence required to prove the existence of probable cause. Hence, the search and seizure
warrant issued on the basis of the evidence presented is void.
(3) The settled rule is that where entry into the premises to be searched was gained by virtue of a void search
warrant, prohibited articles seized in the course of the search are inadmissible against the accused.
Prohibited articles may be seized but only as long as the search is valid. In this case, it was not because: 1)
there was no valid search warrant; and 2) absent such a warrant, the right thereto was not validly waived
by the petitioner. In short, the military officers who entered the petitioners premises had no right to be
there and therefore had no right either to seize the pistol and bullets.
WHEREFORE, in view of all the foregoing, the petition is GRANTED. Search and Seizure Warrant is
declared VOID and the articles seized by virtue thereof are declared inadmissible in evidence.

PEOPLE OF THE PHILIPPINES, appellee,


vs.
ANTONIO C. ESTELLA, appellant.
[G.R. Nos. 138539-40. January 21, 2003]
FACTS:
Prior to Nov. 20, 1996, Executive Judge Romulo Estrada of the RTC of Zambales issued a warrant
for the conduct of a search and seizure in the residence of appellant at Purok Yakal, Barangay Baloganon,
Masinloc, Zambales. On same day, Senior Police Officer (SPO1) Antonio Buloron, then Intelligence and
Investigation Officer, together with SPO1 Jose Arca and several other members of the Provincial Special
Operation Group based in Burgos, San Marcelino, Zambales, coordinated with the members of the
Philippine National Police (PNP) in Masinloc and sought the assistance of Barangay Captain Rey
Barnachea of Baloganon, Masinloc for the enforcement of the search warrant. On their way to Purok
Yakal, SPO1 Buloron saw Estrella sitting on a rocking chair located about 2 meters away from a hut owned
by Narding Estella, brother of appellant, and being rented by Estrella's live-in partner, named Eva. They
approached Estrella and showed him the search warrant. Estrella surrendered to the team 2 cans
containing dried marijuana fruiting tops. One can contained 20 bricks of fruiting tops. The team searched
the hut in the presence of Estrella and his live-in partner. They found a plastic container which contained
4 big bricks of dried marijuana leaves and a .38 caliber revolver with four live ammunitions. The team
seized the prohibited drug, the revolver and ammunitions. SPO1 Buloron and his companions arrested
Estrella and brought him to San Marcelino, Zambales.
The defense, however has different version, denied having surrendered to policeman Buloron tin cans
containing marijuana and likewise having any firearm. Appellant also claims that the hut, which was
searched by the police and where the subject marijuana was recovered, does not belong to him. He points
to another house as his real residence. Estella was investigated at San Marcelino, Zambales where he
informed the police officers of the fact that the house they searched was occupied by Spouses Vicente and
Fely Bakdangan. Still, Estrella was charged for possession of prohibited drugs and unlicensed firearms.
On the other hand, Estrella was acquitted from the charge of violation of PD 1866 The .38 caliber revolver
without serial number and 4 live ammunitions, subject of the offense, were however ordered delivered to
any authorized representative of the Philippine National Police, Firearms and Explosives Division, Camp
Crame, Quezon City. Estrella appealed said decision.

ISSUE:
Whether the search undertaken inside the hut during which the incriminating evidence was allegedly
recovered was legal.
RULING:
There is no convincing proof that Estrella indeed surrendered the prohibited drug, whether
voluntarily or otherwise. In fact, the testimony of Prosecution Witness Barnachea clouds rather than
clarifies the prosecution's story. Given this backdrop, the police authorities cannot claim that the search
was incident to a lawful arrest. Such a search presupposes a lawful or valid arrest and can only be invoked
through Section 5 (Arrest without warrant; when lawful), Rule 113 of the Revised Rules on Criminal
Procedure, which provides that "A peace officer or a private person may, without a warrant, arrest a
person: (a) when, in his presence, the person to be arrested has committed, is actually committing, or is
attempting to commit an offense; (b) When an offense has just been committed and he has probable cause
to believe based on personal knowledge of facts or circumstances that the person to be arrested has
committed it; and (c) When the person to be arrested is a prisoner who has escaped from a penal
establishment or place where he is serving final judgment or is temporarily confined while his case is
pending, or has escaped while being transferred from one confinement to another.
All told, without sufficient admissible evidence against appellant, the prosecution failed to establish his
guilt with moral certainty. Not only did its evidence fall short of the quantum of proof required for a
conviction, it has also failed to present any evidence at all. Under our Bill of Rights, among the
fundamental rights of the accused is to be presumed innocent until the contrary is proved. To overcome
such presumption, the prosecution must establish guilt beyond reasonable doubt. Our criminal justice
system dictates that if the prosecution fails to do so, it becomes not only the right of the accused to be set
free, but also the constitutional duty of the court to set them free. This principle leaves this Court no
option but to acquit Appellant Antonio C. Estella for insufficiency of evidence.
WHEREFORE, the appealed Decision is SET ASIDE. Antonio C. Estella is ACQUITTED and ordered
immediately RELEASED from custody.

PEOPLE OF THE PHILIPPINES, plaintiff-appellee,


vs. MELLY SARAP y ARCANGELES and ROGER AMAR y MATEO, accused-appellant.
[G.R. No. 132165. March 26, 2003]
FACTS:
Armed with a search warrant, SPO4 Gelacio R. Guarino, Chief of Police of Banga, Aklan together
with PO2 Jhanny Navida, raided the house of Conrado Ricaforte at Rizal St., Poblacion, Banga, Aklan on
March 2, 1996, relative to the reported sale of marijuana by its occupants, Jonalyn Duran, Joysie Duran
and Pepe Casabuena. The three were apprehended for illegal possession of marijuana and were detained

at the Banga Police Station. In the course of their investigation, the police learned that a certain Melly
from Capiz and Roger Amar were the suppliers of marijuana and that they will be back on March 4, 1996.
On March 4, 1996, Janet Iguiz, caretaker of the house of Conrado Ricaforte informed Guarino
that there were two strangers looking for the Duran sisters. Then they proceeded to the house and saw a
woman, who turned out to be accused-appellant Melly Sarap. Melly saw Guarino and Navida in police
uniform and immediately threw away her black canvass bag, which Roger Amar picked up. Guarino
Blocked Saraps path and grabbed from her the green plastic bag she was holding. The plastic bag was
found to contain two blocks of marijuana fruiting tops. Navida pursued Amar and arrested him. The
accused-apellant denied the accusations against her. The Court fines accused guilty beyond reasonable
doubt of violation of Article II, Sec 4 of Republic Act 6425, otherwise known as the Dangerous Drugs Act.
The accused appealed the decision of the trial court.
ISSUES:
(1) Whether the warrantless search and arrest conducted is legal.
(2) Whether the evidence presented by the prosecution is sufficient to find the accused guilty beyond
reasonable doubt.
RULING:
A search may be conducted by law enforcers only on the strength of a warrant validly issued by a
judge as provided in Article III, Section 2 of the Constitution. Articles which are the product of
unreasonable searches and seizures are inadmissible as evidence, pursuant to Article III, Section 3 (2) of
the Constitution. Warrantless searches and seizures may be made without a warrant in the following
instances: (1) search incident to a lawful arrest, (2) search of a moving motor vehicle, (3) search in
violation of custom laws, (4) seizure of the evidence in plain view, (5) when the accused himself waives
his right against unreasonable searches and seizures, (6) stop and frisk and (7) exigent and emergency
circumstances. These instances, however do not dispense with the requisite of probable cause before a
warrantless search and seizure can be lawfully conducted. In warrantless search cases, probable cause
must only be based on reasonable ground of suspicion or belief that a crime has been committed or is
about to be committed.
Without the illegally seized prohibited drug, the appellants conviction cannot stand. There is
simply no sufficient evidence to convict her. That the search disclosed marijuana fruiting tops in
appellants possession, and thus confirmed the police officers initial information and suspicion, did not
cure its patent illegality. An illegal search cannot be undertaken and then an arrest effected on the
strength of the evidence yielded by the search for being a fruit of a poisonous tree.
All told, the guilt of the accused-appellant was not proven beyond reasonable doubt measured by the
required moral certainty of conviction. The evidence presented by the prosecution was not enough to
overcome the presumption of innocence as constitutionally ordained
Wherefore the Accused-appellant Melly Sarap y Arcangeles is ACQUITTED of the crime charged on the
ground of reasonable doubt.

Ramon B. Ceniza, Federico C. Cabilao Jr., Nelson J. Rosal and Alejandro R. Alinsug,
petitioners
vs.
Commission on Elections, Commission on Audit and National Treasurer, respondents
G.R. No. L-52304
January 28, 1980
Facts:
On December 22, 1978, the Interim Batasang Pambansa enacted Batas Blg. 51, providing rules
regarding the upcoming local election on January 30, 1980. To implement this act, the Commission on
Elections(COMELEC) adopted Resolution No. 1421, which barred qualified voters from highly urbanized
and chartered component cities(unless if its charter provides the contrary) to vote for positions in the
provincial level. The resolution listed the City of Mandaue and Cebu among the cities not entitled to
participate in the pro-provincial elections. Cebu was included because it was classified as a highly
urbanized city(having income not less than 40,000,000 pesos) while Mandaue though only a component
city, however its charter expressly provides that its registered voters cannot vote, except to be a candidate
thereof. Ramon Ceniza, in behalf of a group called DOERS (Democracy or Extinction: Resolved to
Succeed), filed a petition assailing that Batas Pambansa Blg. 51 and Republic Act No. 5519(Charter of
Mandaue City) are unconstitutional. The petitioners contend that, Section 96, Art. XVIII of the Charter of
Mandaue is unconstitutional for it went into effect without the benefit of ratification by the residents in a
plebiscite or referendum. They also claimed that political and gerrymandering motives were behind the
passage of BP Blg. 51 pointing out the province of Cebu is politically and historically known as an
opposition bailiwick. Most importantly, Sec. 3 of Batas Blg. 51, that classifies Cebu City as a highly
urbanized city as the only basis for not allowing its electorate for provincial officials is inherently and
palpably unconstitutional and such classification is not based upon substantial distinctions making it
unreasonable which amounts to denial of equal protection.
Issue:
Are the voters of Mandaue and Cebu City denied with the equal protection of the law?

Ruling:
The Court found no merit in the petition. The classification of cities into highly urbanized cities
and components cities on the basis of their regular income is based upon substantial distinction. The
revenue of a city would show whether or not is capable of existence and development as a relatively
independent social, economic and political unit. Cities with smaller income need the continued support of
the provincial government thus justifying the continued participation of the voters in the election of
provincial officials in some instances.
The equal protection of the law contemplates equality in the enjoyment of similar right and privileges
granted by law. The law would have been discriminatory and a denial of equal protection of the law if the
statute prohibited an individual or group of voters in the city from voting for provincial officials while
granting it to another individual or groups of voters in the same city. Moreover, the practice of allowing
voters in one component city to vote for provincial officials and denying the same privilege to voters in
another city is a matter of legislative discretion which neither violates the Constitution nor the voters
right of suffrage. It cannot be considered also an infringement upon such right since the Constitution
confers no right to a voter in a city to vote for provincial officials of the province where the city is located.
Their right is limited to the right to vote for elective city officials in local elections in which the questioned
statute neither withdraw nor restrict.
On the constitutionality of the Charter of the City of Mandaue; the constitutional requirement that the
creation, division, merger, abolition or alteration of the boundary of a province, city, municipality, or in a
barrio should be subjected to the approval by the majority of votes cast in a plebiscite in the government
unit or units affected is a new requirement that came into being only with the 1973 Constitution, which is
prospective therefore cannot affect the creation of Mandaue City on June 21, 1969.
Finally, on the claims of political and gerrymandering motives is of no factual or legal basis.
Gerrymandering is a term employed to describe an appointment of representative districts as to give an
unfair advantage to the party in power. The questioned statutes do not apportion representative districts
nor has it been shown that there has been an unfair advantage in favor of the candidates in power.
WHEREFORE, the petition is dismissed. Costs against the petitioners

Gil V. Manlavi, complainant,


vs.
Judge Eustaquio Z. Gacott, Jr., Regional Trial Court, Branch 47, City of Puerto Princesa,
respondent
A.M. No. RTJ-95-1293
May 9, 1995

Facts:
On January 18, 1991 at Brgy. Mandaragat, Pureto Princesa City, the accused dis then willfully,
unlawfully and feloniously possess illegally caught assorted fish with the use of explosives weighing more
or less eight thousand (8,000) kilos. The complainant filed thereafter Criminal Cases No. 9210(Illegal
Possession of Explosives Intended for Fishing) and 9211 (Illegal Possession of Illegally Caught Fish)
against the accused. The cases were consolidated for trial at the sala of the Honorable respondent. The
accused then moved to quash Criminal Case No. 9210 on the ground that the evidence of the prosecution
was a product of a warrantless and illegal search and seizure and also moved to quash Criminal Case
Criminal case 9211 on the ground that the information failed to charge the offense of illegal possession of
fish caught by explosives for its failure to allege the element of profit.
The respondent granted both the motion to quash the criminal cases filed. The prosecution moved for the
reconsideration of the order but was denied. The complainant then charged the respondent with
partiality, miscarriage of justice and knowingly rendering an unjust decision in connection with the
dismissal of the Criminal Cases Nos. 9210 and 9211.
Issue:
Whether or not the judge erred in dismissing the case due to warrantless arrest and search and seizure.
Ruling:
The complaint is dismissed. As to the dismissal of Criminal Case No. 9210, complainant himself
admitted that the search and seizure was conducted in the absence of a warrant and that the search
warrant was only produced by the complainant after the search and seizure took place. The complainant
invoked Circular No. 130(s.1967) of the Office of the President to justify the warrantless search. The said
circular pertains to the procedure in the confiscation of fish caught by the use of explosives. Such
confiscation may be exercised only by the Commissioner of Fisheries or his representatives who can only
take a sample of the fish caught (not to exceed one kilo) for testing if the fish were indeed caught through
the use of explosives. It is only upon the determination that the fish were caught through the use of
explosives when the seizure of the entire catch may be authorized. Thereafter, an appraisal of the value of
the fish caught shall be made, which shall be paid to the accused should he be subsequently acquitted in
the criminal case filed against him. The arresting officer failed to show compliance with the procedure
prescribed by the very circular they invoked.
As to the dismissal of the Criminal Case 9211, though the respondent erred in holding that the
information was defective that the information was defective in not alleging that the offense was
committed knowingly because the element of knowledge was encompassed within the word willfully;
however, the information failed to allege the element of profit. Though it is true that the prohibits
Presidential Decree No. 704 prohibits the separate acts of possessing, dealing in, selling or disposing of
illegally caught fish and aquatic products, but said acts must not only be done knowingly but also for
profit, as essential element of the offense.

People of the Philippines, plaintiff- appellee,


vs.
Rolando Codilla, German Lucanas and Marcelo Putulin, accused-appellants
Facts:
On May 24, 1990 at about 3:00 oclock in the early morning at Brgy. Concepcion, Ormoc City,
Helen and her sister Leticia Pepito were awakened from their sleep. When they opened their eyes they
were surprised to see two men carrying bolos and flashlights. One of the men asked if they had money;
when Helen replied that they have none, she was ordered to go down the kitchen while her sister was
brought to the sala by and there Helen was allegedly raped by Rolando Codilla and while her siste was
raped by Marcelo Putulin.
Also on November 27, 1990 at 3:00 o clock in the morning, Margarita Alpos was sexually abused by
two men who she identified as Rolando Codilla and German Lucanas. At around 2:30 of the same date,
Sgt. Romeo Penarada together PFC Mamento Sarcol Jr, PFC Diosdado Tagalog, Pat. Eduardo Bituin and
CVO Manuel Pepito proceeded to the place where the alleged rape suspects were hiding and thus the
police were able to apprehend the suspects and brought them to the Ormoc City Police Station. The RTC
convicted the appellants for the crime of rape and as well as to indemnify the victims for damages.
The accused-appellants then filed a petition to review and reverse the decision. However, during the
pendency of the appeal, Roland Codilla escaped from jail on July 27, 1991 while German Lucanas
whereabouts remains unknown after a flashflood hit their cells. As such only the appeal of Putulin was the
only petition the court has resolved.
Issue:
Whether or not the nature and circumstances surrounding their arrest is violative of their constitutional
right against illegal warrantless arrest.
Ruling:
The appellant started his defense by challenging his warrantless arrest and detention for two
days without any charges being filed against him. However, this argument must be rejected by the court
for the simple reason that he is estopped from questioning the legality of his arrest. Any objection
involving a warrant of arrest or the procedure in the acquisition by the court of jurisdiction over the
person of an accused must be made before he enters his plea, otherwise the objection is deemed waived.

Besides, this issue is being raised for the first time by the appellant before this court. He did not move for
the quashal of the information before the trial court. Hence, any irregularity for his arrest, if any was
cured when he voluntarily submitted himself to the jurisdiction of the trial court by entering a plea of not
guilty and participating in the trial.
Wherefore, the assailed judgment of the lower court is affirmed, with costs against the petitioners.
The People of the Philippines, plaintiff-appellee
vs.
Fidel Abrenica Cubcubin Jr., accused-appellant
Facts:
At about 3:30 in the morning of August 26, 1997, the Cavite City Police Station has receiveda
telephone call that a person had been shot near the cemetery along Julian Felipe Boulevard in San
Antonio, Cavite City. For this reason, a police team composed of SPO1 Malinao Jr. , PO3 Rosal, PO3 Estoy
Jr., PO3 Manicio and SPO3 Manalo responded to the call and found Henry P. Piamonte slumped dead on
his tricycle. A tricycle driver, who refused to divulge his name, then told PO3 Rosal that the accusedappellant and the victim were last seen together coming out of the Sting Caf, located about a kilometer
and a half away from the crime scene. Danet Garcellana, a food server/ waitress at the said caf told the
police investigators that she had seen the accused arrive together with the victim however she did not
know if they left together. Garcellano described the accused-appellant in which then another tricycle
driver told the investigators that he knows a person that fits the description given by the waitress and told
them where his house is.
As they went to the house and upon entering, SPO1 Malinao noticed a bloodied white Hanes tshirt. As he picked up the shirt, two(2) spent .38 caliber shells fell. As they proceeded the search, PO3
Estoy found on top of a plastic water container outside the bathroom a homemade Smith and Wesson
caliber .38 revolver and five live ammunitions. The police station then took custody of Cubcubin and the
evidences found. After an evaluation of the evidence, a formal criminal complaint was then filed against
the accused-appellant.
The trial court convicted the accused-appellant guilty as charged for murder.
Issues:
Whether or not the arrest of the accused- appellee was valid.
Whether or not the search and seizure of the alleged incriminating evidences was valid and legal.
Ruling:
Section 5 of Rule 113 of the 1985 Rules on Criminal Procedure as amended points out the three
instances when warrantless arrest is lawful. The case at bar, falls under par. B Sec. 5 which provides that

for a warrantless arrest to be valid, two conditions must concur: first, the offender has just committed an
offense and second, the arresting peace officer has personal knowledge of the facts indicating that the
person to be arrested has committed it. It has been held that personal knowledge of facts in an arrest
without warrant must be based upon probable cause, which means an actual belief or reasonable ground
for suspicion. In this case, the arrest of the accused-appellant was shortly after the victim was killed. The
question therefore, is whether there was probable cause for the arresting officer to believe that the
accused committed the crime. The court found none. The arresting officers did not have personal
knowledge of the facts as their knowledge of the circumstance from which they allegedly inferred that the
accused was probably guilty was based entirely on what they had been told by others. They merely relied
on information given to them by others.
As to the validity of the search and seizure, even assuming that the warrantless arrest to be valid
as the prosecution contends, the search cannot be considered incidental thereto. A valid warrantless
search incidental to a valid arrest allows only the seizure of evidence or dangerous weapons either in the
person of the one arrested or within the area of his immediate control. It is clear that the warrantless
arrest cannot be justified on this ground for neither the t-shirt nor the gun were within the area of
accused-appellants control. Nor the warrantless search is justified under the plain-view doctrine. The
alleged evidences against the accused did not merely stumble upon the police officers as such they are
purposely sought it. Hence the things obtained as a result of the search are illegal and are inadmissible
evidence against the Cubcubin.
Wherefore, the decision of the RTC charging the accused guilty of murder is reversed and he is
acquitted on the ground of reasonable doubt.

ople of the Philippines, plaintiff-appellee


vs.
Antolin Cuizon y Ortega, Steve Pua y Clofas alias Stephen Po y Uy or Tommy Sy and
Paul Lee y Wong alias Paul Leung, accused-appellants
Facts:
On February 21, 1992 accused Cuizon and his wife from Hongkong at the NAIA. After which they
proceeded to the arrival area of the airport where they handed down four (4) travelling bags to the
accused Steve Pua and Paul Lee. On the other hand, a group of NBI agents have posted themselves before
the arrival of the couple at the airport for the purpose of apprehending them because they allegedly
brought with them a huge quantity of shabu as tipped by an informant from Hongkong. The accused Pua
and Lee loaded the bags in a taxicab which they boarded in leaving the airport while the accused Cuizon

and his wife took another vehicle. At this juncture, the other team positioned at the parking area was
radioed to intercept the vehicle accused Pua and Lee boarded, however the communication was not
completely received as they radio ran out of battery power.
Realizing the failure, the group followed the vehicle taken by Pua and Lee which proceeded to
Manila Peninsula Hotel in Makati. With the help of the Chief Security Officer of the hotel, they
apprehended Lee and Pua in their hotel room and confiscated four (4) travelling bags; three (3) of which
yielded a plastic containing a white substance suspected to be shabu. Thereafter, the team proceeded to
the house of accused Cuizon, taking with them Pua and Lee. Retrieved from the accused Cuizons
residence was another bag containing a substance likewise suspected to be shabu; in addition a .38 caliber
firearm.
All of them were then taken into custody by the NBI and was the charged (with the exception of
Cuizons wife) in violation of Section 15 of R.A. 6425 or the Dangerous Drugs Act of 1972. The RTC found
the accused-appellants guilty as charged.
Issue:
Whether or not the warrantless arrests and warrantless search and seizures conducted by the
NBI against the accused are legal and constitutional.
Ruling:
The court cannot agree at the conclusion of the trial court that the appellants were caught in
flagrante delicto which would have justified the warrantless search. Paragraph (a) of Sec. 5, Rule 113 of
the Rules of Court on lawful arrest without warrant requires that the person be arrested after he has
committed or while he is actually committing or is at least attempting to commit an offense in the
presence of the arresting officer. These requirements are not present, for at the time of the arrest,
appellants Pua and Lee were merely resting in their hotel room and appellant Cuizon was in his bed
resting with his wife and children. No offense had just been committed, or was being committed by the
accused n the presence of the lawmen.
Paragraph (b) of the same rule is also inapplicable as such its requirements have not been met. The
prosecution failed to establish that at the time of the arrest an offense has in fact just been committed and
the arresting officer has personal knowledge of the facts indicating that the accused-appellants have
committed it. Appellant Cuizon could not, by the mere act of handing over four(4) pieces of luggage to the
other two appellants be considered to have committed the offense of carrying and transporting
prohibited drugs. The only reason why such it became a felonious deed was because of an alleged tip
received by the NBI that morning. The NBI merely relied on hearsay information and such under the
circumstance it failed to establish that they have personal knowledge sufficient and reasonable enough
to believe that the appellants had committed a crime at the point when the search and seizure were made.
Therefore, accused-appellant Cuizon is acquitted on constitutional ground.

G.R. NO. 128845,


JUNE 1, 2000
INTERNATIONAL SCHOOL ALLIANCE OF EDUCATORS (ISAE), petitioner,
vs.
HON. LEONARDO A. QUISUMBING in his capacity as the Secretary of Labor and
Employment; HON. CRESENCIANO B. TRAJANO in his capacity as the Acting Secretary of
Labor and Employment; DR. BRIAN MACCAULEY in his capacity as the Superintendent of
International School-Manila; and INTERNATIONAL SCHOOL, INC., respondents.
FACTS:
Private respondent International School, Inc. (School), pursuant to PD 732, is a domestic
educational institution established primarily for dependents of foreign diplomatic personnel and other
temporary residents. The decree authorizes the School to employ its own teaching and management
personnel selected by it either locally or abroad, from Philippine or other nationalities, such personnel
being exempt from otherwise applicable laws and regulations attending their employment, except laws
that have been or will be enacted for the protection of employees. School hires both foreign and local
teachers as members of its faculty, classifying the same into two: (1) foreign-hires and (2) local-hires.
Classification varies from four (4) queries, viz;
a. What is one's domicile?
b. Where is one's home economy?
c. To which country does one owe economic allegiance?
d. Was the individual hired abroad specifically to work in the School and was the School responsible for
bringing that individual to the Philippines?
The School grants foreign-hires certain benefits not accorded local-hires. Foreign-hires are also
paid a salary rate 25% more than local-hires.
When negotiations for a new CBA were held on June 1995, petitioner ISAE, a legitimate labor
union and the collective bargaining representative of all faculty members of the School, contested the
difference in salary rates between foreign and local-hires. This issue, as well as the question of whether
foreign-hires should be included in the appropriate bargaining unit, eventually caused a deadlock between
the parties.

ISAE filed a notice of strike. Due to the failure to reach a compromise in the NCMB, the matter
reached the DOLE which favored the School.
ISSUE:
Should the foreign-hires should be included in bargaining unit of local- hires?

RULING:
NO. The Constitution, Article XIII, Section 3, specifically provides that labor is entitled to
humane conditions of work. These conditions are not restricted to the physical workplace the factory,
the office or the field but include as well the manner by which employers treat their employees.
Discrimination, particularly in terms of wages, is frowned upon by the Labor Code. Article 248
declares it an unfair labor practice for an employer to discriminate in regard to wages in order to
encourage or discourage membership in any labor organization.
The Constitution enjoins the State to protect the rights of workers and promote their welfare, In
Section 18, Article II of the constitution mandates to afford labor full protection. The State has the right
and duty to regulate the relations between labor and capital. These relations are not merely contractual
but are so impressed with public interest that labor contracts, collective bargaining agreements included,
must yield to the common good.
However, foreign-hires do not belong to the same bargaining unit as the local-hires.
A bargaining unit is a group of employees of a given employer, comprised of all or less than all of
the entire body of employees, consistent with equity to the employer indicate to be the best suited to serve
the reciprocal rights and duties of the parties under the collective bargaining provisions of the law.
The factors in determining the appropriate collective bargaining unit are (1) the will of the
employees (Globe Doctrine); (2) affinity and unity of the employees interest, such as substantial
similarity of work and duties, or similarity of compensation and working conditions (Substantial Mutual
Interests Rule); (3) prior collective bargaining history; and (4) similarity of employment status. The basic
test of an asserted bargaining units acceptability is whether or not it is fundamentally the combination
which will best assure to all employees the exercise of their collective bargaining rights.
In the case at bar, it does not appear that foreign-hires have indicated their intention to be
grouped together with local-hires for purposes of collective bargaining. The collective bargaining history
in the School also shows that these groups were always treated separately. Foreign-hires have limited
tenure; local-hires enjoy security of tenure. Although foreign-hires perform similar functions under the
same working conditions as the local-hires, foreign-hires are accorded certain benefits not granted to
local-hires such as housing, transportation, shipping costs, taxes and home leave travel allowances. These
benefits are reasonably related to their status as foreign-hires, and justify the exclusion of the former from
the latter. To include foreign-hires in a bargaining unit with local-hires would not assure either group the
exercise of their respective collective bargaining rights.

WHEREFORE, the petition is GIVEN DUE COURSE. The petition is hereby GRANTED IN PART.
The Orders of the Secretary of Labor and Employment dated June 10, 1996 and March 19, 1997, are
hereby REVERSED and SET ASIDE insofar as they uphold the practice of respondent School of according
foreign-hires higher salaries than local-hires.

G.R. No. 133254-55,


April 19, 2001
THE PEOPLE OF THE PHILIPPINES, plaintiff-appellee
vs.
ROBERTO SALANGUIT y KO, accused-appellant
FACTS:
On December 26, 1995, around 10:30 p.m., a group of about ten (10) policemen, along with one
civilian informer, went to the residence of the accused-appellant, Roberto Salanguit y Ko, a search
warrant was shown, and the police operatives started searching the house. They found heat-sealed
transparent plastic bags containing a white crystalline substance, a paper clip box also containing a white
crystalline substance, and two bricks of dried leaves which appeared to be marijuana. A receipt of the
items seized was prepared, but the accused-appellant refused to sign it. Charges against Roberto Salanguit
y Ko for violations of Republic Act (RA) 6425, i.e. for possession of shabu and marijuana, (Criminal Cases
Q-95-64357 and Q-95-64358, respectively) were filed, and after hearing, the trial court convicted him in
Criminal Cases Q-95-64357 and Q-95-64358 for violation of Section 16 and 8, respectively.
The accused-appellant contended that the evidence against him was inadmissible because the
warrant used in obtaining it was invalid.
ISSUES:
1.

Was the warrant find invalid for failure of providing evidence to support the seizure of drug
paraphernalia?

2.

Whether the marijuana may be included as evidence in light of the plain view doctrine.
RULING:
Yes. The warrant authorized the seizure of undetermined quantity of shabu and drug
paraphernalia. Evidence was presented showing probable cause of the existence of methamphetamine
hydrochloride or shabu. The fact that there was no probable cause to support the application for the

seizure of drug paraphernalia does not warrant the conclusion that the search warrant is void. This fact
would be material only if drug paraphernalia was in fact seized by the police. The fact is that none was
taken by virtue of the search warrant issued. If at all, therefore, the search warrant is void only insofar as
it authorized the seizure of drug paraphernalia, but it is valid as to the seizure of methamphetamine
hydrochloride as to which evidence was presented showing probable cause as to its existence. In sum,
with respect to the seizure of shabu from Salanguits residence, Search Warrant 160 was properly issued,
such warrant being founded on probable cause personally determined by the judge under oath or
affirmation of the deposing witness and particularly describing the place to be searched and the things to
be seized. With respect to, and in light of the plain view doctrine, the police failed to allege the time
when the marijuana was found, i.e., whether prior to, or contemporaneous with, the shabu subject of the
warrant, or whether it was recovered on Salanguits person or in an area within his immediate control. Its
recovery, therefore, presumably during the search conducted after the shabu had been recovered from the
cabinet, as attested to by SPO1 Badua in his deposition, was invalid. Thus, the Court affirmed the decision
as to Criminal Case Q-95-64357, accused-appellant Roberto Salanguit y Ko guilty of possession of illegal
drugs S16 of R.A.No. 6425.

R No. 94054-67
February 19, 1991
VICENTE LIM SR. and MAYOR SUSANA LIM, petitioner
Vs.
HON. NEMESIO S. FELIX and HON. ANTONIO ALFANE, respondents

FACTS:
On March 17, 1989, at about 7:30 o'clock in the morning, at the vicinity of the airport road of the
Masbate Domestic Airport, located at the municipality of Masbate province of Masbate ,Congressman
Moises Espinosa, Sr. and his security escorts, namely Provincial Guards Antonio Cortes, Gaspar Amaro,
and Artemio Fuentes were attacked and killed by a lone assassin. Dante Siblante another security escort of
Congressman Espinosa, Sr. survived the assassination plot, although, he himself suffered a gunshot
wound.
An investigation of the incident then followed.
Thereafter, and for the purpose of preliminary investigation, the designated investigator, Harry O.
Tantiado, TSg, of the PC Criminal Investigation Service at Camp Bagong Ibalon Legazpi City filed an

amended complaint with the Municipal Trial Court of Masbate accusing, among others, Vicente Lim,
Sr.,Mayor Susana Lim of Masbate, Jolly T. Fernandez, Florencio T. Fernandez, Jr., Nonilon A. Bagalihog,
Mayor Nestor C. Lim and Mayor Antonio Kho of the crime of multiple murder and frustrated murder in
connection with the airport incident. The case was docketed as Criminal Case No. 9211.
After conducting the preliminary investigation, the court issued an order dated July 31, 1989
stating therein that . . . after weighing the affidavits and answers given by the witnesses for the
prosecution during the preliminary examination in searching questions and answers, concludes that a
probable cause has been established for the issuance of a warrant of arrest of named accused in the
amended complaint, namely, Jimmy Cabarles, Ronnie Fernandez, Nonilon Bagalihog, Jolly Fernandez,
Florencio Fernandez, Jr., Vicente Lim, Sr., Susana Lim, Nestor Lim, Antonio Kho, Jaime Liwanag, Zaldy
Dumalag and Rene Tuallaalias Tidoy.
Petitioners Vicente Lim, Sr. and Susana Lim filed with the respondent court several motions and
manifestations which in substance prayed that an order be issued requiring the transmittal of the initial
records of the preliminary inquiry or investigation conducted by the Municipal Judge Barsaga of Masbate
for the best enlightenment regarding the existence of a probable cause or prima facie evidence as well as
the determination of the existence of guilt, pursuant to the mandatory mandate of the constitution that no
warrant shall be issued unless the issuing magistrate shall have himself been personally convinced of such
probable cause.
In another manifestation, the Lims reiterated that the court conduct a hearing to determine if
there really exists a prima facie case against them in the light of documents which are recantations of
some witnesses in the preliminary investigation. It should also be noted that the Lims also presented to
the respondent Judge documents of recantation of witnesses whose testimonies were used to establish a
prima facie case against them.
On July 5, 1990, the respondent court issued an order denying for lack of merit the motions and
manifestations and issued warrants of arrest against the accused including the petitioners herein. The
judge wrote, In the instant cases, the preliminary investigation was conducted by the Municipal Trial
Court of Masbate, Masbate which found the existence of probable cause that the offense of multiple
murder was committed and that all the accused are probably guilty thereof, which was affirmed upon
review by the Provincial Prosecutor who properly filed with the Regional Trial Court four separate
informations for murder. Considering that both the two competent officers to whom such duty was
entrusted by law have declared the existence of probable cause, each information is complete in form and
substance, and there is no visible defect on its face, this Court finds it just and proper to rely on the
prosecutor's certification in each information
Petitioners question the judgment of Judge Felix.
ISSUE:
WON a judge may issue a warrant of arrest without bail by simply relying on the prosecution's
certification and recommendation that a probable cause exists?

RULING:
The questioned Order of respondent Judge Nemesio S. Felix of Branch 56, Regional Trial Court of Makati
dated July 5, 1990 is declared NULL and VOID and SET ASIDE. As held in Soliven v. Makasiar, the Judge
does not have to personally examine the complainant and his witnesses. The Prosecutor can perform the
same functions as a commissioner for the taking of the evidence. However, there should be necessary
documents and a report supporting the Fiscal's bare certification. All of these should be before the Judge.
We cannot determine beforehand how cursory or exhaustive the Judge's examination should be. Usually,
this depends on the circumstances of each case. The Judge has to exercise sound discretion; after all, the
personal determination is vested in the Judge by the Constitution. However, to be sure, the Judge must go
beyond the Prosecutor's certification and investigation report whenever necessary. As mentioned in the
facts (stated above), the Lims presented documents of recantations of the witnesses. Although, the general
rule is that recantations are not given much weight in the determination of a case and in the granting of a
new trial the respondent Judge before issuing his own warrants of arrest should, at the very least, have
gone over the records of the preliminary examination conducted earlier in the light of the evidence now
presented by the concerned witnesses in view of the "political undertones" prevailing in the cases. In
making the required personal determination, a Judge is not precluded from relying on the evidence
earlier gathered by responsible officers. The extent of the reliance depends on the circumstances of each
case and is subject to the Judge's sound discretion. However, the Judge abuses that discretion when
having no evidence before him, he issues a warrant of arrest. Indubitably, the respondent Judge
(Felix)committed a grave error when he relied solely on the Prosecutors certification and issued the
questioned Order dated July 5,1990 without having before him any other basis for his personal
determination of the existence of a probable cause.

G.R. Nos. 130568-69


March 21, 2000.
PEOPLE OF THE PHILIPPINES, plaintiff-appellee
vs.
CHE CHUN TING alias DICK, accuses-appellant
FACTS:
Accused-appellant, Che Chun Ting alias DICK, a Hong Kong national, was charged and
convicted for dispatching in transit and dispatching 999.43 grams of shabu and possessioning and control
5, 578.68 grams of the same drug. He contends that the methylamphetamine hydrochloride or shabu is
inadmissible in evidence as it was seized without a valid search warrant.
ISSUE:

Whether the white crystalline seized under his Unit without valid search warrant an
inadmissible in evidence.

RULING:
The lawful arrest being the sole justification for the validity of the warrantless search under the
exception, the same must be limited to and circumscribed by the subject, time and place of the arrest. As
to subject, the warrantless search is sanctioned only with respect to the person of the suspect, and things
that may be seized from him are limited to dangerous weapons or anything which may be used as proof
of the commission of the offense. With respect to the time and place of the warrantless search, it must be
contemporaneous with the lawful arrest. Stated otherwise, to be valid, the search must have been
conducted at about the time of the arrest or immediately thereafter and only at the place where the
suspect was arrested, or the premises or surroundings under his immediate control.
It must be stressed that the purposes of the exception are only to protect the arresting officer
against physical harm from the person being arrested who might be armed with a concealed weapon, and
also to prevent the person arrested from destroying the evidence within his reach. The exception therefore
should not be strained beyond what is needed in order to serve its purposes.
As a consequence of the illegal search, the things seized on the occasion thereof are inadmissible
in evidence under the exclusionary rule. They are regarded as having been obtained from a polluted
source, the fruit of a poisonous tree. However, objects and properties the possession of which is
prohibited by law cannot be returned to their owners notwithstanding the illegality of their seizure. Thus,
the shabu seized by the NARCOM operatives, which cannot legally be possessed by the accused under the
law, can and must be retained by the government to be disposed of in accordance with law.

GR No. 96177,
January 27, 1993

PEOPLE OF THE PHILIPPINES, plaintiff-appellee


Vs.
MARI MUSA y HANTATALU, accused-appellant

FACTS:
On December 14, 1989, in the City of Zamboanga City, accused-appellant, Mari Musa was
willfully, unlawfully and feloniously sells to one Sgt. Amado Ani, two (2) wrappers of plastic bags
containing dried marijuana leaves during the buy-bust operation.
Accused-appellant contends that seizure of the plastic bag is unreasonable, hence, inadmissible
evidence.
ISSUE:
Whether or not the seizure of the plastic bag and the marijuana inside it is unreasonable and the
found evidence is inadmissible.
RULING:

Yes. It constituted unreasonable search and seizure thus it may not be admitted as evidence. The
warrantless search and seizure, as an incident to a suspects lawful arrest, may extend beyond the person
of the one arrested to include the premises or surroundings under his immediate control. Objects in the
plain view of an officer who has the right to be in the position to have that view are subject to seizure and
may be presented as evidence. The plain view doctrine is usually applied where a police officer is not
searching for evidence against the accused, but nonetheless inadvertently comes across an incriminating
object. It will not justify the seizure of the object where the incriminating nature of the object is not
apparent from the plain view of the object.
In the case at bar, the plastic bag was not in the plain view of the police. They arrested the
accused in the living room and moved into the kitchen in search for other evidences where they found the
plastic bag. Furthermore, the marijuana inside the plastic bag was not immediately apparent from the
plain view of said object.
Therefore, the plain view does not apply. The plastic bag was seized illegally and cannot be
presented in evidence pursuant to Article III Section 3 (2) of the Constitution.

G.R. No. 81958 June 30, 1988


PHILIPPINE ASSOCIATION OF SERVICE EXPORTERS, INC., petitioner,
vs.
HON. FRANKLIN M. DRILON as Secretary of Labor and Employment, and TOMAS D.
ACHACOSO, as Administrator of the Philippine Overseas Employment
Administration, respondents.
FACTS:
The petitioner, Philippine Association of Service Exporters, Inc. (PASEI, for short), a firm
"engaged principally in the recruitment of Filipino workers, male and female, for overseas
placement," challenges the Constitutional validity of Department Order No. 1, Series of 1988, of the
Department of Labor and Employment, in the character of "GUIDELINES GOVERNING THE
TEMPORARY SUSPENSION OF DEPLOYMENT OF FILIPINO DOMESTIC AND HOUSEHOLD
WORKERS. Specifically, the measure is assailed for "discrimination against males or females;" that it
"does not apply to all Filipino workers but only to domestic helpers and females with similar skills;" and
that it is violative of the right to travel. It is held likewise to be an invalid exercise of the lawmaking power,
police power being legislative, and not executive, in character.
On May 25, 1988, the Solicitor General, on behalf of the respondents Secretary of Labor and
Administrator of the Philippine Overseas Employment Administration, filed a Comment informing the
Court that on March 8, 1988, the respondent Labor Secretary lifted the deployment ban in the states of
Iraq, Jordan, Qatar, Canada, Hongkong, United States, Italy, Norway, Austria, and Switzerland. * In
submitting the validity of the challenged "guidelines," the Solicitor General invokes the police power of
the Philippine State. The petitioner has proffered no argument that the Government should act similarly
with respect to male workers.
ISSUES:
Whether or not the Department Order No. 1 Series of 1988 of the Department of labor and
Employment
violates the constitution?
RULING: .
The discrimination of male workers from the female workers was justified because there is no
evidence that the male workers were abused. But the Order was not in contrary to the constitution
because it is the duty of the state to provide full protection to labor and that Department Order No. 1 did
not prescribe a total ban on overseas deployment and the alleged violation of right to travel was not
violated because such right must submit to the demands and necessities of the Statess power of
regulation for public safety.

[A.M. No. RTJ-00-1576. June 28, 2001]


SIMPLICIO ALIB, for himself and in behalf of the members of the MANDALAGAN SMALL
FARMERS COOPERATIVE, complainants, vs. JUDGE EMMA C. LABAYEN of the Regional
Trial Court, Bacolod City, Branch 46,respondent.

FACTS:
Complainants charge the respondent Judge Emma Labayen of the Regional Trial Court of
Bacolod City, Branch 46 with grave abuse of authority and grave misconduct. They allege that an
Information for Perjury docketed to Criminal Case No. 98-19271 was filed against several members of the
Mandalangan Small Farmers Cooperative with Regional Trial Court of Bacolod City. Judge Labayen
issued a warrant of arrest against the accused therein. The accused filed a "Motion for Re- investigation
and Recall of Warrant of Arrest" and a Supplemental thereto alleging that the court has no jurisdiction as
the crime of perjury is within the jurisdiction of the Municipal Trial Court in Cities.
Respondent Judge Labayen filed a Supplemental Pleading reiterating that as pairing judge of Branch 45,
it was ministerial on her part to sign warrants of arrest coming from Branch 45 and that when she realized
that the case was within the jurisdiction of the MTCC, she had the case remanded as shown in her Order
dated October 2, 1998. Respondent Judge claims she acted without malice and in good faith.
ISSUES:
Whether or not acting without malice and in good faith not knowing the exact probable cause of the
warrant of arrest issued constitutes gross ignorance of the law?
RULING:
Yes, before issuing a warrant of arrest, a judge must not rely solely on the report or resolution of
the prosecutor, he must evaluate the report and the supporting documents which will assist him to make
his determination of probable cause. A finding of the existence of a probable cause is a pre-requisite to the
issuance of a warrant of arrest and strict compliance therewith is required of judges.

Republic of the Philippines


SUPREME COURT
Manila
EN BANC
G.R. No. 81510 March 14, 1990
HORTENCIA SALAZAR, petitioner,
vs.
HON. TOMAS D. ACHACOSO, in his capacity as Administrator of the Philippine Overseas
Employment Administration, and FERDIE MARQUEZ, respondents.
FACTS:
On October 21, 1987, Rosalie Tesoro of 177 Tupaz Street, Leveriza, Pasay City, in a sworn
statement filed with the Philippine Overseas Employment Administration (POEA for brevity) charged
petitioner Hortencia Salazar. On November 3, 1987, public respondent Atty. Ferdinand Marquez to whom
said complaint was assigned, sent to the petitioner a telegram to summon him. On the same day, having

ascertained that the petitioner had no license to operate a recruitment agency, public respondent
Administrator Tomas D. Achacoso issued his challenged CLOSURE AND SEIZURE ORDER NO. 1205.
On January 26, 1988 POEA Director on Licensing and Regulation Atty. Estelita B. Espiritu issued
an office order designating respondents Atty. Marquez, Atty. Jovencio Abara and Atty. Ernesto Vistro as
members of a team tasked to implement Closure and Seizure Order No. 1205. Doing so, the group assisted
by Mandaluyong policemen and mediamen Lito Castillo of the People's Journal and Ernie Baluyot of
News Today proceeded to the residence of the petitioner at 615 R.O. Santos St., Mandaluyong, Metro
Manila. There it was found that petitioner was operating Hannalie Dance Studio. Before entering the
place, the team served said Closure and Seizure order on a certain Mrs. Flora Salazar who voluntarily
allowed them entry into the premises. Mrs. Flora Salazar informed the team that Hannalie Dance Studio
was accredited with Moreman Development (Phil.). However, when required to show credentials, she was
unable to produce any. Inside the studio, the team chanced upon twelve talent performers practicing a
dance number and saw about twenty more waiting outside, The team confiscated assorted costumes
which were duly receipted for by Mrs. Asuncion Maguelan and witnessed by Mrs. Flora Salazar.
On January 28, 1988, petitioner filed with POEA stating to return the confiscated items on the
ground that the said seizure was contrary to law and against the will of the owner and among the others
reasons are they have not been given any prior notice or hearing, hence the Closure and Seizure Order No.
1205 dated November 3, 1987 violates "due process of law" guaranteed under Sec. 1, Art. III, of the
Philippine Constitution. Their action also violates Sec. 2, Art. III of the Philippine Constitution which
guarantees right of the people "to be secure in their persons, houses, papers, and effects against
unreasonable searches and seizures of whatever nature and for any purpose." And lastly the premises
invaded by Mr. Ferdi Marquez and five (5) others (including 2 policemen) are the private residence of the
Salazar family, and the entry, search as well as the seizure of the personal properties belonging to our
client were without her consent and were done with unreasonable force and intimidation, together with
grave abuse of the color of authority, and constitute robbery and violation of domicile under Arts. 293 and
128 of the Revised Penal Code.
ISSUES:
Whether the promulgated Presidential Decree No. 2018 is unconstitutional?
RULING:
Yes, the promulgated Presidential Decree was unconstitutional because under Article III, Section
2, of the l987 Constitution, it is only judges, and no other, who may issue warrants of arrest and search
where the only exception is in cases of deportation of illegal and undesirable aliens, for the purpose of
deportation.

SECOND DIVISION
[G.R. No. 138881. December 18, 2000]
THE PEOPLE OF THE PHILIPPINES, plaintiff-appellee, vs.LEILA JOHNSON Y
REYES, accused-appellant.
FACTS:
Accused-appellant Leila Reyes Johnson was, at the time of the incident, 58 years old, a widow,
and a resident of Ocean Side, California, U.S.A. She is a former Filipino citizen who was naturalized as an
American on June 16, 1968 and had since been working as a registered nurse, taking care of geriatric
patients and those with Alzheimers disease, in convalescent homes in the United States.
On June 16, 1998, she arrived in the Philippines to visit her sons family in Calamba, Laguna. She
was due to fly back to the United States on July 26. On July 25, she checked in at the Philippine Village
Hotel to avoid the traffic on the way to the Ninoy Aquino International Airport (NAIA) and checked out at
5:30 p.m. the next day, June 26, 1998.
At around 7:30 p.m. of that day, Olivia Ramirez was on duty as a lady frisker at Gate 16 of
the NAIA departure area. Her duty was to frisk departing passengers, employees, and crew and check for
weapons, bombs, prohibited drugs, contraband goods, and explosives.
When she frisked accused-appellant Leila Johnson, a departing passenger bound for the United
States via Continental Airlines CS-912, she felt something hard on the latters abdominal area. Upon
inquiry, Mrs. Johnson explained she needed to wear two panty girdles as she had just undergone an
operation as a result of an ectopic pregnancy.
Not satisfied with the explanation, Ramirez reported the matter to her superior, SPO4 Reynaldo
Embile, saying Sir, hindi po ako naniniwalang panty lang po iyon. (Sir, I do not believe that it is just a
panty.) She was directed to take accused-appellant to the nearest womens room for inspection. Ramirez
took accused-appellant to the rest room, accompanied by SPO1 Rizalina Bernal. Ramirez then asked her
to bring out the thing under her girdle. Accused-appellant brought out three plastic packs, which
Ramirez then turned over to Embile, outside the womens room. The confiscated packs, marked as
Exhibits C-1, C-2 and C-3, contained a total of 580.2 grams of a substance which was found by NBI
Chemist George de Lara to be methamphetamine hydrochloride or shabu.
ISSUES:
Whether the arrest without warrant is lawful?
RULING:
Yes, because according to Section 5, Rule 113 of the 1985 Rules of Criminal Procedure provides
that a peace officer or a private person may, without a warrant, arrest a person:(a) when in his presence,

the person to be arrested has committed, is actually committing, or is attempting to commit an offense;
(b) when an offense has in fact just been committed, and he has personal knowledge of facts indicating
that the person to be arrested has committed it.

[G.R. Nos. 132875-76. February 3, 2000]


PEOPLE OF THE PHILIPPINES, plaintiff-appellee,
Vs
. ROMEO G. JALOSJOS, accused-appellant.
RESOLUTION
FACTS:The accused-appellant, Romeo G. Jalosjos is a full-fledged member of Congress who is now
confined at the national penitentiary while his conviction for statutory rape on two counts and acts of
lasciviousness on six counts[1] is pending appeal. The accused-appellant filed this motion asking that he be
allowed to fully discharge the duties of a Congressman, including attendance at legislative sessions and
committee meetings despite his having been convicted in the first instance of a non-bailable offense.
ISSUES: Does being an elective official result in a substantial distinction that allows different treatment?
RULING: The Court cannot validate badges of inequality. The necessities imposed by public welfare may
justify exercise of government authority to regulate even if thereby certain groups may plausibly assert
that their interests are disregarded.

People
vs.
Lacerna
G.R. No. 109250
September 25, 1997
Facts:
On September 12, 1992, Marlon Lacerna and Noriel Lacerna were aboard in a taxi cab when the
mobile patrol car of PO3 Angelito Camer and PO3 Carlito P. Valenzuela, members of the Mobile Patrol
Division of Western Police District stop them for being so suspicious. The police officer asked permission
if they can search the vehicle, and the occupants answered yes. When the police officer went searching
they found in the occupants luggage a knapsack and a dark blue plastic bag. When the police officer asked
what is the content of the plastic bag, Noriel Lacerna immediately answered that it contains his
vomit. Sceptical to Noriel Lacernas answer, PO3 Valenzuela made a small hole and peeped inside and
saw several bricks wrapped in a newspaper. Officer Valenzuela took a brick and when he opened it, it
turned out to be Marijuana, a prohibited drug.
Issue:

Whether or not the bricks of Marijuana be admissible in the court and use as evidence against the
accused
Held:
Yes, the constitutional right of the accused against unreasonable searches and seizure are not
violated and the evidences are obtained legally and the evidence does not constitute as fruit of a
poisonous tree. The accused allowed to be searched when he gave the consent to be search to the police
officers. It was his consent which validated the search, waiver being a generally recognized exception to
the rule against warrantless search. The marijuana bricks were, therefore, obtained legally through a valid
search and seizure. They were admissible in evidence.

Jose Burgos
vs.
Chief of Staff
G.R. No L-64261
December 26, 1984
Facts:
Two warrants were issued against petitioners for the search on the premises of Metropolitan Mail
and We Forum newspapers and the seizure of items alleged to have been used in subversive activities.
Petitioners prayed that a writ of preliminary mandatory and prohibitory injunction be issued for the
return of the seized items and articles, and thus those seized articles will not be used by the respondents
against the petitioners in the court.
The petitioners questioned the warrants for the lack of probable cause and that the two warrants
issued indicated only one and the same address. In addition, the items seized subject to the warrant were
real properties.
Issue:
Whether or not, the two warrants are valid to justify the seizure of the items.
Held:
The warrants are null and void. The Court found out that the affidavit for the application of the
warrant did not satisfy the requirement of probable cause because the statements of the witnesses are
mere generalizations. Furthermore, jurisprudence prohibits the issuance of general warrants. In this case,
the warrant issued doesnt describe and enumerate the items to be searched and seized and did not
indicate and specify the subversive nature of the said items.

People
vs.
Mendoza
G.R. No. 109279
January 18, 1999
Facts:
On November 11, 1988, Octavio Mendoza, his wife Cecilia and their 10 yr. Old daughter
Charmaine went to the birthday party of a relative in McDonalds, Harrison Plaza. During the party,
Octavio left without telling his wife and kid, and went to Kentucky Fried Chicken and Restaurant where he
had some beer. Since Cecilia and Charamaine couldnt find him in the party, they went home at #2
TRAMO ST. CAMELLA HOMES, PHASE III, PAMPLONA, LAS PINAS. They arrived home at 7pm, and
still Octavio was not at home and Cecilia left again to go to her parents in Bacoor to bring perfume. They
got home around 9pm and saw Octavios car parked in the garage of their neighbour. All the lights were
opened but the front door was locked. After a while, Octavio opened the back door and let them in. He was
drunk and told her daughter to get cold water and douse him. She followed and was instructed to go to her
room. She went and got ready for bed. She heard her parents arguing about them leaving the party
without Octavio. Afterwards, she heard THREE GUNSHOTS, ran to their room and saw her mom on the
floor bleeding. She also saw her dad hide a gun under the bed. Octavio called his brother-in-law Sgt.
Antonio Gabac. When Gabac arrived, they all brought him to Perpetual Help Hospital where Cecila was
declared dead on arrival. The policemen investigated Gabac and found a gun in his waist. A .38 calibre
revolver. He told them that Octavio handed it over to him as soon as he arrived at the crime scene.
Cecilias father, Alipio Eusebio learned of his daughters death and that valuable were being taken away
from her house. He and his sons decided to go there and remove the rest of the property, including a
memorandum receipt signed by Octavio and a mission order authorizing him to carry such weapon. At
court, Charmaine testified that she saw her father hide the gun under the bed. On her second testimony,
she said she saw no such act. Octavio also denied that he killed his wife and that he owned that gun. He
said that the memorandum receipt and mission order were illegally procured by Eusebio in violation of
his right against unreasonable search and seizure.
Issue:
Does the Constitutional right of Octavio Mendoza against unreasonable searches and seizures
have been violated when Eusebio took the memorandum receipt to the court?

Held:
No, because the peoples Constitutional right against unreasonable searches and seizure can only
be invoke if there is interference from the Government and it these rights cannot be extended if the acts
are committed by private individuals. In this case, the memorandum receipt and other articles were
discovered by Alipio, Cecilias father, a private individual and handed it over to Eusibio.

Peralta
vs.
COMELEC
G.R.No. L-47771
March 11, 1978
Facts:
Peralta was an independent candidate in the April 1978 Interim Batasang Pambansa Elections.
He, along with others, assailed the constitutionality of PD 1269 or the 1978 Election Code. Secs140 and
155, sub-paragraphs 26 to 28, of the 1978 Election Code, grants the voter the option to vote either for
individual candidates by filling in the proper spaces in the ballot the names of candidates he desires to
elect, or to vote for all the candidates of a political party, group or aggrupation by simply writing in the
space provided for in the ballot the name of the political party, group or aggrupation or office-block ballot.
Peralta was determined in contending that the optional block voting scheme is violative of this provision
of the Constitution: Bonafide candidates for any public office shall be free from any form of harassment
and discrimination. He sought the shelter of its protection for himself and other independent candidates
who, according to him, would be thus made to suffer if the assailed provision is not nullified. Essentially,
in terms of individual rights, he would raise a due process and equal protection question. The main
objection of Peralta against the optional straight party voting provided for in the Code is that an
independent candidate would be discriminated against because by merely writing on his ballot the name
of a political party, a voter would have voted for all the candidates of that party, an advantage which the
independent candidate does not enjoy. In effect, it is contended that the candidate who is not a partymember is deprived of the equal protection of the laws, as provided in Sec 1 of Article IV, in relation to Sec
9 of Article XII, of the 1973 Constitution.
Issue:
Whether or not 1978 Election Code violates the equal protection clause
Held:
The SC ruled that the 1978 Election Code is valid. Before a voter prepares his ballot, the voter will
be able to read all the names of the candidates. No candidate will receive more than one vote, whether he
is voted individually or as a candidate of a party group or aggrupation. The voter is free to vote for the
individual candidates or to vote by party, group or aggrupation. The choice is his. No one can compel him
to do otherwise. In the case of candidates, the decision on whether to run as an independent candidate or
to join a political party, group or aggrupation is left entirely to their discretion. Certainly, before filing his
certificate of candidacy, a candidate is aware of the advantages under the law accruing to candidates of a
political party or group. If he wishes to avail himself of such alleged advantages as an official candidate of
a party, he is free to do so by joining a political party group or aggrupation. In other words, the choice is
his. In making his decision, it must be assumed that the candidate had carefully weighed and considered
the relative advantages and disadvantages of either alternative. So long as the application of the rule

depends on his voluntary action or decision, he cannot, after exercising his discretion, claim that he was
the victim of discrimination.

Waterous Drug Corporation


vs.
NLRC
G.R. No. 113271
October 16, 1997
Facts:
On August 15, 1988, Antonia Melodia Catolico was hired as a pharmacist by Waterous Drug
Corporation. Catolico purchased 10 bottles of Voren tablets from Yung Shin Pharmaceuticals a price of
364php per bottle with its original price of 320php per bottle, overpriced amounting 64php per bottle.
Yung Shin Pharmaceuticals sent a check to WDRC for refund of jack-up price amounting 640php
addressed to Catolico. Ms. Saldana, the WDRC clerk opened the envelope and found out that there was a
check amounting 640php issued by YSP. On March 5, 1990, Waterous Drug Corporation issued a
memorandum for Catolicos termination by reason of dishonesty.
Issue:
Whether or not the check admissible in the court as evidence against Catolico
Held:
Yes. The rationale is the doctrine laid down in the case of People vs. Marti which states that the
Bill of Rights does not protect citizens from unreasonable searches and seizures perpetrated by private
individuals. Since the envelope was opened by Saldana, a private individual, the check is admissible to
court as evidence.

G.R. No. 113811, October 7, 1994


ISHMAEL HIMAGAN, petitioner,
vs.
People of the Philippines and Hon. Judge Mapayo ,RTC, Br. 11, Davao City, respondents
Facts:
Ishmael Himagan, a policeman was assigned in the medical company of the Philippine National
Police Regional Headquarters at Camp Catitigan, was implicated in the killing of Benjamin Machitar, Jr.
and the attempted murder of Bernabe Machitar. After the informations were filed with the Regional Trial
Court of Davao City on September 16, 1992, the trial court issued and Order suspending petitioner until
the termination of the case on the basis of Section 47, R.A. 6975, otherwise known as Department of
Interior and Local Government Act of 1990, which provides the accused suspension from the office until
the case is terminated and such case shall be terminated within 90 days from arraignment of the accused.
The petitioner filed to lift the order of suspension relying on Section 42 of P.D. 807 of the Civil Service
Decree and cases of Layno & Deloso but his motion was denied.
Issue:
1.Whether or not the preventive suspension of the petitioner may be lifted if the case is not
terminated within 90 days.
Ruling:
There is nothing in the R.A. 6975 that suggests that the preventive suspension of the accused will
be lifted if the trial is not terminated within the period. Nonetheless, the Judge who fails to decide the case
within the period without justifiable reason may be subject to administrative sanctions. If the trial is
unreasonably delayed without the fault of the accused such that he is deprived of his right to a speedy
trial, he is not without a remedy, he may ask for the dismissal of the case.
The petitioner had also misapplies Sec, 42 of PD 807 which clearly shows the lifting of preventive
suspension in pending ADMINISTRATIVE INVESTIGATION and not in CRIMINAL CASES.
The petitioners reliance on the cases Layno and Deloso is misplaced for the 2 cases only involved
in graft and corruption and not in the present case which Himagan is accused in the crime of murder
under the Revise Penal Code. It is undisputed that he falls under Sec. 47 of R.A. 6975 which categorically
states that his suspension shall last until the case is terminated.

Furthermore, the reason why members of the PNP are treated differently from other classes of
persons charged criminally or administratively insofar as the application of the rule on preventive
suspension is concerned is that the policemen carry weapons and the badge of the law which can be used
to harass or intimidate witnesses against them, as succinctly brought out in the legislative discussions.
The petition is hereby denied.

G.R. No. 82544, June 28, 1988


Andrew Harvey, John Sherman and Adriaan Van Del Elshout, petitioners,
vs.
Honorable Commissioner Miriam Defensor Santiago, Commission on Immigration and
Deportation, respondent.
Facts:
Andrew Harvey( 52 years old) and John Sherman (72 years old) are both American nationals
residing at Pagsanjan, Laguna while Adriaan Van Elshout (58 years old) is a Dutch citizen also residing at
Pagsanjan, Laguna. The petitioners were among the twenty-two (22) suspected alien pedophiles who were
apprehended after the three months of close surveillance by CID (Commission on Immigration and
Deportation) agents in Pagsanjan, Laguna.
Seized during petitioners arrest were rolls of photo negatives and photos of the suspected child
prostitutes shown in salacious poses as well as boys and girls engaged in the sex act. There were also
posters and other literature advertising the child prostitutes. During the operation Harvey was found
together with two young boys and Sherman was found with two naked young boys inside his room and in
the case of Van Den Elshout there were two children ages 14-16 which subject readily accepted having
been in his care and live-in for quite some time. Petitioners availed for this petition a writ of Habeas
Corpus questioning the validity of their detention.
Issue:
1.

Whether or not the respondent has the authority to arrest and detain the petitioners pending
determination of the existence of probable cause leading to an administrative investigation.

2.

Whether or not the CID agents were clothed with valid warrants, search and seizure as required by the
Constitution.
Ruling:

The right against unreasonable searches and seizure which is guaranteed by the constitution is
available to all persons including ALIENS whether accused of a crime or not. One of the constitutional
requirements of a valid search warrant or warrant of arrest is that it must have a probable cause. In this
case, the arrest of petitioners was based on a probable cause determined after a close surveillance for
three (3) months during which period their activities was monitored. The existence of probable cause
justified the arrest and the seizure of the photo negatives, photographs and posters and the said articles
were seized as an incident to a lawful arrest and are therefore, admissible in evidence.
The petitioners were not caught in the act does not make their arrest illegal. The Petitioners
were found with young boys in their respective rooms, the ones with Sherman were naked. The CID
agents had a reasonable ground to believe that the petitioners had committed pedophilia and it is a
behavior which is offensive to public morals and against State policies. . Also it is a fundamental rule that
a writ of habeas corpus will not be granted when the confinement is or has become legal, although such
confinement was illegal at the beginning. The petition was dismissed and the Habeas Corpus was denied.

G.R. No. 11920, September 20, 1996


People of the Philippines, plaintiff-appellee,
vs.
Nilo Solayao, accused-appelant
Facts:
On July 9, 1992 about 9 oclock in the evening SPO3 Jose Nino with CAFGU members conducted
intelligence patrol in the Barangay Caulangohan, Caibiran, Biliran to verify reports on the presence of
armed persons roaming around the barangays of Caibiran. From Caulangohan they proceeded to another
barangay and met the group of the accused Nilo Solayao. The group of Spo3 Jose Nino had become
suspicious when they observed that that the latter were drunk and the accused himself was wearing a
camouflage uniform. Upon seeing the government agents the group fled and Nilo was left behind. The
Police officer Nino had seized the dried coconut leaves which Nilo was carrying and found wrapped a 49inch long homemade firearm locally known as latong. SPO3 confiscated the firearm and turned him
over to the custody of the policeman of Cabiran and was charged before the RTC of Naval, Biliran with the
crime of illegal possession of firearm and ammunition.
Issue:

1.

Whether or not there is an unlawful search warrant.

2.

Whether or not the prosecutor was able to prove that there is an absence of a license or permit to possess
the subject firearm.
Ruling:
The argument of the accused-appelant in which there is an unlawful search warrant is hardly
tenable. He and his companions drunken actuations aroused the suspicious of SPO3 Ninos group as well
as the fact that he himself was attired in a camouflage uniform or single suit and upon seeing the
government agents, his companions fled. It should be noted that the peace officers were precisely on an
intelligence mission to verify reports that armed persons were roaming around the barangay of Caibiran.
There was a justifiable cause to stop and frisk the accused when his companion fled upon seeing the
group of SPO3 Nino. Thus, there was no violation of the constitutional guarantee against unreasonable
searches and seizures.
The prosecutor was only able to prove by testimonial evidence that the accused-appelant
admitted before the Police Officer at the time he was accosted that he did not have any authority or license
to carry the subject firearm when he was asked if he had one, the prosecutor had relied in the accused
appellants admission and didnt do its duty to establish the lack of a license or permit to carry the firearm
by clear and convincing evidence, like a certification from the government agency concerned. Thus, Nilo
Solayao was acquitted for insuffiency of evidence.

G.R. No. 11318, August 28, 1996


Columbia pictures Inc., petitioners
vs.
Court of Appeals, respondents
Facts:
The Petitioners with the National Bureau of Investigation filed a complaint for violation of P.D.
49 and sought assistance with the NBIS anti-film privacy against Sunshine Home Video Inc. which is
owned and operated by Danilo A. Pelindario. On November 14, 1987 NBI Senior Agent Reyes applied for a
search warrant with a court a quo against Sunshine Home Video Inc. seeking the seizure, among others, of
pirated video tapes of copyrighted films and television sets, video cassettes, laser recording equipments
and other machines and paraphernalia used for the unlawful exhibition, showing, reproduction, sale,

lease or disposition of videogram tapes in the premises. The search warrant was served about 1:45 p.m. on
December 14, 1987 and seized various video tapes of duly copyrighted motion pictures/films owned or
exclusively distributed by private complainants, machines and equipments. On December 16, 1987 a
RETURN OF SEARCH WARRANT was filed with the court and MOTION TO LIFT THE ORDER OF
SEARCH WARRANT for lack of merit in which the master tapes from which the pirated films were
allegedly copies were never presented in the proceedings in the issuance of search warrant.
Issue:
Whether or not there is a probable cause for the issuance of the Search Warrant.
Ruling:
Probable cause for a search warrant has been defined as such facts and circumstances which
would lead a reasonably discrete and prudent man to believe that an offense has been committed and that
objects sought in connection with the offense are in the place sought to be search. Hence, the applicant
must present to the court the copyrighted films to compare with the purchase evidence of the video tapes
allegedly pirated to determine whether the latter is an authorized reproduction of the owner. The judge
bases for the issuance of the search warrant is only the personal knowledge of the subject matter of NBI
Agent Reyes, Atty. Rico Domingo and Rene Baltazar and also their respective testimonies without stating
the fact by which these were pirated and it is a conclusion of facts without basis. A search warrant not
based on a probable cause is a nullity, or is void, and the issuance thereof is, in legal contemplation and
arbitrary.

G.R. No. 95122-23, May 31, 1991


Board of Commissioner, petitioner
vs.
Judge De la Rosa
Facts:
Santiago Gatchalian, grandfather of William Gatchalian, was recognized by the Bureau of
Immigration as a native born Filipino citizen following the citizenship of his natural mother, Machana
Gatchalian, on July 12, 1960. Santiago had 5 children and one of those children had a son named William

Gatchalian who arrived in Manila from Hongkong together with Gloria, Francisco and Johnson carried
with them the Certificates of Registration and identity issued by the Philippine Consulate in Hongkong
which was signed by the Secretary of Foreign Affairs. On January 24, 1962 the Secretary of Justice issued
a memorandum No. 9 directed the Board of Commissioner to review all the cases where entry was allowed
on the ground that the entrant was a Philippine Citizen among these cases was William Gatchalian and
others. The citizenship of William was questioned and he was alleged as an alien. A warrant of arrest was
issued by the Commissioner of Immigration for purposes of investigation of William.
Issue:
Whether or not the warrant of arrest issued by the Commissioner of Immigration is null and void
for being unconstitutional.
Ruling:
The Immigration Act of 1940 reads that the following aliens shall be arrested upon the warrant
of the Commissioner of Immigration or any other officer designated by him for the purpose and deported
upon the warrant of the Commissioner of Immigration after a determination by the Board of
Commissioner of the existence of the ground for deportation as charged against the alien.
From a perusal of the above provision, it is clear that in matters of implementing the
Immigration Act insofar as deportation of aliens are concerned, the Commissioner of Immigration may
issue warrants of arrests only after a determination by the Board of Commissioners of the existence of the
ground for deportation charged against the alien. In other words, a warrant of arrest issued by the
Commissioner of Immigration, to be valid, must be for the sole purpose of executing final order of
deportation.
It is not necessary that an alien be arrested for the purpose of investigation. If the purpose of the
issuance of warrant of arrest is to determine the existence of probable cause, surely, it cannot pass the test
of constitutionality for only JUDGES can issue the same.
Hence, a warrant issued by the Commissioner of Immigration for purposes of investigation only
is null and void for being unconstitutional.

G.R. No. L-45987 May 5, 1939


The People of the Philippines
Vs
Cayat
Facts:

Cayat is a native of Baguio prosecuted for violation of Act No. 1639 and was sentenced by the
justice of the peace of Baguio to pay a fine or suffer subsidiary imprisonment in case of insolvency.
On January 25, 1937, the City of Baguio accused Cayat of illegally possessing a gin, which the
members of his tribe have been accustomed themselves to make prior to the passage of Act No. 1639.
Cayat interposed a demurrer which was overruled. At the trial, Cayat admitted the alleged facts but
pleaded not guilty. But trial court found him guilty and sentenced him of the fine or imprisonment.
Cayat challenges the constitutionality of the Act on the following grounds: (1) That it is
discriminatory and denies the equal protection of the laws;
(2) That it is violative of the due process clause of the Constitution: and.
(3) That it is improper exercise of the police power of the state.
Issue:
1.Whether or not Cayat accused of illegally possessing a gin?
2. Whether or not that is improper exercise of the police power of the state?
Held:
It is an established principle of constitutional law that the guaranty of the equal protection of the
laws is not equal protection of the laws is not violated by a legislation based on reasonable classification.
And the classification, to be reasonable, (1) must rest on substantial distinctions; (2) must be germane to
the purposes of the law; (3) must not be limited to existing conditions only; and (4) must apply equally to
all members of the same class.
(1) Substantial Distinction: . It is not based upon "accident of birth or parentage," as counsel to the
appellant asserts, but upon the degree of civilization and culture. "The term 'non-Christian tribes' refers,
not to religious belief, but, in a way, to the geographical area, and, more directly, to natives of the
Philippine Islands of a low grade of civilization, usually living in tribal relationship apart from settled
communities." This distinction is unquestionably reasonable, for the Act was intended to meet the
peculiar conditions existing in the non-Christian tribes.
(2) Germane to the purpose of law: it is unquestionably designed to insure peace and order in and among
the non-Christian tribes. It has been the sad experience of the past, as the observations of the lower court
disclose, that the free use of highly intoxicating liquors by the non-Christian tribes have often resulted in
lawlessness and crimes, thereby hampering the efforts of the government to raise their standard of life
and civilization.
(3) it must not be limited to conditions: The law is not limited in its application to conditions existing at
the time of its enactment. It is intended to apply for all times as long as those conditions exist.

(4) apply to all members of the class: that the Act applies equally to all members of the class is evident
from a perusal thereof. That it may be unfair in its operation against a certain number non-Christians by
reason of their degree of culture, is not an argument against the equality of its application.
Act No. 1639, as above stated, is designed to promote peace and order in the non-Christian tribes so as to
remove all obstacles to their moral and intellectual growth and, eventually, to hasten their equalization
and unification with the rest of their Christian brothers. Its ultimate purpose can be no other than to unify
the Filipino people with a view to a greater Philippines.

People
vs
Encinaba, G.R. No. April 3, 1998
Facts:
On December 13, 1988, P/Lt. Abello was tipped off by his informant, known only as Benjie, that a
certain "Aling Rosa" would be arriving from Baguio City the following day, December 14, 1988, with a
large volume of marijuana. Acting on said tip, P/Lt. Abello assembled a team composed of P/Lt. Jose
Domingo, Sgt. Angel Sudiacal, Sgt. Oscar Imperial, Sgt. Danilo Santiago and Sgt. Efren Quirubin.
Said team proceeded to West Bajac-Bajac, Olongapo City at around 4:00 in the afternoon of
December 14, 1988 and deployed themselves near the Philippine National Bank (PNB) building along
Rizal Avenue and the Caltex gasoline station. Dividing themselves into two groups, one group, made up of
P/Lt. Abello, P/Lt. Domingo and the informant posted themselves near the PNB building while the other
group waited near the Caltex gasoline station.
While thus positioned, a Victory Liner Bus with body number 474 and the letters BGO printed on
its front and back bumpers stopped in front of the PNB building at around 6:30 in the evening of the same
day from where two females and a male got off. It was at this stage that the informant pointed out to the
team "Aling Rosa" who was then carrying a traveling bag.
Having ascertained that accused-appellant was "Aling Rosa," the team approached her and
introduced themselves as NARCOM agents. When P/Lt. Abello asked "Aling Rosa" about the contents of
her bag, the latter handed it to the former.
Upon inspection, the bag was found to contain dried marijuana leaves packed in a plastic bag
marked "Cash Katutak." The team confiscated the bag together with the Victory Liner bus ticket to which
Lt. Domingo affixed his signature. Accused-appellant was then brought to the NARCOM office for
investigation where a Receipt of Property Seized was prepared for the confiscated marijuana leaves.

Upon examination of the seized marijuana specimen at the PC/INP Crime Laboratory, Camp
Olivas, Pampanga, P/Maj. Marlene Salangad, a Forensic Chemist, prepared a Technical Report stating
that said specimen yielded positive results for marijuana, a prohibited drug.
After the presentation of the testimonies of the arresting officers and of the above technical
report, the prosecution rested its case.
Instead of presenting its evidence, the defense filed a "Demurrer to Evidence" alleging the
illegality of the search and seizure of the items thereby violating accused-appellant's constitutional right
against unreasonable search and seizure as well as their inadmissibility in evidence.
The said "Demurrer to Evidence" was, however, denied without the trial court ruling on the
alleged illegality of the search and seizure and the inadmissibility in evidence of the items seized to avoid
pre-judgment. Instead, the trial court continued to hear the case.
In view of said denial, accused-appellant testified on her behalf. As expected, her version of the
incident differed from that of the prosecution. She claimed that immediately prior to her arrest, she had
just come from Choice Theater where she watched the movie "Balweg." While about to cross the road, an
old woman asked her help in carrying a shoulder bag. In the middle of the road, Lt. Abello and Lt.
Domingo arrested her and asked her to go with them to the NARCOM Office.
During investigation at said office, she disclaimed any knowledge as to the identity of the woman
and averred that the old woman was nowhere to be found after she was arrested. Moreover, she added
that no search warrant was shown to her by the arresting officers.
After the prosecution made a formal offer of evidence, the defense filed a "Comment and/or
Objection to Prosecution's Formal Offer of Evidence" contesting the admissibility of the items seized as
they were allegedly a product of an unreasonable search and seizure.
Not convinced with her version of the incident, the Regional Trial Court of Olongapo City
convicted accused-appellant of transporting eight (8) kilos and five hundred (500) grams of marijuana
from Baguio City to Olongapo City in violation of Section 4, Article 11 of R.A. No. 6425, as amended,
otherwise known as the Dangerous Drugs Act of 1972 and sentenced her to life imprisonment and to pay a
fine of twenty thousand (P20,000.00) pesos without subsidiary imprisonment in case of insolvency. 2uin
Issue:
Whether or not accused-appellant Rosa Aruta y Menguin charge for with a large volume of Marijuana?
Ruling:
While conceding that the officer making the unlawful search and seizure may be held criminally
and civilly liable, theStonehill case observed that most jurisdictions have realized that the exclusionary
rule is "the only practical means of enforcing the constitutional injunction" against abuse. This approach
is based on the justification made by Judge Learned Hand that "only in case the prosecution which itself
controls the seizing officials, knows that it cannot profit by their wrong, will the wrong be repressed." 35
Unreasonable searches and seizures are the menace against which the constitutional guarantees
afford full protection. While the power to search and seize may at times be necessary to the public welfare,

still it may be exercised and the law enforced without transgressing the constitutional rights of the
citizens, for the enforcement of no statute is of sufficient importance to justify indifference to the basic
principles of government. 36
Those who are supposed to enforce the law are not justified in disregarding the rights of the
individual in the name of order. Order is too high a price to pay for the loss of liberty. As Justice Holmes
declared: "I think it is less evil that some criminals escape than that the government should play an
ignoble part." It is simply not allowed in free society to violate a law to enforce another, especially if the
law violated is the Constitution itself. 37
WHEREFORE, in view of the foregoing, the decision of the Regional Trial Court, Branch 73,
Olongapo City, is hereby REVERSED and SET ASIDE. For lack of evidence to establish her guilt beyond
reasonable doubt, accused-appellant ROSA ARUTA Y MENGUIN is hereby ACQUITTED and ordered
RELEASED from confinement unless she is being held for some other legal grounds. No costs.

PEOPLE
VS.
GERENTE
G.R. No. 95847-48, March 10 1993
FACTS:
Edna Edwina Reyes testified that Gabriel Gerente, together with FredoEchigoren and Totoy
Echigoren, started drinking liquor and smoking marijuana in the house of the appellant. She overheard
the three mentalking about their intention to kill Clarito Blace. Fredo, Totoy Echigoren and Gerente
carried out their plan to kill Clarito Blace . Reyes, testified that she witnessed the killing as
follows: Fredo Echigoren struck the first blow against Clarito Blace, followed by Totoy Echigoren and
Gabriel Gerente who hit him twice with a piece of wood in the head and when he fell, Totoy
Echigoren dropped a hollow block on the victim's head. Thereafter, the three men dragged Blace to a place
behind the house of Gerente.
Patrolman Jaime Urrutia of the Valenzuela Police Station received a report from the Palo Police
Detachment about a mauling incident. He went to the Valenzuela District Hospital where the victim was
brought. He was informed by the hospital officials that the victim died on arrival. The cause of death was
massive fracture of the skull caused by a hard and heavy object.
Right away, Patrolman Urrutia, proceeded to Paseo de Blas where the mauling incident took
place. There they found a piece of wood with blood stains, a hollow block and two roaches of marijuana.
They were informed by Reyes, that she saw the killing and she pointed to Gabriel Gerente as one of the
three men who killed Clarito.
The policemen proceeded to the house of the appellant who was then sleeping. They told him to
come out of the house and they introduced themselves as policemen. Patrolman Urrutia frisked appellant

and found a coin purse in his pocket which contained dried leaves wrapped in cigarette foil. Only the
appellant, Gabriel Gerente, was apprehended by the police. The other suspects, Fredo and Totoy
Echigoren, are still at large.
Two separate informations were filed by Assistant Provincial Prosecutor Benjamin Caraig against
him for Violation of Section 8, Article II, of Republic Act No. 6425, and for Murder. The trial court
convicted him of Violation of Section 8 of R.A. 6425 and of Murder.
ISSUES:
1. Whether or not the court erred in admitting the marijuana leaves adduced in evidence by the
prosecution; and
2. Whether or not the court erred in convicting the accused-appellant of the crimes charged despite
the absence of evidence required to prove his guilt beyond reasonable doubt.
HELD:
The appealed decision was affirmed.
ARREST
The policemen arrested Gerente only some three (3) hours after Gerente and his companions had
killed Blace. They saw Blace dead in the hospital and when they inspected the scene of the crime, they
found the instruments of death: a piece of wood and a concrete hollow block which the killers had used to
bludgeon him to death. The eye-witness, Edna Edwina Reyes, reported the happening to the policemen
and pinpointed her neighbor, Gerente, as one of the killers. Under those circumstances, since the
policemen had personal knowledge of the violent death of Blace and of facts indicating that Gerente and
two others had killed him, they could lawfully arrest Gerente without a warrant. If they had postponed his
arrest until they could obtain a warrant, he would have fled the law as his two companions did.
SEARCH and SEIZURE
The search conducted on Gerente's person was likewise lawful because it was made as an incident
to a valid arrest. This is in accordance with Section 12, Rule 126 of the Revised Rules of Court which
provides: A person lawfully arrested may be searched for dangerous weapons or anything which may be
used as proof of the commission of an offense, without a search warrant.
The frisk and search of appellant's person upon his arrest was a permissible precautionary measure of
arresting officers to protect themselves, for the person who is about to be arrested may be armed and
might attack them unless he is first
CONSPIRACY
When there is a conspiracy to commit a crime, the act of one conspirator is the act of all. The
conspiracy was proven by the eyewitness-testimony of Edna Edwina Reyes, that she overheard the
appellant and his companions conspire to kill Blace, that acting in concert, they attacked their victim
with a piece of wood and a hollow block and caused his death. "When there is no evidence indicating that
the principal witness for the prosecution was moved by improper motive, the presumption is that he was
not so moved and his testimony is entitled to full faith and credit".

G.R. No. 121234, August 23, 1995


Hubert Webb, petitioner
vs
Hon. Raul de leon
Ponente:Puno

Facts:
June 1994, NBI filed with DOJ a letter-complaint charging petitioner Hubert Webb, and other
persons with the crime of rape with homicide. Forthwith, DOJ formed a panel of prosecutors headed by
assistant chief state prosecutor to conduct the PI of the charged with the rape and killing of the Visconti in
their home in Paranaque.
During the PI, NBI presented: (1) sworn statement of their principal witness Alfaro who allegedly
saw the commission of the crime. (2) sworn statements of two former housemaids of the Webb (3) sworn
statement of Cristobal who allegedly that he was a passenger of United Airlines bound for New York and
expressed doubt on whether Webb was his co-passenger in the trip (4) sworn statement of Barrier, former
live-in partner of Belong, who narrated the manner of how Belong investigated and tried to cover up the
crime (5) sworn statements of two Visconti maids, security guard and engineer. (6) Autopsy reports of the
victims showing the number of stab wounds and genital examination of the victims.
Before submitting his counter-affidavit, Webb filed with DOJ panel a motion for production and
examination of evidence and documents for the NBI to produce some documents like (1) certification of
US FBI on the admission to and stay of Hubert Webb in the US from March 9, 1991 to October 22, 1992
(2) Laboratory report of the medico legal officer (3) sworn statement of Belong (4) photographs of the
fingerprints lifted from the Visconti residence taken during investigation (5) investigation of NBI.
The motion was granted by DOJ and the NBI submitted the photocopies of the requested
documents. Then Webb filed a civil case in RTC of Makati for the purpose of obtaining the original of said
sworn statement and has succeeded to obtain the original copy. This was submitted to the DOJ together
with his other evidence. But Webb failed to obtain the document from US FBI.
During PI, Webb denied the crime as he went to the US and was not in the Philippines when the
crime happened. This alibi was supported by the other persons accused and the documentary evidence of
Webb's purchase of bicycle in the US, the driver's license State of California issued for him and the letter
of the legal attach of the US embassy confirming his arrival at San Francisco, California on March 9,
1991.
The other respondents submitted their sworn statement as well.
On August 1995, DOJ Panel issued a resolution finding probable cause to hold the respondents
for trial and recommending that an information for rape with homicide be filed against petitioners and
their co-respondents, which was complied on the same date with the RTC of Paranaque. The case was
raffled to branch 258 with Judge Cano, however it was Judge de Leon, pairing judge of Judge Cano who
issued the warrant of arrest against the accused. But later, Judge Cano voluntarily inhibited himself from
the case to avoid any suspicion considering that he was with NBI before his appointment to the bench.
The case was re-raffled again to branch 274 with Judge Tolentino who issued new warrants of arrest
against the accused, then Webb with other accused voluntarily surrendered to the police.
In their petitions, the petitioners contend: (1) Judge de Leon and Tolentino gravely abused their
discretion when they failed to conduct PI before issuing warrants of arrest (2) DOJ panel gravely abused

its discretion in holding that there is probable cause to charge them with the crime of rape and homicide
(3) DOJ denied them their constitutional right to due process during the PI (40) DOJ panel unlawfully
intruded into judicial prerogative when it failed to charge Jessica Alfaro in the information as an accused.

ISSUES:
1. Whether or not the DOJ Panel
likewisegravely abused its discretion in holdingt h a t t h e r e i s p r o b a b l e c a u s e t o c h a r g e
them with the crime of rape and homicide.
2. Whether or not respondent Judges deLeon and Tolentino gravely abused their
discretion when they failed to conduct a preliminary examination before issuingwarrants of
arrest against them.
3. Whether or not the DOJ Panel denied them their constitutional right to due process
during their preliminary investigation.
4. Whether or not the DOJ Panel unlawfully intrude into judicial prerogative when it
failed to charge Jessica Alfaro in the information as an accused.
Ruling:
Petition without merit. (1) 24 The terms are legally synonymous and their reference is not to a
person with training in the law such as a prosecutor or a judge but to the average man on the street. 25 It
ought to be emphasized that in determining probable cause, the average man weighs facts and
circumstances without resorting to the calibrations of our technical rules of evidence of which his
knowledge is nil. Rather, he relies on the calculus of common sense of which all reasonable men have an
abundance. Misdescription of Alfaro did not erode the credibility of Alfaro. (2) The voluminous number of
exhibits submitted by respondent Webb to support his defense of denial and alibi notwithstanding, the
panel, after a careful and thorough evaluation of the records, believes that they cannot outweigh the
evidence submitted by the complainant. Alibi cannot prevail over the positive identification made by a
prosecution witness. Verily, alibi deserves scant consideration in the face of positive identification
especially so where the claim of alibi is supported mainly by friends and relatives. The receipts of the
bicycle purchase and the driver's license were considered weak also compared to the affirmative
testimonies of the witnesses affirming that Webb is in the country. (3) In arrest cases there must be
probable cause that a crime has been committed and that the person to be arrested committed it, which of
course can exist without any showing that evidence of the crime will be found at premises under that
person's control. With respect to warrants of arrest, section 6 of Rule 112 simply provides that "upon filing
of an information, the Regional Trial Court may issue a warrant for the arrest of the accused. That before
issuing warrants of arrest, judges merely determine personally the probability, not the certainty of guilt of
an accused. In doing so, judges do not conduct a de novo hearing to determine the existence of probable

cause. They just personally review the initial determination of the prosecutor finding a probable cause to
see if it is supported by substantial evidence. (4) We reject these contentions. The records will show that
the DOJ Panel did not conduct the preliminary investigation with indecent haste. Petitioners were given
fair opportunity to prove lack of probable cause against them. Petitioners cannot also assail as premature
the filing of the Information in court against them for rape with homicide on the ground that they still
have the right to appeal the adverse resolution of the DOJ Panel to the Secretary of Justice. The filing of
said Information is in accord with Department of Justice Order No. 223, series of 1993, dated June 25,
1993. We quote its pertinent sections, viz.:
Sec. 4. Non-Appealable Cases; Exceptions. No appeal may be taken from a resolution of the Chief State
Prosecutor/Regional State Prosecutor/Provincial or City Prosecutor finding probable cause except upon
showing of manifest error or grave abuse of discretion. Notwithstanding the showing of manifest error or
grave abuse of discretion, no appeal shall be entertained where the appellant had already been arraigned.
If the appellant is arraigned during the pendency of the appeal, said appeal shall be dismissed motu
propio by the Secretary of Justice.
An appeal/motion for reinvestigation from a resolution finding probable cause, however, shall not
hold the filing of the information in court.
Sec. 2. When to appeal. The appeal must be filed within a period of fifteen (15) days from
receipt of the questioned resolution by the party or his counsel. The period shall be interrupted only by
the filing of a motion for reconsideration within ten (10) days from receipt of the resolution and shall
continue to run from the time the resolution denying the motion shall have been received by the moving
or his counsel. (Emphasis supplied)
Without doubt then, the said DOJ Order No. 223 allows the filing of an Information in court after the
consummation of the preliminary investigation even if the accused can still exercise the right to seek a
review of the prosecutor's recommendation with the Secretary of Justice.
Next, petitioners fault the DOJ Panel for not including Alfaro in the Information considering her
alleged conspiratorial participation in the crime of rape with homicide. The non-inclusion of Alfaro is
anchored on Republic Act
No. 6981, entitled "An Act Providing For A Witness Protection, Security And Benefit Program
And For Other Purposes" enacted on April 24, 1991. . In truth, the prosecution of crimes appertains to the
executive department of government whose principal power and responsibility is to see that our laws are
faithfully executed. A necessary component of this power to execute our laws is the right to prosecute their
violators. The right to prosecute vests the prosecutor with a wide range of discretion the discretion of
whether, what and whom to charge, the exercise of which depends on a smorgasbord of factors which are
best appreciated by prosecutors. We thus hold that it is not constitutionally impermissible for Congress to
enact R.A. No. 6981 vesting in the Department of Justice the power to determine who can qualify as a
witness in the program and who shall be granted immunity from prosecution. (5) petitioners charge the
NBI with violating their right to discovery proceedings during their preliminary investigation by
suppressing the April 28, 1995 original copy of the sworn statement of Alfaro and the FBI Report.41

Sections 10 and 11 of Rule 117 do provide an accused the right to move for a bill of particulars and for
production or inspection of material evidence in possession of the prosecution. 42 But these provisions
apply after the filing of the Complaint or Information in court and the rights are accorded to the accused
to assist them to make an intelligent plea at arraignment and to prepare for trial. We hold that the finding
of a probable cause by itself subjects the suspect's life, liberty and property to real risk of loss or
diminution. In the case at bar, the risk to the liberty of petitioners cannot be understated for they are
charged with the crime of rape with homicide, a non-bailable offense when the evidence of guilt is strong.

Raro
vs
Sandiganbayan,
G.R. No. 108431, July 14, 2000
Facts:
Petitioner Oscar G. Raro, a lawyer, was the Corporate Secretary of the Philippine Charity
Sweepstakes Office (PCSO). As such, petitioner was the Acting Manager of the Special Projects
Department that was in charge of the experimental Small Town Lottery (STL), which under PCSO
Resolution No. 118, dated April 1987, was to be operated in certain areas of the country. On July 30, 1987,
the PCSO, through Atty. Reynaldo E. Ilagan of the Special Projects Department, authorized Elmec Trading
and Management Corporation (ELMEC) to operate the STL in the province of Camarines Norte. ELMEC
in turn employed Luis (Bing) F. Abao, a resident of Daet, Camarines Norte, as Provincial Manager of
the experimental STL in said province.[1] Abao allegedly invested P100,000.00 in the STL operation in
that province.
In a complaint that he filed with the Tanodbayan in Manila on May 20, 1988, Abao alleged that
petitioner, in his capacity as PCSO Corporate Secretary, personally and directly intervened in the
operation of said lottery to his financial benefit and advantage .
The complaint against petitioner for violation of the Anti-Graft and Corrupt Practices Act was
referred by the Deputy Ombudsman to the NBI for investigation. The NBI recommended the prosecution
of the petitioners. However, the petitioners argue that the four-year delay in the completion of the
preliminary investigation violated right to speedy disposition of cases.
Issue:

Whether or not the Sandiganbayan gravely abused its discretion in denying a motion to quash an
information on the ground that the preliminary investigation allegedly violated the right of the accused to
due process of law.
HELD:
It took the NBI 2 years to complete its report. The resolution recommending the filing of the case
against petitioner has to be reviewed. The length of time it took before the conclusion of the preliminary
investigation may only be attributed to the adherence of the Ombudsman and NBI to the rudiments of fair
play.
Ruling:
Finally, there is no ground to give credence to petitioners claim that the complainant should be
charged as a briber on account of his admission that he gave petitioner some sum of money; or that
evidence presented during the preliminary investigation, specifically the affidavits of witnesses, were
hearsay and inadmissible. As we stated earlier, this Court cannot supplant the Ombudsmans discretion in
the determination of what crime to charge an accused.
All

told,

this

Court

finds

no

reason

to

reverse

the

assailed

Resolutions

of

the

Sandiganbayan. Petitioners insinuation that he was subjected to the proceedings before the Ombudsman
and

the

Sandiganbayan

for

politically

motivated

reasons,

has

not

been

established

with sufficient evidence. In the absence of any imputation that public respondents were impelled by illmotive in filing the case against him, it is presumed that there is no such motive and that public
respondents merely filed the case to correct a public wrong.[69]
WHEREFORE, the instant petition for certiorari and prohibition is DISMISSED for lack of merit.
The assailed Resolutions of the Sandiganbayan are hereby AFFIRMED. The Sandiganbayan is
DIRECTED to proceed with deliberate dispatch in the disposition of Criminal Case No. 17800.
G.R. No. 101837 February 11, 1992
Rolito Go y Tambunting, petitioner
vs.
The Court of Appeals, The Hon. Benjamin V. Pelayo, Presiding Judge Branch 168 Regional
Trial Court, NCJR Pasig, M.M., and People of the Philippines, respondents.
Facts:
On 8 July 1991, petitioner presented himself before the San Juan Police Station, accompanied by
two (2) lawyers. The police forthwith detained him. An eyewitness of the incident was able to take down
petitioners plate number and reported the same to the police. Another eyewitness who was at the police
station at that time identified petitioner as the gunman who shoot Eldon Maguan on July 2, 1991.
According to the police reports and of the eyewitnesses,petitioner, while traveling in the wrong direction

on a one-way street, almost had a collision with another vehicle. Petitioner thereafter got out of his car,
shot the driver of the other vehicle, and drove off.
First Assistant Provincial Prosecutor Dennis Villa Ignacio informed petitioner, in the presence of
his lawyers, that he could avail himself of his right to preliminary investigation but that he must first sign
a waiver of the provisions of Article 125 of the Revised Penal Code. Petitioner refused to execute any such
waiver.
Issues:
The principal issues at stake are whether or not the arrest by the San Juan Police with respect of
the petitioner was unlawful and whether or not petitioner effectively waived his right to preliminary
investigation.
Held:
Both the petitioner and the prosecutor erred on relying on Umil v. Ramos, wherein the Court
upheld the warrantless arrest as valid effected 1 to 14 days from actual commission of the offenses, which
however constituted continuing crimes, i.e. subversion, membership in an outlawed organization, etc. In
the instant case, the offense for which petitioner was arrested was obviously commenced and completed
at one definite location in time and space and not a continuing crime.
Furthermore, the warrantees "arrest" or detention of petitioner in the instant case does not fall
within the terms of Section 5 of Rule 113 of the 1985 Rules on Criminal Procedure. Petitioner's "arrest"
took place six (6) days after the shooting of Maguan. The arresting officers obviously were not present at
the time of the commission and none of the arresting officers had any personal knowledge of facts thereof.
Moreover, petitioner was not arrested at all as he walked into San Juan Police Station, he neither
expressed surrender nor any statement that he was or was not guilty of a crime. There was substantive
error, for petitioner was entitled to a preliminary investigation and that right should have been accorded
him without any conditions. Since petitioner had not been arrested, with or without a warrant, he was
entitled to be released forthwith subject only to his appearing at the preliminary investigation.

G.R. No. 119246. January 30, 1998


THE PEOPLE OF THE PHILIPPINES, plaintiff-appellee,
vs.
ANTONIO CORREA y CAYTON @ BOYET, RITO GUNIDA y SESANTE @ DODONG, and
LEONARDO DULAY y SANTOS @ BOY KUBA accused-appellants
Facts:
On or about June 18, 1994, in the City of Manila, Philippines, the accused Antonio Correa, Rito
Gunida and Leonardo Dulay conspired together, not being authorized by law to possess, sell, deliver, and
transport eight (8) bundles of dried flowering tops of MARIJUANA, a prohibited drug, wrapped in pieces

of papers and plastic tapes weighing 16.1789 kilograms. On 12 July 1994, an Information was filed with
the Regional Trial Court of Manila (Branch 35), docketed as Criminal Case No. 94-137528, indicting
appellants Antonio Correa y Cayton @ Boyet, Rito Gunida y Sesante @ Dodong, and Leonardo Dulay y
Santos @ Boy Kuba for having violated Section 4, Article II of Republic Act No. 6425, as amended. The
defense, however, contends that the 3 accused were arrested without warrant in Camarin D, Caloocan
City, enroute to Dulays house to get the things of his child allegedly rushed previously to the Metropolitan
Hospital, for an alleged charge of trafficking on 'shabu,' and were brought to the WPDC headquarters at
U.N. Avenue, where they were detained. After trial, on March 3, 1995, the lower court found the
appellants guilty beyond reasonable doubt and was sentenced to death to be executed by the means
provided by law, and to pay a fine of P 10,000,000.00, plus the costs.
Issue:
Whether the accused are precluded from assailing the warrantless search and seizure, due to waiver
on their part.
Held:
Antonio Correa y Cayton @ "Boyet," Rito Gunida y Sesante @ "Dodong," and Leonardo Dulay y
Santos @ "Boy Kuba" are precluded from assailing the warrantless search and seizure when they
voluntarily submitted to it as shown by their actuation during the search and seizure. They never
protested when the police officer opened the tin can loaded in their vehicle, nor when he opened one of
the bundles, nor when they, together with their cargo of drugs and their vehicle, were brought to the
police station for investigation and subsequent prosecution. When one voluntarily submits to a search or
consents to have it made on his person or premises, he is precluded from later complaining thereof The
right to be secure from unreasonable search may, like every right, be waived and such waiver may be
made either expressly or impliedly. Further, they effectively waived their constitutional right against the
search and seizure by their voluntary submission to the jurisdiction of the trial court, when they entered a
plea of not guilty upon arraignment and by participating in the trial. While the conviction of the
appellants of the crime charged is proper, we find, however, that the penalty of death imposed by the trial
court is not in accordance with the law. There being no aggravating or mitigating circumstance which
attended the commission of the offense in this case, and considering that the quantity of the subject
prohibited drug exceeded 750 grams, the proper penalty that should be imposed on each of the appellants
is reclusion perpetua and a fine of Ten Million Pesos.

G.R. No. 143944July 11, 2002


The People of the Philippines, plaintiff-appellee,
vs.
Basher Bongcarawan y Macarambon, accused-appellant.
Facts:
On December 27, 1999, the Regional Trial Court of Iligan City rendered judgment finding the
accused Basher Bongcarawan y Macarambon guilty beyond reasonable doubt for violating Section VI,

Article III of Republic Act 6425 as amended, otherwise known as the Dangerous Drugs Act of 1972 as
amended by RA 7659.
The antecedent facts of his conviction showed that on March 11, 1999, the accused boarded M/V
Super Ferry 5, sailing from Manila to Iligan City. At about 3:00 a.m. on March 13, 1999, the vessel was
about to dock at the port of Iligan City when its security officer, Mark Diesmo, received a complaint from
passenger Lorena Canoy about a missing jewelry. Diesmo and four (4) other members of the vessel
security force accompanied Canoy to search for the suspect whom they later found at the economy
section. The suspect was identified as the accused, Basher Bongcarawan. The accused was informed of the
complaint and was invited to go back to cabin no. 106. With his consent, he was bodily searched, but no
jewelry was found. He was then escorted by two (2) security agents back to the economy section to get his
baggage and took a Samsonite suitcase. When requested by the security, the accused opened the suitcase,
revealing a brown bag and small plastic packs containing white crystalline substance. Suspecting the
substance to be "shabu," the security personnel immediately reported the matter to the ship captain and
took pictures of the accused beside the suitcase and its contents.
The accused testified that the suitcase was not his but was owned by Alex Macapudi who
requested him to give it to Macapudis brother in the Iligan port. On appeal, the accused contends that the
Samsonite suitcase was forcibly opened and searched without his consent, and hence, in violation of his
constitutional right against unreasonable search and seizure. Any evidence acquired pursuant to such
unlawful search and seizure, he claims, is inadmissible in evidence against him.
Issue:
A question was raised whether the contention of the accused-appelant on unlawful search and
seizure valid.
Held:
The Supreme Court held the contention devoid and without merit. The right against unreasonable
search and seizure is a fundamental right protected by the Constitution. Whenever this right is challenged,
an individual may choose between invoking the constitutional protection or waiving his right to the search
and seizure. It should be stressed, however, that protection is against transgression committed by the
government or its agent. The search and seizure performed by the vessel security personnel should be
considered as one conducted by the police authorities tasked to maintain peace and order.
In a prosecution for illegal possession of dangerous drugs, the following facts must be proven
beyond reasonable doubt, : (1) that the accused is in possession of the object identified as a prohibited or a
regulated drug; (2) that such possession is not authorized by law; and (3) that the accused freely and
consciously possessed the said drug. The first two elements were sufficiently proven in this case, and were
in fact undisputed. In the case at bar, the third fact was missing. It has been ruled, however, that
possession of dangerous drugs constitutes prima facie evidence of knowledge or animus

possidendi sufficient to convict an accused in the absence of a satisfactory explanation of such


possession. Hence, the burden of evidence is shifted to the accused to explain the absence of knowledge
or animus possidendi.
The decision of the Regional Trial Court of Iligan City, Branch 06, in Criminal Case No. 06-7542,
is affirmed.
People
vs.
Albofera,
FACTS:
Sometime in June or July 1980, accused Albofera and 3 others killed Teodoro Carancio a forester.
Rodrigo Esma was at the house of one of the accused but did not participate in the killing.
The matter was later brought to the attention of the authorities by a certain Sisneros and accused Albofera
was arrested. The accused Lawi-an was subsequently arrested.
Albofera executed an extra-judicial confession before the Municipal Circuit Judge. He stated
therein that he was forced to join the NPA movement for fear of his life; that said group had ordered the
arrest of the victim, Carancio, and that the group sentenced him (the victim) to die by stabbing.
Esma testified against the accused during the trial. While in prison, accused Albofera sent a letter to
Esma. Said letter was thereafter introduced as evidence by prosecution. In his letter, accused Albofera was
asking Esma to change his declaration in his Affidavit and testify in his favor instead.
Later the accused were convicted of murder.
ISSUE:
Whether the Alboferas letter to Esma should be excluded as evidence in light of alleged unwarranted
intrusion or invasion of the accuseds privacy?
HELD:
No. The production of that letter by the prosecution was not the result of an unlawful search and
seizure nor was it through unwarranted intrusion or invasion into Alboferas privacy. Albofera admitted
having sent the letter and it was its recipient, Rodrigo Esma himself, who produced and identified the
same in the course of his testimony in Court. Besides, there is nothing really self-incriminatory in the
letter. Albofera mainly pleaded that Esma change his declaration in his Affidavit and testify in his
(Alboferas) favor. Furthermore, nothing Alboferas tated in his letter is being taken against him in arriving
at a determination of his culpability.

PEOPLE OF THE PHILIPPINES


vs.
ANDRE MARTI
Facts:
On August 14, 1987, the appellant and his common-law wife, Shirley Reyes went to Manila
Packaging and Export Forwarders to send packages to Zurich, Switzerland. It was received by Anita Reyes
and ask if she could inspect the packages. Shirley refused and eventually convinced Anita to seal the

package making it ready for shipment. Before being sent out for delivery, Job Reyes, husband of Anita and
proprietor of the courier company, conducted an inspection of the package as part of standard operating
procedures. Upon opening the package, he noticed a suspicious odor which made him took sample of the
substance he found inside. He reported this to the NBI and invited agents to his office to inspect the
package. In the presence of the NBI agents, Job Reyes opened the suspicious package and found driedmarijuana leaves inside. A case was filed against Andre Marti in violation of R.A. 6425 and was found
guilty by the court a quo. Andre filed an appeal in the Supreme Court claiming that his constitutional right
of privacy was violated and that the evidence acquired from his package was inadmissible as evidence
against him.
Issue:
Can the Constitutional Right of Privacy be enforced against private individuals?
Ruling:
The Supreme Court held based on the speech of Commissioner Bernas that the Bill of Rights
governs the relationship between the individual and the state.
The constitutional proscription against unlawful searches and seizures therefore applies as a
restraint directed only against the government and its agencies tasked with the enforcement of the law. It
is not meant to be invoked against acts of private individuals. It will be recalled that Mr Job Reyes was the
one who opened the box in the presence of the NBI agents in his place of business. The mere presence of
the NBI agents did not convert the reasonable search effected by Mr. Reyes into a warrantless search and
siezure proscribed by the constitution. Merely to observe and look at that which is in plain sight is not a
search.
The judgement of conviction finding appeallant guilty beyond reasonable doubt of the crime
charged was AFFIRMED.

Tan
vs
Del Rosario
G.R. No. 109289, October 3, 1994
FACTS:
These two consolidated special civil actions for prohibition challenge, in G.R. No. 109289, the
constitutionality of Republic Act No. 7496, also commonly known as the Simplified Net Income Taxationn
Scheme (SNIT), amending certain provisions of the National Internal Revenue Regulations No. 293,
promulgated by public respondents pursuant to said law.
Petitioner intimates that Republic Act No. 7496 desecrates the constitutional requirement that taxation
shall be uniform and equitable in that the law would now attempt to tax single proprietorships and
professionals differently from the manner it imposes the tax on corporations and partnerships. Petitioners
claim to be taxpayers adversely affected by the continued implementation of the amendatory legislation.

ISSUES:
1. Is Republic Act No. 7496 a misnomer or, at least, deficient for being merely entitled, Simplified Net
Income Taxation Scheme for the Self-Employed and Professionals Engaged in the Practice of their
Profession (Petition in G.R. No. 109289)
2. Does Republic Act No. 7496 violate the Constitution for imposing taxes that are not uniform and
equitable.
3. Did the Secretary of Finance and the BIR Commissioner exceed their rule-making authority in applying
SNIT to general professional partnerships?
HELD:
The Petition is dismissed. Uniformity of taxation, like the kindred concept of equal protection,
merely requires that all subjects or objects of taxation, similarly situated, are to be treated alike both in
privileges and liabilities (Juan Luna Subdivision vs. Sarmiento, 91 Phil. 371). Uniformity does not forfend
classification as long as: (1) the standards that are used therefor are substantial and not arbitrary, (2) the
categorization is germane to achieve the legislative purpose, (3) the law applies, all things being equal, to
both present and future conditions, and (4) the classification applies equally well to all those belonging to
the same class (Pepsi Cola vs. City of Butuan, 24 SCRA 3; Basco vs. PAGCOR, 197 SCRA 771).
What may instead be perceived to be apparent from the amendatory law is the legislative intent to
increasingly shift the income tax system towards the schedular approach in the income taxation of
individual taxpayers and to maintain, by and large, the present global treatment on taxable corporations.
We certainly do not view this classification to be arbitrary and inappropriate.
Having arrived at this conclusion, the plea of petitioner to have the law declared unconstitutional
for being violative of due process must perforce fail. The due process clause may correctly be invoked only
when there is a clear contravention of inherent or constitutional limitations in the exercise of the tax
power.
Tiu
vs
Ca G.R. No. 127410. January 20, 1999
J. Panganiban
Facts:
Congress, with the approval of the President, passed into law RA 7227 entitled "An Act
Accelerating the Conversion of MilitaryReservations Into Other Productive Uses, Creating the Bases
Conversion and Development Authority for this Purpose, Providing Funds Therefor and for Other
Purposes." Section 12 thereof created the Subic Special Economic Zone and granted there to special
privileges. President Ramos issued Executive Order No. 97, clarifying the application of the tax and duty
incentives. The President issued Executive Order No. 97-A, specifying the area within which the tax-andduty-free privilege was operative. The petitioners challenged before this Court the constitutionality of EO
97-A for allegedly being violative of their right to equal protection ofthe laws. This Court referred the
matter to the Court of Appeals.Proclamation No. 532 was issued by President Ramos. It delineated the
exact metes and bounds of the Subic Special Economic and Free Port Zone, pursuant to Section 12 of RA
7227. Respondent Court held that "there is no substantial difference between the provisions of EO 97-A
and Section 12 of RA 7227. In both, the 'Secured Area' is precise and well-defined as '. . . the lands

occupied by the Subic Naval Base and its contiguous extensions as embraced, covered and defined by the
1947 Military Bases Agreement between the Philippines and the United States of America, as
amended . . .'"
Issue:
Whether or not Executive Order No. 97-A violates the equal protection clause of the Constitution
Held:
No. The Court found real and substantive distinctions between the circumstances obtaining inside
and those outside the Subic Naval Base, thereby justifying a valid and reasonable classification. The
fundamental right of equal protection of the lawsis not absolute, but is subject to reasonable classification.
If the groupings are characterized by substantial distinctions that make real differences, one class may be
treated and regulated differentlyfrom another. The classification must also be germane to the purpose of
the law and must apply to all those belonging to the same class. Classification, to be valid, must (1) rest on
substantial distinctions, (2) be germane to the purpose of the law, (3) not be limited to existing conditions
only, and (4) apply equally to all members of the same class. The Supreme Court believed it was
reasonable for the President to have delimited the application of some incentives to the confines of the
former Subic military base. It is this specific area which the government intends to transform and develop
from its status quo ante as an abandoned naval facility into a self-sustaining industrial and commercial
zone, particularly for big foreign and local investors to use as operational bases for their businesses and
industries.

G.R. No. 56515 April 3, 1981


United Democratic Opposition (UNIDO), petitioner,
vs.
Commission on Elections (COMELEC), respondent.
Facts:
At a time when the country was already under martial law, amendments to the 1973 Constitution
were proposed by the Batasang Pambansa. The amendments to the constitution were to be placed in a
plebiscite for the peoples approval. The Commission on elections, pursuant to the powers vested in it by
the Constitution, the 1978 Election Code and pertinent enactments of the Batasang Pambansa, issued
three (3) Resolutions 1467-1469 providing for equal opportunity on public discussions and debates,
equal time on the use of broadcast media, and equal space on the use of print media.
On March 12, 1981, President Marcos advanced his campaign for the YES votes on the
constitutional amendments in his nationwide Pulong-Pulong sa Pangulo radio-television program
carried live from 9:30 to 11:30 PM via radio and television. Pursuant to the resolutions promulgated by
the COMELEC, petitioner UNIDO demand exactly the same opportunity, the same prime tune, the same
number of TV and radio stations all over the country at the earliest possible date to campaign for NO
votes in the forthcoming plebiscite.

After due and careful deliberation, the Commission held and ruled that the demand of the UNIDO
cannot be granted and thereby denied. UNIDO filed a Motion for Reconsideration to the Commission
appealing that such denial is a basic ground for contradiction to the Constitution and the Law, and
moreover, violate the basic principles of equality, good faith and fair play.
Issue:
The question was raised whether or not UNIDO was denied equal protection by virtue of the
COMELECs denial of their request.
Held:
The Supreme Court held that UNIDO was not denied due process nor were they not afforded
equal protection.
It is considered view of the Commission that when President Marcos conducted his pulongpulong or consultation with the people on March 12, 1981, he did so in his capacity as President/Prime
Minister of the Philippines and not as the head of any political party. The President/Prime Minister is
responsible for the program of government and the guidelines of policy. It cannot be denied that seeking
constitutional changes constitutes a program of government imbued with the nature of highest
importance to enlighten the people on its sense and significance.
The UNIDO or any of its leaders does not have the same constitutional prerogatives vested in the
President/Prime Minister as above discussed. As such, it has no right to 'demand' equal coverage by
media accorded President Marcos. The UNIDO, however, is free to enter into appropriate contracts with
the TV or radio stations concerned.

G.R. No. 83988 September 29, 1989


Ricardo C. Valmonte and Union of Lawyers and Advocates for Peoples Rights (ULAP),
petitioners,
vs.
Gen. Renato De Villa and National Capital Region District Command (NCRDC),
respondents.
Facts:
On 20 January 1987, the National Capital Region District Command (NCRDC) was activated
pursuant to Letter of Instruction 02/87 of the Philippine General Headquarters, AFP, with the mission of
conducting security operations within its area of responsibility and peripheral areas, for the purpose of
establishing an effective territorial defense, maintaining peace and order, and providing an atmosphere
conducive to the social, economic and political development of the National Capital Region. As part of its
duty to maintain peace and order, the NCRDC installed checkpoints in various parts of Valenzuela, Metro
Manila.
Petitioners aver that, because of the installation of said checkpoints, the residents of Valenzuela
are worried of being harassed and of their safety being placed at the arbitrary, capricious and whimsical
disposition of the military manning the checkpoints, considering that their cars and vehicles are being
subjected to regular searches and check-ups, especially at night or at dawn, without the benefit of a search

warrant and/or court order. Their alleged fear for their safety increased when, at dawn of 9 July 1988,
Benjamin Parpon, a supply officer of the Municipality of Valenzuela, Bulacan, was gunned down allegedly
in cold blood by the members of the NCRDC manning the checkpoint along McArthur Highway at
Malinta, Valenzuela, for ignoring and/or refusing to submit himself to the checkpoint and for continuing
to speed off inspire of warning shots fired in the air. Petitioner Valmonte also claims that, on several
occasions, he had gone thru these checkpoints where he was stopped and his car subjected to
search/check-up without a court order or search warrant. Instances have occurred where a citizen, while
not killed, had been harassed. Petitioners contended that the checkpoints gave the respondents blanket
authority to make searches and seizures without search warrant or court order in violation of the
Constitution. Finally, on 17 July 1988, military and police checkpoints in Metro Manila were temporarily
lifted and a review and refinement of the rules in the conduct of the police and military manning the
checkpoints was ordered by the National Capital Regional Command Chief and the Metropolitan Police
Director.
Issue:
The question was raised as to whether or not checkpoints violate the right of the people against
unreasonable search and seizures.
Held:
The Supreme Court held to dismiss the petition. True, the manning of checkpoints by the military
is susceptible of abuse by the men in uniform, in the same manner that all governmental power is
susceptible of abuse. But, at the cost of occasional inconvenience, discomfort and even irritation to the
citizen, the checkpoints during these abnormal times, when conducted within reasonable limits, are part
of the price we pay for an orderly society and a peaceful community. Between the inherent right of the
state to protect its existence and promote public welfareand and individuals right against a warrantless
search which is however reasonably conducted, the former should prevail.

G.R. No. L-30026,


January 30, 1971
MARIO GUMABON, BLAS BAGOLBAGOL, GAUDENCIO AGAPITO, EPIFANIO PADUA and
PATERNO PALMARES, petitioners,
vs.
THE DIRECTOR OF THE BUREAU OF PRISONS, respondent.
FACTS:
Gumabon, after pleading guilty, was sentenced on May 5, 1953 to reclusion perpetua for the
complex crime of rebellion with multiple murder, robbery, arson and kidnapping (along with Agapito,
Palmares and Padua). The decision for the first two petitioners was rendered on March 8, 1954 and the
third on Dec. 5, 1955. The last petitioner Bagolbagol was penalized with reclusion perpetua on Jan. 12,
1954. Each of the petitioners have been imprisoned for more than 13 years by virtue of their convictions.
They now invoke the doctrine laid down in People v. Hernandez which negated such complex
crime, a ruling which was not handed down until after their convictions have become final. In People v.
Hernandez, the SC ruled that the information against the accused for rebellion complexed with murder,
arson and robbery was not warranted under Art. 134 of the RPC, there being no such complex offense.
This ruling was not handed down until after their convictions have become final. Since Hernandez served

more than the maximum penalty that could have been served against him, he is entitled to freedom, and
thus, his continued detention is illegal.
ISSUE:
Whether or not Art. 22 of the RPC which gives a penal judgment a retroactive effect is applicable in this
case.
RULING:
Yes. Judicial decisions favourable to the accused must be applied retroactively. Petitioners relied
on Art. 22 of the RPC, which states the penal laws shall have a retroactive effect insofar as they favour the
accused who is not a habitual criminal. The Civil Code also provides that judicial decisions applying or
interpreting the Constitution forms part of our legal system. Petitioners even raised their constitutional
right to equal protection, given that Hernandez et al., has been convicted for the same offense as they
have, though their sentences were lighter. Habeas corpus is the only means of benefiting the accused by
the retroactive character of a favorable decision.

G.R. No. 79543.


October 16, 1996
JOSE D. FILOTEO, JR., petitioner,
vs.
SANDIGANBAYAN and THE PEOPLE OF THE PHILIPPINES, respondents.
Facts:
Petitioner Jose D. Filoteo, Jr. was a police investigator of the Western Police District in Metro
Manila, an old hand at dealing with suspected criminals. A recipient of various awards and
commendations attesting to his competence and performance as a police officer, he could not therefore

imagine that one day he would be sitting on the other side of the investigation table as the suspected
mastermind of the armed hijacking of a postal delivery van. Filoteo admitted involvement in the crime
and pointed to three other soldiers, namely ,Eddie Saguindel, Bernardo Relator and Jack Miravalles (who
turned out to be adischarged soldier), as his confederates. At 1:45 in the afternoon of May 30,
1982,petitioner executed a sworn statement in Tagalog before M/Sgt. Arsenio C. Carlos and Sgt. Romeo P.
Espero. Peitioner however sought later that his confession be inadmissible evidence, saying that the law
should favour him as an accused.
Issue:
Whether or not Article III, Section 12 of the 1987 Constitution shall be given a retroactive effect
and petitioners extrajudicial confession be held as inadmissible evidence.
RULING:
No, since what he did was not a penal offense. Under the penal law, a person guilty of felony who
is not a habitual criminal may be given favour by the law.

[G.R. No. 148825. December 27, 2002]


PEOPLE OF THE PHILIPPINES, appellee,
vs.
SUSAN CANTON, appellant.
FACTS:
On February 12, 1998, at about 1:30 p. m., Susan Canton was at the Ninoy Aquino International
Airport, being a departing passenger bound for Saigon, Vietnam. When the metal detector alarmed while
Susan was passing through, Mylene Cabunoc, a civilian employee of the National Action Committee on
Hijacking and Terrorism (NACHT) and the frisker on duty at that time, made a pat down search on the
former. Upon Frisking, Susan, Mylene felt something bulging at her abdominal area and when the latter
inserted her hand under the skirt of Susan, She noticed that the packages contained what felt like rice
granules. Mylene then reported the matter tom SPO4 Victorio de los Santos, her supervisor on duty. The
supervisor then instructed Mylene to call Customs Examiner Lorna Jalac and bring Susan to a comfort
room for a thorough physical investigation. Upon further frisking, Mylene and Lorna discovered three
packages individually wrapped and sealed in grey colored packing tape which Susan voluntarily handed to
them. Mylene turned over the packages to SPO4 De los Santos and after laboratory examination, it yielded
positive results for methamphetamine hydrochloride or shabu, a regulated drug.
SPO2 Jerome Cause, an investigator of the First Regional Aviation Office, testified that no
investigation was ever conducted on Susan. However, Susan signed a receipt of the following articles
seized from her: (1) three bags of methamphetamine hydrochloride or shabu approximately 1,100 grams;
(2) one American passport bearing Number 700389994; (3) one Continental Micronesia plane ticket with
stock control number 0414381077; and (4) two panty girdles. He said that he informed Susan of her

constitutional rights but admitted that she did not have a counsel when she signed the receipt. Yet he told
her that she had the option to sign or not to sign the receipt.
ISSUES:
1.

Whether or not the search conducted on Susan was incidental to a lawful arrest.

2.

Whether or not the scope of a search pursuant to airport security is confined only to search weapons
under Terry Search doctrine.

3.

Whether or not Susan was lawfully arrested without a warrant.

4.

Whether or not the constitutional right to counsel afforded an accused under custodial investigation was
violated.

5.

Whether or not Susans conviction and penalty on her are correct.


RULING:
The Supreme Court did not agree with the trial court and the OSG that the search and seizure
conducted in this case were incidental to a lawful arrest. In a search incidental to a lawful arrest, the law
requires that there be first a lawful arrest before a search can be made; the process cannot be
reversed. Susans arrest did not precede the search. . It was only after the strip search upon the discovery
by the police officers of the white crystalline substances inside the packages, which they believed to
be shabu, that SUSAN was arrested.
Under Section 9 of Republic Act No. 6235, the provision is clear that the search, unlike in
the Terry search, is not limited to weapons. The Terry search or the stop and frisk situation refers to a
case where a police officer approaches a person who is acting suspiciously, for purposes of investigating
possibly criminal behavior in line with the general interest of effective crime prevention and detection. To
assure himself that the person with whom he is dealing is not armed with a weapon that could
unexpectedly and fatally be used against him, he could validly conduct a carefully limited search of the
outer clothing of such person to discover weapons which might be used to assault him. In this case, after
the metal detector alarmed Susan, R.A. No. 6235 authorizes search for prohibited materials or substances.
Thus, the strip search in the ladies room was justified under the circumstance.
Warrantless search and seizure were legal. Armed with the knowledge that Susan was
committing a crime, the airport security personnel and police authorities were duty-bound to arrest her,
under paragraph (a) of Section 5, Rule 113 of the Rules of Court.
As testified to by the lone witness for the defense, SPO2 Jerome Cause, no custodial investigation
was conducted after Susans arrest. She affixed her signature to the receipt of the articles seized from her,
but before she did so, she was told that she had the option to sign or not to sign it. In any event, her
signature to the packages was not relied upon by the prosecution to prove its case. Moreover, no
statement was taken from her during her detention and used in evidence against her. Hence, her claim of
violation of her right to counsel has no leg to stand on.

As regards the fine, courts may fix any amount within the limits established by law. For
possession of regulated drugs, the law fixes the range of the fine from P500,000 to P10 million. In view of
the net weight of methamphetamine hydrochloride found in the possession of Susan, the trial courts
imposition of fine in the amount of P1 million is well within the range prescribed by law.
Susan Canton was found guilty beyong reasonable doubt of the violation of Section 16, Article III
of the Dangerous Act of 1972 ( Republic Act No. 6425) as amended and sentenced her to suffer the penalty
of reclusion perpetua and pay a fine of One Million Pesos (P1,000,000.00). The appellants passport,
plane tickets, and girdles are hereby ordered to be returned to her.

EN BANC
G.R. Nos. L-6025-26. July 18, 1956.]
THE PEOPLE OF THE PHILIPPINES, Plaintiff-Appellee,
Vs.
AMADO V. HERNANDEZ, ET AL.,Defendants-Appellants.
FACTS:
About March 15, 1945, (1) Amado V. Hernandez alias Victor alias Soliman alias Amado alias AVH
alias Victor Soliman, (2) Guillermo Capadocia alias Huan Bantiling alias Cap alias G. Capadocia, (3)
Mariano P. Balgos alias Bakal alias Tony Collantes alias Bonifacio, (4) Alfredo Saulo alias Elias alias Fred
alias A.B.S. alias A.B., (5) Andres Baisa, Jr. alias Ben alias Andy (6) Genaro de la Cruz alias Gonzalo alias
Gorio alias Arong, (7) Aquilino Bunsol alias Anong, (8) Adriano Samson alias Danoy, (9) Juan J. Cruz
alias Johnny 2, alias Jessie Wilson alias William, (10) Jacobo Espino, (11) Amado Racanday, (12) Fermin
Rodillas, and (13) Julian Lumanog alias Manue, were accused of being members of PKP Community Party
of the Philippines which wasactively engaged in an armed rebellion against the government of the Philippines. With
the party of HUKBALAHAP (Hukbo ng Bayan Laban sa mga Hapon), they committed the crime of
rebellion causing murder, pillage, looting plunder, etc., enumerated in 13 attacks on government forces or
civilians by HUKS.
The prosecution maintained that Hernandez is charged with rebellion complexed with murders,
arsons and robberies, for which the capital punishment may be imposed. The defense contends, among
other things, that rebellion cannot be complexed with murder, arson, or robbery. The lower court
sentenced Hernandez merely to life imprisonment. A petition for bail was filed by Amado Hernandez on
December 28, 1953, which was denied by a resolution of the Supreme Court dated February 2 , 1954. A
similar petition for bail was filed by Hernandez on June 26, 1954 and renewed on December 22, 1955.

ISSUE:
Whether or not Amado V. Hernandez is entitle to bail.
RULING:
The court ruled that murder, arson, and robbery are mere ingredient of the crime of rebellion as
means necessary for the perpetration of the offense. Such common offense is absorbed or inherent of
the crime of rebellion. In as much as the acts specified in Article 135 of the Revised Penal Code, one single
crime it follows that said acts offer no occasion for the application of Article 48 of the Revised Penal Code
which requires therefore the commission of at least two crimes.
The crime charged in the amended information is, therefore, simple rebellion, not the complex crime of
rebellion with multiple murder, arsons and robberies; that the maximum penalty imposable under such
charge cannot exceed 12 years of prision mayor and a fine of P20,000; and that, in conformity with the
policy of the Supreme Court in dealing with accused persons amenable to a similar punishment, said
defendant may be allowed bail.

G.R. No. 130644. October 27, 1997]


THE MINOR FRANCISCO JUAN LARRANAGA, represented in this suit by his mother
MARGARITA G. LARRANAGA, petitioner,
vs.
COURT OF APPEALS and PEOPLE OF THE PHILIPPINES, respondents.
FACTS:
On October 1, 1997, petitioner Margarita G. Larranaga filed a petition for certiorari, prohibition
and mandamus with writs of preliminary prohibitory and mandatory injunction seeking to annul the
information for kidnapping and serious illegal detention against her minor son, Francisco Juan
Larranagga alias Paco, filed in the RTC of Cebu City as well as the warrant of arrest issued as a
consequence thereof. On October 6, 1997, petitioner filed a Supplemental Petition asking for the issuance
of the writ of habeas corpus to relieve her son from his alleged illegal confinement or to grant him bail.
On September 15, 1997, the police tried to arrest the Larranaga without a warrant for the alleged
crime of kidnapping and raping Marijoy and Jacqueline Chiong in Cebu City on July 16, 1997 but his
counsel persuaded the arresting officer that he would be presented in the preliminary investigation to be
conducted in Cebu City on September 17, 1997. On September 17, 1997, Larranagas counsel attended the

preliminary investigation and made a request to the prosecutor that his client be given a preliminary
investigation and that he be granted a period of twenty days to file the defence affidavit. As well pointed of
his motion, Larranaga travelled from Cebu City to Quezon City on June 8, 1997 to pursue a Diploma at the
Centre for culinary arts in Quezon City. On July 16, 1997, he was taking examinations during the entire
day and then went to a restaurant in the evening. He stayed with friends until the next morning. On July
17, 1997, he took another examination before taking a plane back to Cebu City at 5pm. The prosecutor
denied this request, arguing that Larranaga was entitled only to an inquest investigation. On September
19, 1997, Larranagas counsel appealed to the Court of Appeals to prevent the filing of criminal
information against Larranaga. However, criminal charges had already been filed on September 17, 1997
with the Regional Trial Court of Cebu City. On September 22, 1997, counsel filed a petition with the Court
of Appeals requesting that the Regional Trial Court of Cebu City prevent Larranagas arrest. Nevertheless,
he was arrested on that day with a warrant issued by the Executive Judge of the RTC of Cebu City, the
Honorable Priscilla Agana. Another petition was filed in the Court of Appeals against his arrest and
dismissed on September 25, 1997. This decision was appealed to the Supreme Court. Despite this pending
appeal, Larranaga was brought before a judge on October 14, 1997. He did not enter a plea and the judge
thus entered a plea of not guilty to two counts of kidnapping with serious illegal detention. On October 16,
1997, the Supreme Court temporarily restrained this judge from proceeding with the case to prevent the
issues before the court from becoming moot.
ISSUES:
1. Whether or not petitioner is entitled to a regular preliminary investigation.
2. Whether or not petitioner should be released from detention pending the investigation.
3. Whether or not the arresting officer had legal authority to make warrantless arrest of the petitioner

RULING:
The Court resolves: (1) to set aside the inquest investigation of petitioner and to order the Office
of the City Prosecutor of Cebu to conduct a regular preliminary investigation of the petitioner in accord
with section 3, Rule 112; (2) to annul the order for Detention During The Pendency of the Case issued by
Executive Judge Priscilla Agana against the petitioner in Crim. Case No. CBU-45303 and 45304; (3) to
order the immediate release of petitioner pending his preliminary investigation and (4) to order the
Presiding Judge of Br. VII, RTC of Cebu City to cease and desist from proceeding with the arraignment
and trial of petitioner in Crim. Case No. CBU-45303 and 45304, pending the result of petitioners
preliminary investigation.
It is within petitioners constitutional and legal rights to demand that a regular preliminary
investigation rather than a mere inquest be conducted before resolving the issue of whether or not to file

informations against him. The petition be given due course and petitioner be accorded his right to
preliminary investigation and during the pendency thereof, petitioner be released from detention.
The arresting officer had no legal authority to make warrantless arrest. Under Sec 5. Arrest
without warrant; when lawful. A peace officer or a private person may, without a warrant, arrest a person:
(a) When in his presence, the person to be arrested has committed, is actually committing, or is
attempting to commit an offense; (b) When an offense has in fact just been committed, and he has
personal knowledge of facts indicating that the person to be arrested has committed it; and (c) When the
person to be arrested is a prisoner who has escaped from a penal establishment or place where he is
serving final judgment or temporarily confined while his case is pending, or has escaped while being
transferred from one confinement to another. In cases falling under paragraphs (a) and (b) hereof, the
person arrested without a warrant shall be forthwith delivered to the nearest police station or jail, and he
shall be proceeded against in accordance with Rule 112, Section 7. The arresting officers obviously were
not present, within the meaning of Section 5(a), at the time petitioner had allegedly committed the crime.
Moreover, none of the arresting officers had any personal knowledge of facts.

MELVIN L. ESPINO and ESTRELLITA L. IGPIT, complainants,


vs.
HON. ISMAEL L. SALUBRE, Municipal Trial Court, Tagum, Davao del Norte, respondent.

FACTS:
On July 17, 1997, Wilfreda Clamucha filed a complaint for frustrated murder before the
Municipal Trial Court of Tagum, Davao del Norte, presided by respondent Judge Ismael Salubre, against
Joemar Telleron and two (2) unknown assailants, only known as alias Dodong and John Doe, for
stabbing and mortally wounding her son, Ruel Clamucha who later died from his wounds.
On November 20, 1997, Wilfreda Clamucha executed a Supplemental Sworn Statement before
the Tagum Police Station to the effect that she was informed by the accused Joemar Telleron that the true
name of alias Dodong is Peter Erer and that of their other companion designated as John Doe is
Melvin Espino alias Bentot.
Despite the fact that respondent Judge had no more jurisdiction over the case, he conducted a
further preliminary examination and after allegedly finding the existence of probable cause, respondent
Judge issued a warrant for the arrest of Peter Erer and Melvin Espino on November 27, 1997.

On February 23, 1998, Melvin Espino, represented by his aunt, Estrellita Igpit, filed a Petition
for Habeas Corpus before Judge Bernardo V. Saludares of the Regional Trial Court , Branch 2, Tagum
City naming Judge Ismael Salubre, Police Chief Pytagoras Cervantes and the Warden of Tagum District
Jail as respondents. In said petition, it was averred that Melvin Espino was restrained of his liberty
without a valid warrant.
ISSUE:
Whether or not Judge Salubre is guilty of gross ignorance of law.
RULING:
Yes. The Court finds respondent Judge Ismael L. Salubre GUILTY of gross ignorance of the
law and is hereby FINED in the amount of Five Thousand (P5,000.00) Pesos, with a warning that a
repetition of the same will merit a more severe penalty.
To constitute gross ignorance of the law, the acts complained of must not only be contrary to
existing law and jurisprudence, but were motivated by bad faith, fraud, dishonesty and corruption. These
circumstances were not at all attendant in the case at bar.
Ignorance of the law, which everyone is bound to know, excuses no one - not even judges. They
are expected to keep abreast of our laws and the changes therein as well as with latest decisions of the
Supreme Court. A judge should be acquainted with legal norms and principles as well as with the statutes
and procedural rules. Unfamiliarity with the Rules of Court is a sign of incompetence, which goes against
Canon 3, specifically Rule 3.01, of the Code of Judicial Conduct. Having accepted the exalted position of a
judge, respondent judge owes the public and the court she sits in to be proficient in the law. She must
have the basic rules at the palm of her hand as she is expected to maintain professional competence at all
times.

G.R. Nos. 76649-51 August 19, 1988


20TH CENTURY FOX FILM CORPORATION, petitioner,
vs.
COURT OF APPEALS, EDUARDO M. BARRETO, RAUL SAGULLO and FORTUNE
LEDESMA, respondents.
FACTS:
On August 26, 1985, petitioner 20th Century Fox Film Corporation through counsel, in a lettercomplaint, sought the National Bureau of Investigation's (NBI) assistance in the conduct of searches and

seizures in connection with the latter's anti-film piracy campaign. Specifically, the letter-complaint alleged
that certain videotape outlets all over Metro Manila are engaged in the unauthorized sale and renting out
of copyrighted films in videotape form which constitute a flagrant violation of Presidential Decree No. 49
(otherwise known as the Decree on the Protection of Intellectual Property).
The NBI, in response to the letter-complaint, conducted surveillance and investigation of the
outlets pinpointed by the petitioner and subsequently filed three (3) applications for search warrants
against the video outlets owned by the private respondents. The applications were consolidated and heard
by the Regional Trial Court of Makati, Branch 132.
On September 4, 1985, the lower court issued the desired search warrants on the basis of the statements
of applicant NBIs witnesses which were taken through searching questions and answers. The NBI,
accompanied by the petitioner's agents, raided the video outlets and seized the items described in the
three warrants.
The lower court lifted the three questioned search warrants against the private respondents on
the ground that it acted on the application for the issuance of the said search warrants and granted it on
the misrepresentations of applicant NBI and its witnesses that infringement of copyright or a piracy of a
particular film have been committed.
On October 8, 1985, the lower court issued an order in lifting the three search warrants issued earlier
against the private respondents by the court.
The petitioner filed a motion for reconsideration but was denied by the lower court in its order
dated January 2, 1986.
The petitioner filed a petition for certiorari with the Court of Appeals to annul the October 8, 1985 and
January 2, 1986 orders of the lower court.
The petition was dismissed. The questioned decision and resolution of the Court of Appeals are affirmed.
ISSUES:
1. Whether or not the judge properly lift the search warrants he issued earlier.
2. Whether or not respondent court did commit a grave abuse of discretion when it issued the questioned
order.
RULING:
YES, the judge properly lifted the search warrants he issued earlier.
In the absence of probable cause that the private respondents violated P.D. 49, the copyright
infringement law, the court lifted the three questioned search warrants.
NBI agents who acted as witnesses did not have personal knowledge of the subject matter. Of the
three witnesses in the application of search warrant, only one of them, who is the petitioners counsel, had
the personal knowledge of the subject matter. The lower court declared that the testimony of the

petitioners counsel did not have much credence because the master tapes of the allegedly pirated tapes
were not shown to the court during the application.
A careful review of the record of the case shows that the respondent Court did not commit a grave
abuse of discretion when it issued the questioned orders. Grave abuse of discretion' implies such
capricious and whimsical exercise of judgment as is equivalent to lack of jurisdiction, or, in other words,
where the power is exercised in an arbitrary or despotic manner by reason of passion or personal hostility,
and it must be so patent and gross as to amount to an evasion of positive duty or to a virtual refusal to
perform the duty enjoined or to act at all in contemplation of law. But far from being despotic or arbitrary,
the assailed orders were motivated by a noble desire of rectifying an error, much so when the erroneous
findings collided with the constitutional rights of the private respondents. In fact, the petitioner did not
even contest the righteousness and legality of the questioned orders but instead concentrated on the
alleged denial of due process of law.
G.R. No. L-52245 January 22, 1980
Patricio Dumlao, Romeo B. Igot, and Alfredo Sapantan, Jr., petioners
Vs.
Commission on Elections, respondent
Facts:
Petitioner Dumlao is a former Governor of Nueva Vizcaya, who has filed his certificate of
candidacy for said position of Governor in the forthcoming elections of January 30, 1980. He specifically
questions the constitutionality of section 4 of Batas Pambansa Blg. 52 as discriminatory and contrary to
the equal protection and due process guarantees of the Constitution. Section 4 - Any retired elective
provincial, city of municipal official who has received payment of the retirement benefits to which he is
entitled under the law and who shall have been 65 years of age at the commencement of the term of office
to which he seeks to be elected, shall not be qualified to run for the same elective local office from which
he has retired. He claimed that the aforecited provision was directed insidiously against him, and that the
classification provided therein is based on "purely arbitrary grounds and, therefore, class legislation.
His colleague Igot, assailed the same law for the prohibition for candidacy of a person who was
convicted of a crime given that there was judgment for conviction and the prima facie nature of the filing
of charges for the commission of such crimes. He also questioned the accreditation of some political
parties by respondent COMELEC, as authorized by Batas Pambansa Blg. 53, on the ground that it is
contrary to section 9(1), Art. XII(C) of the Constitution, which provides that a "bona fide candidate for any
public office shall be free from any form of harassment and discrimination." Apart form this, he also
attacked the term of office and the election period. These were Sec 7 of BP 51, Sec 4; Sec 6, and Sec 1 of BP
52.
Issues:
1. Did petitioners have standing?
2. Are the statutory provisions violative of the Constitution?
Ruling:

1. No.
2. Dumlao's petition dismissed. Igot's petition partially granted.
Petition granted
Ratio:
1. Dumlao sued as a candidate while Igot sued as a taxpayer. In order to determine judicial review, three
requisites are present:
a. actual case and controversy
b. proper party
c. existence of a constitutional question
a. Dumlao has not yet been affected by the statute. No petition has yet been filed for his disqualification. It
was only a hypothetical question.
b. Did they sustain direct injury as a result of the enforcement? No one has yet been adversely affected by
the operation of the statutes.
c. They are actually without cause of action. It follows that the necessity for resolving the issue of
constitutionality is absent, and procedural regularity would require that his suit be dismissed.
However, they relaxed the procedural standard due to the public interest involved and the imminent
elections.
2. Section 4 of BP Blg. 52 is not contrary to equal protection. The constitutional guarantee ofequal
protection of the laws is subject to rational classification. If the groupings are based on reasonable and
real differentiations, one class can be treated and regulated differently from another class. For purposes of
public service, employees 65 years of age, have been validly classified differently from younger employees.
Employees attaining that age are subject to compulsory retirement, while those of younger ages are not so
compulsorily retirable. The requirement to retire government employees at 65 may or may not be a
reasonable classification. Young blood can be encouraged to come in to politics. But, in the case of a 65year old elective local official who has already retired, there is reason to disqualify him from running for
the same office, as provided for in the challenged provision. The need for new blood assumes relevance.
The tiredness of the retiree for government work is present, and what is emphatically significant is that
the retired employee has already declared himself tired an unavailable for the same government work,
but, which, by virtue of a change of mind, he would like to assume again. It is for the very reason that
inequality will neither result from the application of the challenged provision. Just as that provision does
not deny equal protection, neither does it permit such denial. In fine, it bears reiteration that the equal
protection clause does not forbid all legal classification. What is proscribes is a classification which is
arbitrary and unreasonable. That constitutional guarantee is not violated by a reasonable classification is
germane to the purpose of the law and applies to all those belonging to the same class. The purpose of the
law is to allow the emergence of younger blood in local governments. The classification in question being
pursuant to that purpose, it cannot be considered invalid "even if at times, it may be susceptible to the
objection that it is marred by theoretical inconsistencies.
Regarding Igot's petition, the court held that explicit is the constitutional provision that, in all
criminal prosecutions, the accused shall be presumed innocent until the contrary is proved, and shall
enjoy the right to be heard by himself and counsel. An accusation, according to the fundamental law, is
not synonymous with guilt. The challenged proviso contravenes the constitutional presumption of

innocence, as a candidate is disqualified from running from public office on the ground alone that charges
have been filed against him before a civil or military tribunal. It condemns before one is fully heard. In
ultimate effect, except as to the degree of proof, no distinction is made between a person convicted of acts
of disloyalty and one against whom charges have been filed for such acts, as both of them would be
ineligible to run for public office. A person disqualified to run for public office on the ground that charges
have been filed against him is virtually placed in the same category as a person already convicted of a
crime with the penalty of arresto, which carries with it the accessory penalty of suspension of the right to
hold office during the term of the sentence. And although the filing of charges is considered as but prima
facie evidence, and therefore, may be rebutted, yet, there is "clear and present danger" that because the
proximity of the elections, time constraints will prevent one charged with acts of disloyalty from offering
contrary proof to overcome the prima facie evidence against him. A legislative/administrative
determination of guilt should not be allowed to be substituted for a judicial determination. Igot's petition
was meritorious.

G.R. No. 93516, August 12, 1992


The People of the Philippines, plaintiff-appellee,
Vs.
Basilio Damaso @ Bernardo/Bernie Mendoza @ Ka Dado, accused-appellant
Facts:
Accused-appellant charged in an information of violation of PD 1866 in connection with the
crime of subversion assailed the legality of a search and seizure conducted at his house at night when he
was not around, on the ground that it violated constitutional rights against unreasonable search and
seizure.
Issue:
Whether or not a search on a house of a person without the owners presence is valid.
Ruling:
No. The search in the dwelling of the accused-appellant without his knowledge is a violation of
the constitutional immunity from unreasonable searches and seizures.
G.R. No. 136292, January 15, 2002
Rudy Caballes, petitioner
Vs.
Court of Appeals and People of the Philippines, respondents
Facts:
While on a routine patrol in Brgy. Sampalucan, Pagsanjan, Laguna, Sgt. Victorino Nocejo and
Pat. Alex de Castro spotted a passenger jeep unusually covered with kakawati leaves. Suspecting that the
jeep was loaded with smuggled goods, the two officers flagged down the vehicle. Being the driver of the
jeep, Caballes was asked by the officers as to what was loaded in the jeep, to which he did not respond, appearing pale
and nervous. The officers checked the cargo and discovered bundles of galvanized conductor wires
exclusively owned by National Power Corporation. Caballes and the vehicle with the high-voltage wires
were brought to the Pagsanjan Police Station, where he was imprisoned for 7 days. The trial court found
Caballes guilty of the crime of Theft of property. Upon appeal, the Court of Appeals affirmed the trial
courts judgment of conviction.

Issue:
Whether or not the evidence taken from the warrantless search is admissible against Caballes.
Ruling:
No; the evidence are not admissible in evidence.
The constitutional proscription against warrantless searches and seizures is not absolute, but
admits of certain exceptions. The situation in the case at bar does not fall under any of the accepted
exceptions.
1.

Search of a moving vehicle


The rules governing searches and seizures of moving vehicles have been liberalized for the
purposes of practicality. Obtaining a warrant for a moving vehicle is particularly difficult for want of
a specific description of the place, things, and persons to be searches. Also, it is not practicable to secure a
warrant because the vehicle can be quickly moved out of the jurisdiction in which the warrant must
be sought. Still, however, there must be probable cause to conduct such warrantless search. One form
of search of moving vehicles is the stop-and-search without warrant at checkpoints, which has been
declared as not illegal per se, for as long as it is warranted by the exigencies of public order and
conducted in a way least intrusive to motorists. A checkpoint may either be a mere routine inspection or it may involve
an extensive search. Routine inspections are not regarded as violative of an individuals right against
unreasonable search. The circumstances in this case, however, do not constitute a routine inspection.
They had to reach inside the vehicle, lift the leaves and look inside the sacks before they were able to see
the cable wires. When a vehicle is stopped and subjected to an extensive search, such a search would be
constitutionally permissible only if the officers have probable cause to believe that either the motorist is a
law-offender or they will find the instrumentality or evidence pertaining to a crime in the vehicle to be
searched. In this case, the officers flagged down the jeep because they became suspicious when they saw
that the back of the vehicle was covered with kakawati leaves, which, to them, was unusual and uncommon. The Court
believes that the fact that the vehicle looked suspicious simply because it is not common for such to be covered in kakawati
leaves does not constitute probable cause to justify a search without a warrant. In addition, there was no
tip or confidential information that could have backed up their search, as jurisprudence is replete with
cases where tipped information has become sufficient to constitute probable cause.

2.

Plain view doctrine


It is clear from the records that the cable wires were not exposed to sight because they were placed in sacks
andcovered with leaves. They had no clue as to what was underneath the leaves. Object was not in plain
view which could have justified mere seizure without further search.

3.

Consented search
At most, there was only implied acquiescence, a mere passive conformity, which is no consent
at all within the purview of the constitutional guarantee. Evidence is lacking that Caballes intentionally
surrendered his right against unreasonable searches.
G.R. No. L-68635 May 14, 1987
In the matter of proceedings for disciplinary action against Atty. Wenceslao Laureta, and
of contempt proceedings against Eva Maravilla-Illustre in G.R. No. 68635, entitled Eva
Maravilla-Illustre
vs.
Hon. Intermediate Appellate Court, et al.
Facts:
Maravilla Illustre wrote to the justices of the SC, complaining about the dismissal of her case (a
land dispute involving large estate) by a minute-resolution. Illustre claims that it was an unjust resolution
deliberately and knowingly promulgated by the 1st division, that it was railroaded with such hurry beyond
the limits of legal and judicial ethics. Illustre also threatened in her letter that, there is nothing final in
this world. This case is far from finished by a long shot. She threatened that she would call for a press
conference. Illustres letter basically attacks the participation of Justice Pedro Yap in the first division.

It was established that Justice Yap was previously a law partner of Atty. Ordonez, now the Solgen
and counsel for the opponents. The letters were referred to the SC en banc. The SC clarified that when the
minute-resolution was issued, the presiding justice then was not Justice Yap but Justice Abad Santos
(who was about to retire), and that Justice Yap was not aware that Atty. Ordonez was the opponents
counsel. It was also made clear that Justice Yap eventually inhibited himself from the case. Still, Illustre
wrote letters to the other justices (Narvasa, Herrera, Cruz), again with more threats to expose the kind
of judicial performance readily constituting travesty of justice. True to her threats, Illustre later filed
a criminal complaint before the Tanodbayan, charging the Justices with knowingly rendering an unjust
Minute Resolution. Justice Yap and Solgen Ordonez were also charged of using their influence in the First
Division in rendering said Minute Resolution. Atty. LAURETA was the counsel of Illustre. He circulate
copies of the complain to the press, without any copy furnished the Court, nor the Justices charged. It was
made to appear that the Justices were charged with graft and corruption. The Tanodbayan dismissed the
complaint. Now, the SC is charging them with contempt. They claim that the letters were private
communication, and that they did not intend to dishonor the court.
Issue:
WON privacy of communication was violated.
Held:
The letters formed part of the judicial record and are a matter of concern for the entire court.
There is no vindictive reprisal involved here. The Courts authority and duty under the premises is
unmistakable. It must act to preserve its honor and dignity from the scurrilous attacks of an irate lawyer,
mouthed by his client, and to safeguard the morals and ethics of the legal profession.

G.R. No. 130716 December 9, 1998


Francisco I. Chavez, petitioner
Vs.
Presidential Commission on Good Government (PCGG) and Magtanggol Gunigundo, (in
his capacity as chairman of the PCGG), respondents
Facts:
Petitioner Francisco I Chavez (in his capacity as taxpayer, citizen and a former government
official) initiated this original action seeking (1) to prohibit and enjoin respondents [PCGG and its
chairman] from privately entering into, perfecting and/or executing any agreement with the heirs of the
late President Ferdinand E. Marcos . . . relating to and concerning the properties and assets of Ferdinand
Marcos located in the Philippines and/or abroad including the so-called Marcos goldhoard"; and (2)
to compel respondent[s] to make public all negotiations and agreement, be they ongoing or perfected,
and all documents related to or relating to such negotiations and agreement between the PCGG and the
Marcos heirs."-Chavez is the same person initiated the prosecution of the Marcoses and their cronies who
committed unmitigated plunder of the public treasury and the systematic subjugation of the country's
economy; he says that what impelled him to bring this action were several news reports 2 bannered in a
number of broadsheets sometime in September 1997. These news items referred to (1) the alleged
discovery of billions of dollars of Marcos assets deposited in various coded accounts in Swiss banks; and
(2) the reported execution of a compromise, between the government (through PCGG) and the Marcos
heirs, on how to split or share these assets.-PETITIONER DEMANDS that respondents make public any
and all negotiations and agreements pertaining to PCGG's task of recovering the Marcoses' ill-gotten
wealth. He claims that any compromise on the alleged billions of ill-gotten wealth involves an issue of
"paramount public interest," since it has a "debilitating effect on the country's economy" that would be
greatly prejudicial to the national interest of the Filipino people. Hence, the people in general have aright
to know the transactions or deals being contrived and effected by the government.-RESPONDENT
ANSWERS that they do not deny forging a compromise agreement with the Marcos heirs. They claim,
though, that petitioner's action is premature, because there is no showing that he has asked the PCGG to
disclose the negotiations and the Agreements. And even if he has, PCGG may not yet be compelled to

make any disclosure, since the proposed terms and conditions of the Agreements have not become
effective and binding.-PETITIONER INVOKES Sec. 7 [Article III]. The right of the people to information
on matters of public concern shall be recognized. Access to official records, and to documents, and papers
pertaining to official acts, transactions, or decisions, as well as to government research data used as basis
for policy development, shall be afforded the citizen, subject to such limitations as may be provided by
law. Sec. 28 [Article II]. Subject to reasonable conditions prescribed by law, the State adopts and
implements a policy of full public disclosure of all its transactions involving public interest.RESPONDENT ANSWERS that the above constitutional provisions refer to completed and operative
official acts, not to those still being considered.
Issue:
Whether or not the Court could require the PCGG to disclose to the public the details of any
agreement, perfected or not, with the Marcoses.
Ruling:
WHEREFORE, the petition is GRANTED. The General and Supplemental Agreement dated
December 28, 1993, which PCGG and the Marcos heirs entered into are hereby declared NULL AND
VOID for being contrary to law and the Constitution. Respondent PCGG, its officers and all government
functionaries and officials who are or may be directly or indirectly involved in the recovery of the alleged
ill-gotten wealth of the Marcoses and their associates are DIRECTED to disclose to the public the terms of
any proposed compromise settlement, as well as the final agreement, relating to such alleged ill-gotten
wealth, in accordance with the discussion sembodied in this Decision. No pronouncement as to cost.

G.R. NO. L-22196June 30, 1967


Esteban Morano, Chan Sau Wan & Fu Yan Fun, petitioner-appellants
v.
Hon. Martiniano Vivo, Actibg Commissioner of Immigration, respondant-appellant
FACTS:
Chan Sau Wah, a Chinese citizen born in Fukein, China on Jan.6,1932, arrived in the Philippines on
Nov.23, 1961 to visit her cousin, Samuel Lee Malaps. She left in China two of her children bt the first
marriage. With her was Fu Yan Fun, her minor son also by the first marriage on Sept, 11, 1957.She and her
son were permitted only into the Phils. Under a temporary visitors visa for 2 months and after they
posted a cash bond of Php4,000.
On Jan. 24 1962,Chan Sau Wan married Esteban Morano, a native-born Filipino citizen, born to his
union on Sept. 16, 1962 was Esteban Morano, Jr.
To prolong their stay in the Phils., Chan Sau Wan & Fu Yan Fun obtained several extentions. The last
extention was Sept. 10,1962.
On Aug. 10 1962, the Commissioner ordered her and son to leave the countryon or before Sept. 10,
1962 w/ a warning that upon failure to do so, he will issue a warrant for their arrest and will cause the
confiscation of the bond. But instead of leaving the country, on Sept. 10 1962, Chan Sau Wan w/ her
husband Esteban Morano & Fu Yan Fun petitioned the court of First Instance of Mla. for Mandamus to
compel the Commissioner of Immigration to cancel petitioners alien certificate of registration,
prohibition to stop him from issuing a warrant for their arrest & preliminary injunction of confiscating

their cash bond & from issuing warrants of arrest pending resolution of the case. The trial court on Nov. 3,
1962, issued the writ of preliminary injunction prayed for, upon a Php. 2,000 bond.
ISSUE:
Whether or not Chan Sau Wan and her son Fu Yan Fun violated sec. 37(a) of the Phil. Immigration Act
and the Naturalization Law requisite.

COURT RULING:
The petition for mandamus and prohibition w/ respect to petitioner Chan Sau Wah was denied and
the judgement declaring her a citizen of the Phils. directed respondent to cancel her alien certificate of
registration & other immigration papers, and declaring the preliminary injunction w/ respect to her
permanency were all set aside. With respect to her citizenship, Chan Sau Wah didnt possessed all the
qualifications required by the Naturalization Law.
G.R. NO.78596
July 13, 1989
Lucien Tan Van Nghia, petitioner
v.
Hon. Ramon J. Liwag, Acting Commisioner of the Commision on Immigration and Deportation(CID), and
John Doer, agents of the CID, respondents
FACTS:
Lucien Tran Van Nghia is a French national w/ temporary address in Sta. Ana, Mla.He was in the
Phils. on Nov. 1, 1981 as a temporary visitor but his status was changed to an immigrant on Nov.16, 1984
based on his representation that he is financially capable & will invest in the Phils. but has not made any
investment & has engaged only in French tutoring & practice acupressure.
On May 28, 1987, CID Commissioner Liwag recieved a sworn complaint from a certain Dionisio G.
Cabrera. Jr. , as the landlord of Lucien Tran allegedly accused the latter of being an undesirable alien for
being hostile to public safety progress.
On June 1, 1987, Commisioner Liwag issued a mission order to a team of seven CID agents to locate
& bring Lucien to Intelligence Division for proper disposition & submission of report. But on the following
day, as the CID went to the residence of Lucien in Sta. Ana inviting him to the formers headquarter for
verification of his status but failed to obey instead lacked themselves w/ his lady companion in their
bedroom, refused indeed to talk to the agents. In such manner compelled them to sought assistance of
members of the Western Police District. But then again, Lucien adamantly refused to be taken in resulted
to the injury of both parties due to ensuing struggle until Lucien subdued & immediately taken to the CID
intelligence office.The warrant of arrest was issued on this day.
ISSUE:
Whether or not the arrest and detention of petitioner by the authority, the Immigration Commision
is legal.
COURT RULING:
The petition was dismissed. Petitioner Lucien Tran Van is not similarly restrained with the
condition in his bailbond is to obey by appearing and answering the complaint with will hold himself,

amenable to the courts orders and processes & after conviction will surrender in execution of such
judgement. The records therein show that formal deportation proceeding have been initiated against
Lucien before the Board of Special Inquiry of the CID.
The restrain has therefore become legal as well as the writ of habeas corpus has served its purpose.

G.R. NO. 115455


Aug. 25, 1994
Arturo M. Tolentino, et al
v.
Executive Sec., Sec. of Finance, Commission of Internal Revenue, et al
FACTS:
Petitioners Tolentino et al are questioning the constitutionality of RA 7716 otherwise known as the
Expanded Value Added Tax (EVAT) Law. Tolentino declare that the revenue bill did not originate
exclusively as required Art. VI s. 24 of the Constitution from the House of Representatives. The result of
the consolidation of 2 distinct bills, House No. 1197 & Senate No. 1630 must retain the essence of H.No.
1197. Albeit RA 7716 originated as House Bill 11197 and that it passed the 3 readings in the House of
Representatives, the same did not complete the 3 readings in Senate for after the 1st reading, the
2nd reading as well as the approval were on the same day by votes of 13 of its members w/ 1 abstain.
Deprived in fact the succinct scrutiny as a vital piece of legislation. That upon referral to the Senate Ways
& Means Committee thereafter Senate passed its own version known as Senate Bill 1630. Petitioner
declare that what Senate could have done is amend HB 11197 by striking out its text and substituting it w/
the text of Senate Bill 1630 in such way the bill remains a House Bill and the Senate version just becomes
the text of the House Bill.
ISSUE:
Whether or not Expanded Value Added Tax Law is constitutionally valid.
Whether the Senate committed grave abuse of discretion by passing its own version of the Bill.

COURT RULING:
The Supreme court rejected the challenge, holding that such consolidation was consistent with the
power of the Senate to propose or concur with amendments to the version originated in the House of
Representatives. According to the 9 justices, the constitution meant that the initiative must come from
the House of Representatives. There were instances indeed before where Senate passed its own version
rather than having the HoR version as far as revenue and other such bills are concerned. This has always
been accepted in terms of amendments by substitution. There is no showing that it would make a
significant difference. What Tolentinos concerned was basically a matter of form but failed to established
a substantial difference on both Bills.
Therefore, R.A. No. 7716 is not unconstitutional. It is not the only instance in which the senate
proposed an amendment to a House revenue bill by enacting its own version. This has happened before
twice during the eight Congress.
In like manner on the question if theres grave abuse of discretion though Art. VI sec. 24, the
Supreme Court ruled as theres none.

G.R.NO. 136066-67
Feb. 3, 2003
People of the Phils., plaintiff-appelle
v.
Binad Sy Chua, accused-apellant

FACTS:
On Sept. 21, 1996 in the city of Angeles, Binad Sy Chua was accused of a criminal case, illegal
possession of plastic bags containing methamphetamine Hydrocloride known as SHABU weighing
1,955.815 grams w/c is a regulated drug w/o any authority. Another criminal act was charged on him on
the same date and place of willfully & unlawfully in his possession and under his controla 20 pcs.of live .
22 cal. Ammunitions w/o the license or permit to carry them.
ISSUE:
Whether or not Binad Sy Chua is guilty of criminal cases nos.96-507 & 96-513.
COURT RULING:
The decision of the RTC of Angeles City in criminal cases nos. 96-507 & 96-513 convicting the
accused-appellant Binad Sy Chua was reversed and set aside in violation of sec. 16 Art.III, R.A. No. 6425,
sentencing the latter to suffer the penalty of reclusion perpetua & to pay a fine of Php 1000000. He was
acquitted on the ground of reasonable doubt. The lack of merit of his arrest then was reviewed.
Consequently, he is ordered to be released from the custody unless hes lawfully held for another crime.

ICHONG
VS.
HERNANDEZ
Facts:
The Congress of the Philippines enacted the act which nationalizes theretail trade business, Republic Act
No. 1180 entitled An Act to Regulate theRetail Business, prohibiting aliens in general to engage in retail
trade in ourcountry.Petitioner, for and in his own behalf and on behalf of other alien
residents,corporations and partnerships adversely affected by the provisions of RA No.1180, brought this
action to obtain a judicial declaration that said Act isunconstitutional.
Issue:
Whether Congress in enacting R.A. No. 1180 violated the UN Charter, theUN Declaration of Human
Rights and the Philippine-Chinese Treaty of Amity.
Held:
The UN Charter imposes no strict or legal obligations regarding the rightsand freedom of their subjects,
and the Declaration of Human Rights containsnothing more than a mere recommendation, or a common
standard of achievement for all peoples and all nations. The Treaty of Amity between the Republic of the
Philippines and the Republic of China guarantees equality of treatment to the Chinese nationals upon the
sameterms as the nationals of any other country. But the nationals of China are notdiscriminated against
because nationals of all other countries, except those of the United States, who are granted special rights
by the Constitution, are allprohibited from engaging in the retail trade.But even supposing that the law
infringes upon the said treaty, the treaty isalways subject to qualification or amendment by a subsequent
law, and the samemay never curtail or restrict the scope of the police power of the State

THE PEOPLE OF THE PHILIPPINE ISLANDS,


plaintiff-appellee,
vs.
KAGUI MALASUGUI,
defendant-appellant.
G.R. No. L-44335 July 30, 1936DIAZ,
:Facts:
On March 5, 1935, Tan Why, a Chinese merchant, a resident of Cotabato, a victim of robbery was found lying on the
ground,with several wounds in the head, on a path leading to the barrio of Carcar, Cotabato. Shortly before
the victims death he was ableto mention the appellants first name, Kagui, when he was asked about
assailant.Appellant was later searched by the investigating police, without opposition or protest on his
part, and it was discovered that healso had the victims pocketbook, containing P92 in bills, the victims
identification card and a memorandum of amounts with someChinese characters. In one of the pockets of
his pants was found some change, making the total amount of money found in hispossession P92.68. The
said search was conducted after the appellant had voluntarily produced the bracelets Exhibit A and
placedthem on Lieutenant Jacaria's table, because, upon being asked if he had anything, he tremblingly answered in the
negative. The appellant testified at the trial that Lieutenant Jacaria and Sergeant Urangut had forcibly and
through intimidation taken fromhim the bracelets the pocketbook and all the money which he and that, but for the
printing thereon, the identification card found inthe pocketbook then was blank and there was no memorandum
of the kind, in Tan Why's handwriting, inside the pocketbook,'
Issue:
WON the search and seizure conducted on the accused legal?Article III, section 1(3), of the 1935 Constitution
:The right of the people to be secure in their persons, houses, papers, and effectsagainst
unreasonable search and seizures shall not be violated, and no warrant shall issue, but upon probable
cause, supported byoath or affirmation, and particularly describing the place to be searched and the
persons or things to be seized," contains noprohibition of arrest, search, or seizure without a warrant, but only against
"unreasonable" searches and seizures.
SC ruling:
Yes. The SC held that When the search of the person detained or arrested and the seizure of the
effects found in hispossession are incidental to an arrest made in conformity with the law, they cannot be considered
unreasonable, much lessunlawful. To hold that no criminal can, in any case, be arrested and searched for the evidence and
tokens of his crime without awarrant, would be to leave society, to a large extent, at the mercy of the shrewdest, the most expert,
and the most depraved of criminals, facilitating their escape in many instances. The record shows that before
proceeding with the trial in the lower court, the appellant asked for the return of saideffects to him on the
ground that they were unlawfully taken away from him. Leaving aside the foregoing considerations,
histestimony cannot prevail against nor is it sufficient to counteract that of the government witnesses,
Lieutenant Jacaria andSergeant Urangut, who testified that when Lieutenant Jacaria asked him what
other things he carried, after having voluntarilyplaced the two pairs of bracelets, Exhibit A, on the
table, and Sergeant Urangut felt his body, he did not show the least opposition. It follows,
therefore, that the lower court committed no error in accepting as evidence the items taken from
theaccused, not only because the appellant did not object to the taking thereof from him when searched,
but also because theeffects found in his possession of a person detained or arrested are perfectly
admissible as evidence against him, if theyconstitute the
corpus delicti
or are pertinent or relevant thereto. It is certainly repugnant to maintain the opposite
viewbecause it would amount to authorizing the return to the accused of the means of conviction seized

from him, notwithstandingtheir being eloquent proofs of crime, for him to conceal, destroy or otherwise dispose of, in
order to assure his impunity.
Pasion Vda. De Garcia vs. Locsin
Facts:
Mariano G. Almeda, an agent of the Anti-Usuary Board, obtained from the justice of the peace of
Tarlac, a search warrant commanding any officer of the law to search the person, house or store of the
petitioner at Victoria, Tarlac, for certain books, lists, chits, receipts, documents and other papers relating
to her activities as usurer. The search warrant was issued upon an affidavit given by the said Almeda.
On the same date, the said Mariano G. Almeda, accompanied by a captain of the Philippine Constabulary,
went to the office of the petitioner in Victoria, Tarlac and, after showing the search warrant to the
petitioners bookkeeper, Alfredo Salas, and, without the presence of the petitioner who was ill and
confined at the time, proceeded with the execution thereof
The papers and documents seized were kept for a considerable length of time by the Anti-Usury Board
and thereafter were turned over by it to the respondent fiscal who subsequently filed six separate criminal
cases against the herein petitioner for violation of the Anti-Usury Law.
The legality of the search warrant was challenged by counsel for the petitioner in the six criminal cases
and the devolution of the documents demanded. The respondent Judge denied the petitioners motion for
the reason that though the search warrant was illegal, there was a waiver on the part of the petitioner.
HELD:
Freedom from unreasonable searches and seizures is declared a popular right and for a search
warrant to be valid, (1) it must be issued upon probable cause; (2) the probable cause must be
determined by the judge himself and not by the applicant or any other person; (3) in the
determination of probable cause, the judge must examine, under oath or affirmation, the complainant and
such witnesses as the latter may produce; and (4) the warrant issued must particularly describe the place
to be searched and persons or things to be seized.
In the instant case the existence of probable cause was determined not by the judge himself
but by the applicant. All that the judge did was to accept as true the affidavit made by agent
Almeda. He did not decide for himself. It does not appear that he examined the applicant and his
witnesses, if any. Even accepting the description of the properties to be seized to be sufficient and on the
assumption that the receipt issued is sufficiently detailed within the meaning of the law, the properties
seized were not delivered to the court which issued the warrant, as required by law.
Instead, they were turned over to the resp. provincial fiscal & used by him in building up cases against
petitioner. Considering that at the time the warrant was issued, there was no case pending against the
petitioner, the averment that the warrant was issued primarily for exploration purposes is not without
basis.
Filed under Constitution and tagged arrest, Bill of rights, case digest, examination of witness, se
RAMIREZ V CA
7NOV
G.R. No. 93833 |
Facts:

A civil case damages was filed by petitioner Socorro Ramirez in the Quezon City RTC alleging that
the private respondent, Ester Garcia, in a confrontation in the latters office, allegedly vexed, insulted and
humiliated her in a hostile and furious mood and in a manner offensive to petitioners dignity and
personality, contrary to morals, good customs and public policy.
In support of her claim, petitioner produced a verbatim transcript of the event and sought
damages. The transcript on which the civil case was based was culled from a tape recording of the
confrontation made by petitioner.
As a result of petitioners recording of the event and alleging that the said act of secretly taping the
confrontation was illegal, private respondent filed a criminal case before the Pasay RTC for violation of
Republic Act 4200, entitled An Act to prohibit and penalize wire tapping and other related violations of
private communication, and other purposes.
Petitioner filed a Motion to Quash the Information, which the RTC later on granted, on the
ground that the facts charged do not constitute an offense, particularly a violation of R.A. 4200.
The CA declared the RTCs decision null and void and denied the petitioners MR, hence the
instant petition.
Issue:
W/N the Anti-Wiretapping Act applies in recordings by one of the parties in the conversation
Held:
Yes. Section 1 of R.A. 4200 entitled, An Act to Prohibit and Penalized Wire Tapping and Other
Related Violations of Private Communication and Other Purposes, provides:
Sec. 1. It shall be unlawful for any person, not being authorized by all the parties to any private
communication or spoken word, to tap any wire or cable, or by using any other device or arrangement, to
secretly overhear, intercept, or record such communication or spoken word by using a device commonly
known as a dictaphone or dictagraph or detectaphone or walkie-talkie or tape recorder, or however
otherwise described.
The aforestated provision clearly and unequivocally makes it illegal for any person, not authorized
by all the parties to any private communication to secretly record such communication by means of a tape
recorder. The law makes no distinction as to whether the party sought to be penalized by the statute ought
to be a party other than or different from those involved in the private communication. The statutes
intent to penalize all persons unauthorized to make such recording is underscored by the use of the
qualifier any. Consequently, as respondent Court of Appeals correctly concluded, even a (person) privy
to a communication who records his private conversation with another without the knowledge of the
latter (will) qualify as a violator under this provision of R.A. 4200.
A perusal of the Senate Congressional Records, moreover, supports the respondent courts
conclusion that in enacting R.A. 4200 our lawmakers indeed contemplated to make illegal, unauthorized

tape recording of private conversations or communications taken either by the parties themselves or by
third persons.
The nature of the conversations is immaterial to a violation of the statute. The substance of the
same need not be specifically alleged in the information. What R.A. 4200 penalizes are the acts of
secretly overhearing, intercepting or recording private communications by means of the devices
enumerated therein. The mere allegation that an individual made a secret recording of a private
communication by means of a tape recorder would suffice to constitute an offense under Section 1 of R.A.
4200. As the Solicitor General pointed out in his COMMENT before the respondent court: Nowhere (in
the said law) is it required that before one can be regarded as a violator, the nature of the conversation, as
well as its communication to a third person should be professed.
Petitioners contention that the phrase private communication in Section 1 of R.A. 4200 does not
include private conversations narrows the ordinary meaning of the word communication to a point of
absurdity. The word communicate comes from the latin word communicare, meaning to share or to
impart. In its ordinary signification, communication connotes the act of sharing or imparting
signification, communication connotes the act of sharing or imparting, as in a conversation, or signifies
the process by which meanings or thoughts are shared between individuals through a common system of
symbols (as language signs or gestures)
These definitions are broad enough to include verbal or non-verbal, written or expressive
communications of meanings or thoughts which are likely to include the emotionally-charged exchange,
on February 22, 1988, between petitioner and private respondent, in the privacy of the latters office. Any
doubts about the legislative bodys meaning of the phrase private communication are, furthermore, put
to rest by the fact that the terms conversation and communication were interchangeably used by
Senator Taada in his Explanatory Note to the Bill.
VILLEGAS VS HIU CHIONG
Facts:
Pao Ho is a Chinese national employed in the City of Manila. On 27 March 1968, then Manila
mayor Antonio Villegas signed Ordinance No. 6537. The said ordinance prohibits foreign nationals to be
employed within the City of Manila without first securing a permit from the Mayor of Manila. The permit
will cost them P50.00. Pao Ho, on 04 May 1968 filed a petition for prohibition against the said Ordinance
alleging that as a police power measure, it makes no distinction between useful and non-useful
occupations, imposing a fixed P50.00 employment permit, which is out of proportion to the cost of
registration and that it fails to prescribe any standard to guide and/or limit the action of the Mayor, thus,
violating the fundamental principle on illegal delegation of legislative powers. Judge Arca of Manila CFI
ruled in favor of Pao Ho and he declared the Ordinance as being null and void.
ISSUE:
Whether or not there is undue delegation to the Mayor of Manila.
HELD:

The decision of Judge Arca is affirmed. Ordinance No. 6537 does not lay down any criterion or
standard to guide the Mayor in the exercise of his discretion. It has been held that where an ordinance of a
municipality fails to state any policy or to set up any standard to guide or limit the mayors action,
expresses no purpose to be attained by requiring a permit, enumerates no conditions for its grant or
refusal, and entirely lacks standard, thus conferring upon the Mayor arbitrary and unrestricted power to
grant or deny the issuance of building permits, such ordinance is invalid, being an undefined and
unlimited delegation of power to allow or prevent an activity per se lawful. Ordinance No. 6537 is void
because it does not contain or suggest any standard or criterion to guide the mayor in the exercise of the
power which has been granted to him by the ordinance. The ordinance in question violates the due
process of law and equal protection rule of the Constitution.

You might also like